Social Exam I Combined

Réussis tes devoirs et examens dès maintenant avec Quizwiz!

When most people think of the character Captain Kirk from the original Star Trek series, they think of the line, "Beam me up, Scotty!" But that line of dialogue never appeared on the show. This example illustrates that memory is often A) a product of our schemas. B) influenced by our peers. C) more accurate in thinking about real people than about fictitious characters. D) completely random. E) inconsistent with our schemas.

A) a product of our schemas.

Dispositional (internal) attribution

-explain a behavior in terms of a person's disposition (character, morals, etc) -people tend to make these attributions for others' internal behavior

Fundamental Attribution (FAE)

-just observers and involves BOTH attribution types -when people underestimate role of disposition; underestimate situational influences -ie when cut off driving you jump to conclusions and call person mean names

"we respond not to reality as it is but to reality as we construe it"

-our reality is constructed by our biased -perceptions -judgements -explanations -expectations

Which of the following is an example of a situation which should evoke a sense of loss of control?

Alana's landlord will not allow her to plant flowers in her yard or hang pictures on her walls.

When using the experimental method, it is crucial that in each condition ___.

All extraneous events are identical.

The major difference between sociology and social psychology is A. the level of analysis used. B. the methods of research used. C. the ability to apply knowledge to address social problems. D. the kinds of topics studied.

A. The level of analysis used

Research by social psychologists has demonstrated that the more unpleasant the hazing required of initiates to join a group, the more they actually like the group. What do these findings suggest about a behaviorist approach?

Although they can explain much, behaviorists have trouble explaining some social behaviors.

One of Kurt Lewin's boldest intellectual contributions to social psychology was ___.

Applying Gestalt principles to social perception.

Professor Takahami is a personality psychologist interested in divorce. Which question is she most likely to investigate?

Are some types of people more likely to divorce than others?

Professor Swenson is interested in university students' reactions to the death of a popular rock star. For two weeks, Professor Swenson spends one hour a day in a popular cafeteria, inconspicuously listening to students, joining in their conversations when the topic of the dead rock star comes up, and recording what the students have to say. Professor Swenson is conducting ________ research. A) interactive experimentation B) ethnographic C) historical D) correlational E) archival

B) ethnographic

Ethnography can be defined as the method by which researchers attempt to understand a group or culture by a. independently observing and coding a set of data. b. examining the accumulated documents of that culture, such as newspapers. c. asking questions of a representative sample of people by means such as telephone interviews. d. observing it from the inside, without imposing any preconceived notions.

D

If two variables have no correlation with one another, this means that a. you can only predict one of the two variables. b. when one is higher, the other is lower. c. they are probably very similar. d. you cannot predict one from the other.

D

When Jessica plays checkers with her younger sister, she lets her sister win. When she plays with her older brother, she does everything she can to beat him. A social psychologist would suggest that: A: Jessica's personality is unstable. B: Jessica is responding to different social situations. C: Jessica is blindly obedient to the rules of the game. D: Jessica is ambivalent in how she feels about her siblings.

B: Jessica is responding to different social situations.

We can form impressions of people we interact with rather quickly and effortlessly. We do this by engaging in an automatic analysis of our environments, based on ___.

Biological signals we receive from other people.

In a controlled experiment, Rosenthal and Jacobson (1968) informed grade school teachers that some of their students (called "bloomers") would show great academic improvement in the upcoming year. In reality, the "bloomers" were chosen randomly by the researchers, and were no smarter than any of the other students. Which of the following best describes the results of this study? At the end of the year...

Bloomers improved more on an IQ test than did non-bloomers.

Which of the following correlations shows the strongest relation between two variables? A) 0.68 B) -0.19 C) -0.74 D) 0.07 E) 0.00

C) -0.74

Professor Yarnofsky wonders whether recent acts of airline terrorism have made the public more fearful of airline flight. He secures records of the amount of flight insurance that people have purchased via machines at the airport, and compares the records of insurance purchases before the last hijacking to purchases after the last hijacking. Professor Yarnofsky has employed a(n) ________ research method. A) correlational B) participant observation C) archival D) systematic observation E) experimental

C) archival

According to the authors of your text, when people use automatic thinking, they usually can size up new situations: A. quickly and inaccurately. B. slowly and accurately. C. quickly and accurately. D. slowly and inaccurately.

C. quickly and accurately.

People tend to have ________ emotional reactions the ________ it is to "mentally undo" an outcome. A.stronger; harder B. weaker; easier C. stronger; easier D. weaker; harder

C. stronger; easier

In trying to decide which of two classmates is smarter, you use the rule, "the faster people talk, the smarter they are," so you choose the classmate who talks faster. You have applied a ________ to decide who is smarter. A: self-fulfilling prophecy B: cultural truism C: judgmental heuristic D: systematic judgment

C: judgmental heuristic

Robert is interested in the ways in which romantic partners think about each other. His research most neatly fits the realm of _____. A:sociology B:personality psychology C:social cognition D:behaviorism

C: social cognition

The "father" of modern experimental social psychology, Kurt Lewin, pointed out the importance of understanding how people ___ their social environments.

Construe

According to information from the registrar's office, Lee has discovered that people who score higher on the SAT tend to have a higher GPA in their first year of college. Lee has used a(n) ___ research method.

Correlational

When psychologists observe something in everyday life that is interesting, how is such information useful in research? A: It is used to prove hypotheses that were based on previous theory. B: It serves no good purpose beyond stirring up controversy. C: It serves as information that can be used in theoretical debates about human behavior. D: It can stimulate researchers to develop a hypothesis and design studies to test it.

D: It can stimulate researchers to develop a hypothesis and design studies to test it.

________ and ________ are the hallmarks of the experimental method, setting it apart from the observational and correlational methods. A) Factorial designs; dependent variables B) Probability levels; meta-analysis C) Random sampling; control over extraneous variables D) Random assignment; probability levels E) Control over extraneous variables; random assignment

E) Control over extraneous variables; random assignment

Marvin flips a coin 5 times, and gets 5 "heads" in a row. Now Marvin thinks he is almost sure to get a "tails" on the next flip. According to the text, Marvin A) is guilty of biased sampling. B) is correct. C) needs therapy. D) is falling victim to the availability bias. E) is making an error due to the representativeness heuristic.

E) is making an error due to the representativeness heuristic.

Assil has an educated guess about the social behavior of teenagers when their high school team wins the football game. That means that she has a(n) ___ about how the teens behave.

Hypothesis

"Computer nerds" are widely held to be socially awkward, introverted, unpopular, and obsessed with anything related to technology. This description represents a(n) ___.

Implicit personality theory

Josh and his friends have just started up a new fraternity on campus, and want to recruit men who will be loyal and love it. Based on the self-esteem approach, which initiation strategy would you recommend to Josh?

Make them suffer a little: ride backwards in elevators all week.

Eva runs an unscrupulous organization that attempts to scam money from people while posing as a charity. Based on the research by Shariff and Norenzayan (2007), the best strategy for doing this involves ___.

Mentioning fairness several times in the brochures.

Aoi is from Japan. If you want her to think analytically, what will you have to do?

Prime analytical thinking with pictures or a story

In reading descriptions of the behaviors of the New York University students who participated in Latané and Darley's (1968) experiment on bystander intervention, you wonder whether a similar study conducted in Tokyo in 1968 would have yielded the same results as those participants from NYU. In essence, your question is, "Will these results generalize across ________?"

People

The authors of your text discuss three snares in peoples' ability to reason accurately. All of the following are

People are seldom motivated to be accurate.

Why do researchers go to great lengths to create cover stories and devise elaborate situations in their studies to cover up the true purpose of the study?

People cannot always accurately predict how they would respond in a situation.

According to the authors of your text, one of the major problems with relying solely on evolutionary theory as the only explanation for social behavior is that ___.

Psychologists are making assumptions about the conditions that existed long ago that produced reproductive advantages for certain genes.

According to the authors of your text, the ultimate test of an experiment's external validity is ___.

Replications.

The ________ heuristic refers to the mental shortcut whereby people classify a person or thing based on how similar it appears to be to the typical case

Representativeness

When Becky meets her new roommate, she sees that she has her hair dyed blue and has decorated her side of the room with lots of handmade ceramic pots and paintings. She then assumes that her roommate has multiple piercings and listens to punk rock. She has used the ________ heuristic in making this assumption.

Representativeness

Which of the following is the best illustration of a "self-fulfilling prophecy"?

Ryan heard that Grace is opinionated; upon meeting her, he strikes up a conversation about politics and finds that she is willing to express her opinion.

According to social cognition researchers, we sometimes have difficulty making sense of the social world because we

lack the stamina to seek out all the relevant facts.

Causal conclusions

made through use of experimental studies

If your professor asks you to list ten ways to improve the course and you cannot do it, how will that affect your course ratings overall?

more positive

Social psychological research has convincingly demonstrated that the ________ the initiation to join a group, the ________ the initiates like the group.

more severe; more

Based on the text, which of the following is not a motive that influences people's thoughts and behaviors?

need to nurture others

Tony has been doing research on age and aggression. He has discovered that the older a person gets, the less likely he or she is to aggress against another person. What kind of relationship best describes Tony's findings?

negative correlation

A behaviorist would be most likely to focus on the role of ________ in influencing behavior.

objective properties of the environment

When there is no other information available, it is ________ reasonable to rely on the representativeness heuristic to make a quick judgment.

perfectly

The word "construal" refers to

personal interpretations.

Ethan and Heather see a man stumbling around as he walks down the street. Ethan, who belongs to Alcoholics Anonymous, thinks the man is drunk, but Heather, who just watched a TV special on Michael J. Fox, thinks the man has Parkinson's Disease. These differing interpretations of the same behavior seem to be caused by Ethan and Heather's differences in

accessibility

The availability heuristic is based on the principle of using ________ as a cue to probability. Correct! accessibility

accessibility

Attribution

an explanation for a behavior

variable

anything being tested or something that varies

In trying to make sense of the mass suicide in Jonestown, a Gestaltist would probably

ponder the subjective meaning of the act to Jim Jones's followers.

One strength of observational research methods is that they can be useful for describing behavior. However, these methods do not allow researchers to ________ human behavior.

predict and explain

Complete the analogy: correlation: __________ as experiment: __________.

relate; cause

Complete the following analogy, based on information from your text: Behaviorism: __________:: Gestalt Psychology: ____________.

rewards; perceptions

Which of the following pairs of perspectives and statements is mismatched?

social cognition; self-justification

social psychology's relatives

sociology anthropology economics neuroscience/ biology clinical psych cognitive psych developmental psych personaliy psych

All of the following EXCEPT the ________ are examples of peripheral cues that might influence people taking the peripheral route to persuasion.

strength of a persuasive argument

When the need for control is unfulfilled, people believe

that they have little or no influence over whether good or bad things happen to them.

If the research on priming and metaphors is correct, what should your organization do if it wants to raise money for its service project? a. make sure there are lights on the table where volunteers sit to raise money b. make sure volunteers offer candy at the table where they sit to raise money c. make sure there are pleasant smells around the table where volunteers sit to raise money d. make sure there is loud music at the table where the volunteers sit to raise money

b.

Instead of considering every single brand and variety of pasta sauce at the grocery store, most people make quick decisions about which one to purchase. This is an example of how people use a. self-fulfilling prophecies. b. judgmental heuristics. c. controlled processes. d. priming.

b.

It is ________ to miss a bus by five minutes than to miss it by an hour because ________. a. less distressing; you don't have to wait as long for the next bus b. more distressing; it's easier to imagine what you might have done to catch the bus c. less distressing; it's easier to imagine what you might have done to catch the bus d. more distressing; images of the people waiting for you are more available in memory

b.

James has a goal of being able to run a marathon by next year. When he sees a woman dart out of a store and start running he immediately thinks, "Oh, she must be a runner too!" rather than, "She's a shoplifter trying to get away." Which of the following is the best explanation for why one schema was accessible rather than another? a. chronic accessibility due to past experience b. a current goal c. base-rate information d. applicability of priming

b.

Jason cannot stop thinking about the fact that because he turned in his term paper late, he missed getting an "A" in the class. He repetitively focuses on this negative part of the class; in other words, his counterfactual thinking has turned into a. thought suppression. b. rumination. c. heuristics. d. controlled thinking.

b.

Julie sees an Asian couple at the New York City Zoo. According to __________, she may assume they are foreigners touring the area, but according to __________, she should assume that given the low tourism rate in her town, they are actually local residents. a. base rate information; availability heuristic b. representativeness heuristic; base rate information c. availability heuristic; base rate information d. representativeness heuristic; availability heuristic

b.

Jacob's friend, Tom asks him, "Do you think I'm short-tempered?" Jacob had never really thought about how short-tempered or calm Tom was until he had asked. Nonetheless, Jacob was able to provide him a quick answer. What social cognitive process was most likely involved in this judgment? a. self-fulfilling prophecy b. controlled thinking c. base rate heuristic d. availability heuristic

d.

Jennifer was walking along the street when she saw a man run out of a convenience store clutching on to a bag. The owner of the store runs out and shouts for the man to stop and come back. Jennifer immediately assumes that there has been a robbery, not that the man was in a hurry and forgot his change. How did she fill in this gap in her knowledge of the situation? a. She was using the base rate heuristic. b. She considered self-fulfilling prophecies. c. She was using racial profiling. d. She was relying on a schema.

d.

Jake had a hypothesis about the outcome of Ross and Samuels's (1993) study about the "Wall Street Game" and the "Community Game." Jake hypothesized that the players would respond based on their personalities, not just the name of the game they played. His hypothesis is most likely based on which tendency?

fundamental attribution error

In the "Wall Street Game" and "Community Game" research, the personality traits of the student participants

had no measurable effect on the participants' behavior.

Knowing the experiment's purpose can alter participants' behavior. This is a major problem with

informed consent.

After seeing the movie Fargo, Craig firmly believes that Minnesotans and North Dakotans are the goofiest people in the United States. You believe that Minnesotans and North Dakotans are no goofier than people any place else, and you would like to get Craig to change his judgment. To do this, you might a. ask him to consider the opposite point of view. b. ask him to think more carefully about his judgment. c. tell him to stop being irrational. d. tell him that he needs a law school course.

a.

Aoi is from Japan. If you want her to think analytically, what will you have to do? a. prime analytical thinking with pictures or a story b. prime holistic thinking with pictures or a story c. it doesn't matter what you do, she cannot think analytically because of her culture d. tell her you want her to think analytically

a.

Applying the lessons learned from Rosenthal and Jacobson, if you wanted your child to succeed, what would be the best nickname for him? a. Champ b. Buddy c. Pipsqueak d. Rascal

a.

Based on your reading of Chapter 3 (Social Cognition), who would you predict would be more frustrated: Katie, who missed an "A" by one point, or James, who missed an "A" by five points? a. Katie, because it's easier for her to imagine getting one more test item right b. James, because it's harder for him to imagine an alternative outcome c. Katie, because she and James studied together d. James, because he and Katie studied together

a.

The shortcuts that people use to make judgments and decisions quickly and efficiently are called

judgmental heuristics

If two variables have no correlation with one another, this means that

you cannot predict one from the other.

All of the following are examples of social influence except

you perceive the bathwater as hot when you first get in, but don't notice the heat ten minutes later.

A teenager wants to borrow the family car to go to an out-of-town concert. Her parents resist the idea. Based on the results of the research study on the effects of generating different numbers of examples of behavior described in your text, the teenager in question is most likely to view her parents as stubborn and unreasonable when they say to her, ___.

"Give us one reason why we shouldn't worry ourselves to death."

Leslie cannot say "no" to anyone. Consequently she takes on too much, and doesn't fulfill her commitments as promptly as she might. Assume that when asked to explain these behaviors, Leslie is motivated to be accurate in her self-perceptions. What would she be most likely to say?

"I guess I'm a bit of a pushover, and take on more than I can responsibly handle."

You might have heard the old Groucho Marx quip, "I wouldn't belong to any club that would have me for a member." From a self-esteem perspective in social psychology, what would be a more accurate-if less humorous-statement?

"I would most want to belong to a club that made me suffer to get into it."

Shanika is an executive, and asked her assistant repeatedly to make some copies for her. The assistant repeatedly failed to successfully complete the assignment. If Shanika is thinking like a social psychologist about this situation, what is she most likely to think about her assistant?

"Perhaps my assistant is under stress from something else."

When Mark goes to give Sven a congratulatory pat on the back after a job well done, he jumps and turns away. Mark has just read Chapter 1, and fancies himself something of a Gestalt psychologist. What is he likely to think about Sven's behavior?

"Sven must have interpreted my gesture differently than I intended."

Rahid's server in the restaurant just can't seem to get his order right. If Rahid has just read Chapter 1 on social influence and is thinking more like a social psychologist than before, what would he be most likely to think?

"This person must have had a bad morning."

Suppose you have invited a new acquaintance over to your apartment and want to make a good impression; in other words, you want this person to like you. Which of the following should you do? a. Serve the person a warm drink and hope that he or she holds it in their hands while you are talking to him or her. b. Serve the person a cold drink and hope that he or she holds it in their hands while you are talking to him or her. c. Bake some bread before the person comes over so that the apartment smells nice. d. Serve the person a snack on a very heavy plate.

(A)

Which of the following is the best summary of research on automatic goal pursuit? a. People can only select which goals to work toward using controlled thinking. b. People often pursue goals that have been recently primed, without realizing that that is why they are pursuing the goal. c. People often pursue goals that have been recently primed, but only if they are consciously aware that the goal has been primed. d. People never choose their goals consciously; they only pursue automatically primed goals.

(B)

Over Thanksgiving break, your parents ask you if you can think of 12 reasons why your college is better than its archrival. You find it hard to come up with many reasons and end up thinking, "Hmm, maybe the schools aren't all that different." Which of the following mental strategies did you probably use to reach this conclusion? a. The representativeness heuristic b. Base rate information c. The anchoring and adjustment heuristic d. The availability heuristic

(D)

After losing a Little League softball game between the Rockets and the Tigers, what is the coach of the losing Rockets likely to say?

"Boy, those Tigers have never played better than they did today."

Availability Heuristic

-ease of information that is readily available influences; how common we think events are (shark vs. plane), how we think of ourselves (geography) -if something is easy to think of, we assume that it is more common and more likely to happen -pretty accurate most of the time -problem" used w/ environment, people, objects that we know very little about

Error Management Theory

-even our errors are adaptive when assessed in a social context -people avoid the more costly error -ie used to explain why men quickly (nd often errneously) infer sexual interest from women (eg smiling, and arm touching) -Type 1 Error: rejection or embarrassment -Type 2 Error: Missed chance to reproduce

Situational (external) attribution

-explain a behavior as influenced by the situation (ie converting to social norms like standing quietly in line at bank facing front desk) -people tend to use this attribution for their own external behaviors

Type 2 Error

-false negative -fail to notice harmful disposition -'side with skepticism' -keep running -error cost= snake bite -cost= potentially high for some negative traits

Type 1 Error

-false positive: assume negative disposition -cost= lost opportunity for social exchange (low) -'play it safe' -stop running -error cost= end run, look foolish

So what are people good at?

-great for solving SOCIAL problems

Actor/ observer (AOE)

-has to do with 2 sides of 1 event -actors focus on external or situational attributions -observers focus on internal dispositional factors -difference in perceptions of at least 2 people

People conserve their mental resources though...

-heuristics -biases

The Wason Selection Task

-if a card shows an even number on one side, then it must have a vowel on the other side -much much easier when framed in terms of a social contract -the mind developed to understand social environments -understanding social rules -not so adaptive to match vowels to even numbers -very helpful to enforce social rules ( ie underage drinking) -people are designed to survive and thrive in social environments

Heuristics

-mental shortcuts -happen automatically -efficient strategy for solving problems or answering questions with small amount of information -provides a 'good enough' answer most of the time -prone to certain types of errors

automatic thinking (system 1)

-more impulsive -easy, not intentional, not conscious -guy reactions, JUDGMENTS, quick assumptions, default mode of thinking

how do people think?

-not purely rational information processors -drive for efficiency leads to heuristics, attributions, attentional biases, etc. -we're geared to process social information and rules -surprisingly accurate with first impressions

Representativeness Heuristic

-perceive/judge something as being likely if it resembles the typical case -ie similar to known events, categories, groups -types of people, stereotypes and categories -given small amt info to make quick judgments -'snap judgements'

what is social psychology

-scientific study of how people think about, influence, and relate to one another -the goal is to uncover/discover universal laws of human behavior, which is why cross-culture research is often essential

Humans are cognitive misers and make attributions because.....

-seek rationale for 'why' -gives justification -time saver for future events (ie next time you go to bank you already accustomed to social norms) -way to become accustomed in society

controlled thinking (system 2)

-takes effort -purposeful and conscious -calc hw -puzzels -tkes more thinking and reasoning

Confirmation Bias

-the tendency to notice and search for information that confirms one's beliefs and to ignore information that disconfirms one's bliefs -expectations shapes information seeking, behavior toward others, impressions of others -people attend to info that confirms their predictions "i knew that all along....." -ignore and forget info that disconfirms expectations -expectations don't just influence what we attend to or remember -also influences how we interact with others and behave

Biases

-use certain types of information -attend to particular kinds of info -too effortful to try to process all info -preference of one over another -limit yourself

Biased Attention

-we tend to over emphasize personality's influence on the behavior of others -also biased by our own expectations

Professor DeVeaux reads an essay in which a student argues in favor of full nuclear war to resolve the conflict in the Middle East. Initially she thinks, "Whoa, what an extremist!" Then she recalls that the student had been assigned to advocate nuclear war as part of the assignment. Why would Professor DeVeaux still be likely to view the student as a political extremist?

Although she adjusted her beliefs, she did not adjust them enough to fully account for the power of the situation.

Although field experiments may be high in ________, they often lack ________. A) mundane realism; psychological realism B) external validity; internal validity C) psychological realism; mundane realism D) internal validity; psychological realism E) internal validity; external validity

B) external validity; internal validity

Marius doesn't often experience the overconfidence barrier because he A) is a physician. B) has taken two courses in research design and statistics. C) is a law school student. D) has been a victim of discrimination. E) is an older person.

B) has taken two courses in research design and statistics.

Warren believes that Tom is an outgoing, gregarious person. "Whom did you hang out with this weekend?" Warren asks Tom. "Tell me about all of the fun things that you have planned for the summer," Warren continues. Although Tom is usually rather quiet and reserved, he responds to Warren in an outgoing, friendly manner. This is an example of ___.

A self-fulfilling prophecy

Most people, when asked, guess that more people die from grisly accidents than from strokes. This is probably due to the greater number of accidental deaths reported in the media, and thus, due to A) the availability heuristic. B) the base rate fallacy. C) upward counterfactual thinking. D) the representativeness heuristic. E) downward counterfactual thinking.

A) the availability heuristic.

Gitte is writing her term paper with the radio playing in the background. Suddenly she realizes that her name is being spoken on the radio because she won a contest. This is referred to as A) the cocktail party phenomenon. B) non-automatic thinking. C) representative listening. D) intuition. E) conscious processing.

A) the cocktail party phenomenon.

Professor Atkins wonders whether more people attend confession during the Christmas season than at other times of the year. Three times per week during the months of February, April, September, and December, he sits quietly at the back of a church and records how many people come in for confession. Professor Atkins is using A) the observational method. B) obtrusive observation. C) the experimental method. D) archival analysis. E) participant observation.

A) the observational method.

Because a rhino horn looks something like an erect penis, some people believe that crushed rhino horn is an aphrodisiac. A couple decides to try it to spice up their sex life. Their decision is most like A) the representativeness heuristic. B) the noncommon effects heuristic. C) the correspondence bias. D) the availability heuristic. E) counterfactual thinking.

A) the representativeness heuristic.

During the last Canadian election, there was a televised debate among the candidates for Prime Minister. After the debate supporters of each side claimed that their candidate had handily won the debate. The supporters' judgments were probably affected by A) their schemas. B) the hostile media effect. C) a self-fulfilling prophecy. D) the fundamental attribution error. E) the representativeness heuristic.

A) their schemas.

Recall that Norbert Schwarz and his colleagues (1991) asked some participants to think of 6 times they had behaved assertively and asked other participants to think of 12 times they had behaved assertively, and then asked all participants to rate how assertive they thought they really were. The findings were that A) those assigned to generate 6 examples rated themselves most assertive due to the availability heuristic. B) those assigned to generate 12 examples rated themselves most assertive due to the availability heuristic. C) those assigned to give 12 items rated themselves more assertive due to the influence of the anchoring and adjustment heuristic. D) assertive people were more likely to generate 12 examples. E) the groups rated themselves equally in assertiveness due to the influence of the representativeness heuristic.

A) those assigned to generate 6 examples rated themselves most assertive due to the availability heuristic.

Kurt Lewin once wrote, "There is nothing so practical as a good theory." By that, Lewin was referring to the idea that A) to solve social problems, psychologists must understand the processes that underlie them. B) basic researchers should have a higher status in the field of social psychology. C) basic researchers should not be afraid to conduct applied research aimed at solving social problems. D) applied researchers should have a higher status in the field of social psychology. E) the line between basic and applied researcher is clearer in social psychology than in other sciences.

A) to solve social problems, psychologists must understand the processes that underlie them.

Tom and Julius are both in line for a movie, but it sells out before either of them can get tickets. Tom is the last person in line, but Julius is directly behind the person who got the last ticket. Based on the idea of counterfactual reasoning, why will Julius be more upset than Tom? A. It was easier for Julius to imagine how he could have gotten a ticket. B. Julius got there earlier than Tom did, so he felt he deserved to see the movie. C. Julius won't be more upset than Tom. D. Julius wanted to see the movie more than Tom did.

A. It was easier for Julius to imagine how he could have gotten a ticket.

According to the authors of your text, why is it that there may be conflicting philosophical positions about human nature? A. The world is a complex place, and situational differences impact behavior. B. Philosophy relies purely on layperson observation. C. Philosophers make their living through debate. D. It is likely that philosophers build incorrect arguments about human nature.

A. The world is a complex place, and situational differences impact behavior.

If accurate judgments are our goal, we should use base rate information instead of the representativeness heuristic A. we lack specific information about the person. B. the decision is a complicated one. C. base rates are low in the sample. D. base rates are high in the sample.

A. we lack specific information about the person.

Seeing pictures of physically attractive celebrities and models, many people in the U.S. assume that these people are also intelligent, kind, or wealthy. Such assumptions illustrate the schema held widely as a culture that A. what is beautiful is good. B. beauty is in the eye of the beholder. C. beauty is only skin deep. D. one should beware of that which is too beautiful.

A. what is beautiful is good.

If you were a researcher studying self-fulfilling prophecies, when would you warn students that they are most likely to occur? A. when we are distracted more so than when we are not distracted B. when we are children more so than for adults C. later rather than earlier in relationships D. when we have positive expectations rather than negative ones

A. when we are distracted more so than when we are not distracted

Leslie cannot say "no" to anyone. Consequently she takes on too much, and doesn't fulfill her commitments as promptly as she might. Assume that when asked to explain these behaviors, Leslie is motivated to be accurate in her self-perceptions. What would she be most likely to say? A: "I guess I'm a bit of a pushover, and take on more than I can responsibly handle." B: "You know me. I'm a go-getter who can't stand being bored." C: "I would have gotten everything done if my sister hadn't come to visit." D: "I guess you can't teach an old dog new tricks."

A: "I guess I'm a bit of a pushover, and take on more than I can responsibly handle."

In which of the following examples does the need for accurate information most conflict with the need for self-esteem? A: Carla has to decide whether to read the detailed red ink comments on the "D" paper she just spent weeks writing. B: Troy has to decide whether to read his opening night reviews after the play's producer has promised that the play will have at least a six-week run. C: Isabel has to decide whether to pay attention to her coach's evaluation of her performance after they have won the semi-final. D: Joy has to decide whether to examine her job review file after receiving a promotion.

A: Carla has to decide whether to read the detailed red ink comments on the "D" paper she just spent weeks writing.

Dr. McDaniels conducts a study in which he places either an expensive or a cheap purse on a park bench. He then counts how many people try to return the purse to its owner. What kind of study is this? A: Field experiment B: Correlational C: Experiment D: Observational

A: Field experiment

Social and personality psychologists have a shared focus on: A: Individuals rather than collectives or institutions. B: individual differences. C: the power of construals to shape human behavior. D: the cultural context.

A: Individuals rather than collectives or institutions.

How do social psychologists differ from those who rely on common sense or folk wisdom in answering questions about human nature? A: Social psychologists use science to test hypotheses about the social world. B: Social psychologists ignore the notion of human consciousness. C: Social psychologists rely primarily on insight. D: Social psychologists seldom disagree with one another.

A: Social psychologists use science to test hypotheses about the social world.

Which statement below best captures the major ethical dilemma faced by experimental social psychologists? A: That which is good science may not always be ethical science. B: Research results can be misused by those in power. C: With the advent of new technologies, it is easier than ever to fake data. D: It is necessary to lie to discover the truth.

A: That which is good science may not always be ethical science.

Warren believes that Tom is an outgoing, gregarious person. "Whom did you hang out with this weekend?" Warren asks Tom. "Tell me about all of the fun things that you have planned for the summer," Warren continues. Although Tom is usually rather quiet and reserved, he responds to Warren in an outgoing, friendly manner. This is an example of A: a self-fulfilling prophecy. B: reconstructive memory. C: the perseverance effect. D: the primacy effect.

A: a self-fulfilling prophecy.

Several studies conducted by Nisbett and his colleagues involved showing participants two similar pictures and asking them to find the differences between them. Nisbett and colleagues found that Western participants had a more __________ thinking style, and East participants had a more ___________ thinking style. A: analytic; holistic B: analytic; logical C: Christian; Buddhist D: holistic; contextual

A: analytic; holistic

Satisfying intellectual curiosity describes ________ research, while solving social problems describes ________ research. A: basic; applied B: social neuroscience; cross-cultural C: cross-cultural; social neuroscience D: applied; basic

A: basic; applied

Stephen is in love with Adrian and views her temper as an endearing example of her "feistiness." Her coworkers, however, interpret Adrian's temper as rude and insensitive. The difference between Stephen and Adrian's coworkers illustrates the power of love to influence our: A: construals. B: influence attempts. C: behaviors. D: relationships.

A: construals.

You stayed up all night cramming for this examination and didn't do as well as you had hoped. "If only I had started studying sooner and gotten a good night's rest, I'd have done much better," you think to yourself. You have just engaged in: A: counterfactual thinking. B: self-justification. C: unrealistic fantasy. D: wish fulfillment.

A: counterfactual thinking.

"Are some people just better leaders than others?" Such a question about human nature is most likely to be asked by a: A: personality psychologist. B: sociologist C: social psychologist. D: philosopher.

A: personality psychologist.

The study of brain processes is ________ in social psychology ________. A) not relevant; as the focus is on social thoughts, feelings and behaviour B) well developed; and especially used to study cultural differences C) a recent area; called social neuroscience D) used; to carry out applied rather than basic research E) only relevant; for research on anti-social behaviour

C) a recent area; called social neuroscience

Mario conducts a study on what Caucasians perceive as physically attractive, then conducts the same study with African Americans, and then with Asian Americans as the sample. Mario is using ________ to determine the external validity of his findings. a. meta-analysis b. a cover story c. surveys d. replications

D

When most Americans walk into a fast food restaurant chain, they know they should walk up to the counter to order, pay, and pick up their food, rather than sit down and wait to be served. This knowledge of how to act in such a situation is called a a. controlled thinking. b. stereotype. c. counterfactual thought. d. schema.

D

Which of the following is not a major drawback of observational methods? a. They cannot be used to study rarely occurring behaviors very well. b. They do not allow scientists to predict behavior. c. Archival research is at the mercy of the quality of the original reports. d. These methods cannot accurately describe behavior.

D

Participants in the Latané and Darley (1968) experimental study of the effects of group size on intervention in an emergency utilized A) a staged assault on the experimenter. B) participants' reports of what they would do in an emergency. C) an audiotaped seizure. D) police footage of the Kitty Genovese murder. E) audiotapes of the Kitty Genovese murder.

C) an audiotaped seizure.

A researcher is interested in how enemies are depicted in times of international conflict. He watches old films and newsreels from the Second World War in which enemy soldiers are depicted as crazed, vicious killers who enjoy torturing people. The researcher is using A) participant observation. B) the correlational method. C) archival analysis. D) an experimental method. E) a quasi-experimental method.

C) archival analysis.

Jeremy is in love with Carol and views her temper as an endearing example of her "feistiness." Hercoworkers, however, interpret Carol's temper as rude and insensitive. The difference between Jeremy and thecoworkers illustrates the power of love to influence our A. relationships B. influence attempts C. construals D. behaviors

C. Construals

According to recent research, why does culture influence our tendency to make internal versus external attributions? A. Collectivist cultures seldom have any unusual or anti-social actions to explain. B. Collectivist cultures do not have schemas for "cause." C. Cultures vary in the extent to which interdependence and conformity are valued. D. Western cultures value "team players" more than do Eastern cultures.

C. Cultures vary in the extent to which interdependence and conformity are valued.

If accurate judgments are our goal, we should use base rate information instead of the representativeness heuristic when A) base rates are high. B) we have a wealth of information about the person in question. C) base rates are low. D) we don't have a lot of specific information about the person in question. E) the decision is a complicated one.

D) we don't have a lot of specific information about the person in question.

Maisie was very upset after she participated in an experiment. During the ___, the researchers made sure that she understood that the situation was not real, and made sure that Maisie was calm and happy before leaving.

Debriefing

According to research conducted by Susskind and his colleagues (2008) presented by the authors of your text, when people make the facial expression of disgust, it involves ___.

Decreasing input from the senses.

Social psychologists are interested in cross-cultural research because it ___.

Demonstrates which aspects of human behavior are universal.

Dimitri went through a lot of hard work during basic training in the Army. At times, drill sergeants yelled at him; he was physically exhausted and emotionally distressed. But, at the end of Basic Training, he felt that joining the Army had been a very good decision. The self-esteem explanation for Dimitri's feelings about Basic Training is

Dimitri has to somehow justify his suffering while maintaining his self-esteem.

From across the room, J.T. sees his mother sigh, and he approaches to give her a hug in the hopes of cheering her up. In this case, J.T.'s behavior is an example of a(n) ___ social influence attempt.

Direct

When asked to guess why Lafayette gave the drugs to Tara, you said it was because he was a caring and generous person. You have just made a ______ attribution

Dispositional

When Marsha is giving her friends advice on dating, her decisions and advice are often simplistic and inaccurate. However, when Marsha is deciding for herself whom to date and how to handle her own relationships, she is a more complex thinker and her judgments are more accurate. This example illustrates the influence of _______ on social thinking. A) unconscious processing B) flexibility C) personal experience D) perspective-taking E) motivation

E) motivation

________ is to independent variable as ________ is to dependent variable. A) Effect; manipulation B) Effect; cause C) Manipulation; cause D) Cause; manipulation E) Cause; effect

E) Cause; effect

After reading Chapter 2, what would you say to a Prime Minister who proudly takes credit for lowering the deficit, reducing crime, and many other positive events? A) Did you sample the entire population? B) Archival data are often suspect. C) By how much did you lower the deficit and reduce crime? D) Cause-and-effect is impossible to determine in the real world. E) Correlation does not mean causation.

E) Correlation does not mean causation.

The _______ led Bibb Latané and John Darley to systematically test the situational factors that influence people's responses to emergencies. A) Bay of Pigs fiasco B) Iran-Contra affair C) Vietnam War D) Watergate scandal E) murder of Kitty Genovese

E) murder of Kitty Genovese

In social psychology, the technique of meta-analysis is most like ________ research. A) archival B) survey C) experimental D) correlational E) observational

E) observational

According to your text, when conducting experiments in social psychology, there is almost always a trade-off between A) mundane and psychological realism. B) random selection and random assignment. C) basic and applied research. D) funding and replication. E) internal and external validity.

E) internal and external validity.

In trying to decide which of two classmates is smarter, you use the rule, "the faster people talk, the smarter they are," and you choose the classmate who talks faster. You have applied a _______ to decide who is smarter. A) self-fulfilling prophecy B) fundamental attribution C) decoding bias D) correspondence bias E) judgmental heuristic

E) judgmental heuristic

According to Daniel Wegner's theory of thought suppression, two processes are involved in suppressing unwanted thoughts. The _______ process searches for evidence of the unwanted thought; the _______ process attempts to provide a distraction whenever the thought comes to mind. A) controlled; automatic B) tracking; diverting C) diverting; tracking D) seeking; distracting E) monitoring; operating

E) monitoring; operating

Your text describes a study in which social psychologists infiltrated a cult who believed that a spaceship would rescue them just before the world would be destroyed. These social psychologists were conducting A) correlational research. B) archival analysis. C) a field experiment. D) a survey. E) observational research.

E) observational research.

Julia eats granola bars and avoids chocolate bars, even though the granola bars contain 50 percent more fat and 15 percent more sodium than the chocolate bars. According to a social cognition approach, why might that be? Julia ___.

Failed to seek out all the relevant facts about the nutritional value of those snacks.

In a study of group dynamics, participants were placed in groups consisting of either three or ten people. During the study, group members worked together trying to solve a puzzle. After completing the task, participants reported how satisfied they were with the other members of their group. ___ is the independent variable in this study.

Group size

In talking to an acquaintance of yours, you note that she never looks you directly in the eye. To figure out whether this is because she is shy, because she dislikes you, or because she comes from a culture that discourages direct eye contact, you would probably ___.

Make an attribution

Recall that Kelley (1950) told some students that a guest instructor was a warm person and others that a guest instructor was a cold person. When the instructor behaved in an arrogant way, he was rated as immodest, no matter what students were told about him at the outset. These findings suggest that ___.

Schemas are specific to the situation in which we find ourselves.

After reading Chapter 1, if you were to advise producers of a safe-sex television campaign, what would you tell them?

Remember that sometimes people would rather feel good than be accurate.

The "Barnum effect," in which vague statements about one's future or personality may be seen as accurate and valid, occurs because of the ___.

Representativeness heuristic.

Which of the following questions is most likely to be asked by a social psychologist?

What situations cause people to behave rudely?

Culture can influence our schemas by influencing ___.

What we notice and remember.

Which of the following is not a likely consequence of the human tendency to prefer self-justifying information to accurate information?

a. People maintain their level of self-esteem. b. People become less likely to learn from their experiences. c. People may make faulty decisions about future behavior. *D. People gain feedback that makes for better decisions.

What do philosophers and social psychologists share in common? Both

address many of the same questions.

We are most likely to rely on schemas when the situation we confront is

ambiguous.

Social psychology is set apart from other ways of interpreting social behavior such as folk wisdom or literature because it is ________.

an experimental science

Social psychology is set apart from other ways of interpreting social behavior, such as folk wisdom or literature, because it is

an experimental science.

When people tend to focus on the properties of objects without consideration of the context, this is described as a(n) ___________ thinking style.

analytic

Several studies conducted by Nisbett and his colleagues involved showing participants two similar pictures and asking them to find the differences between them. Nisbett and colleagues found that Western participants had a more __________ thinking style, and East participants had a more ___________ thinking style.

analytic; holistic

according to the authors of your text, when Darwin developed evolutionary theory, it was aimed at explaining how

animals adapt to their social environment

One of Kurt Lewin's boldest intellectual contributions to social psychology was

applying Gestalt principles to social perception.

Professor Yarnofsky wonders whether recent acts of airline terrorism have made the public more fearful of airline flight. He secures records of the amount of flight insurance that people have purchased via machines at the airport, and compares the records of insurance purchases before the last hijacking to purchases after the last hijacking. Professor Yarnofsky has employed a(n) ________ research method.

archival

Social psychologists differ from anthropologists and sociologists in that social psychologists

are concerned with people's construals of their social environments.

On the campus of the University of Minnesota, there are signs near large expanses of grass that read, "Lie on me, play on me, but please don't walk on me." These signs are more likely to protect the grass than signs that read "Stay off the grass!" because the Minnesota signs

are likely to generate less psychological reactance

Professor Srinivasan is a psychologist, yet does not study cognition, thoughts, or feelings because she claims that they are not rooted in observable behavior. Professor Srinivasan is most likely a ________.

behaviorist

When Suzie whines in a supermarket, her father gives her candy to keep her quiet. After a while, Suzie whines more often in the market because her father has "rewarded" her whining with candy. This explanation is more compatible with a ________ approach.

behaviorist

Julie and Tina have a very exclusive club. In order to make new members loyal and really enjoy being part of the club, Julie says new members should be rewarded for their good behaviors in the club. Tina, however, suggests that they use an embarrassing initiation. In this example, Julie is taking a ________ approach, while Tina is taking a ________ approach.

behaviorist; self-esteem behaviorist; self-esteem

The tendency to stick with an initial judgment even in the face of new information that should prompt us to reconsider is _______.

belief perseverance

proverbial and anecdotal knowledge

can support almost any claim

If the research on priming and metaphors is correct, what should your organization do if it wants to raise money for its service project?

candy

a researcher interested in testing a ____________ hypothesis would be most likely to conduct an experiment

causal

Internal validity is to ________ as external validity is to ________.

causality; generalizability

The ________ route to persuasion is to enduring attitude change as the ________ route to persuasion is to transient change.

central; peripheral

Kenneth believes that Funny-O's cereal is good because it has no sugar, it contains all of the recommended vitamins and minerals, and it has no artificial flavors. Kenneth's attitude toward Funny-O's is a(n)

cognitively based attitude.

Karen returns home from her first year of college, and is very proud of her first-year GPA. She earned a 3.0 ("B") average. She's unhappily surprised to discover that her parents are disappointed that she didn't perform better. This difference in interpretation illustrates the power of ________ in explaining social behavior.

construals

Ted likes Jim and smiles at him every chance he gets. Jim wonders why Ted is always smirking at him sarcastically and studiously avoids him. Ted thinks he's being friendly and cannot understand Jim's standoffishness; Jim thinks Ted is a jerk for continuing to smirk at him without explanation. This difference

construals

Ted likes Jim and smiles at him every chance he gets. Jim wonders why Ted is always smirking at him sarcastically and studiously avoids him. Ted thinks he's being friendly and cannot understand Jim's standoffishness; Jim thinks Ted is a jerk for continuing to smirk at him without explanation. This difference illustrates the role of ________ in explaining social behavior.

construals

Jeremy is in love with Carol and views her temper as an endearing example of her "feistiness." Her coworkers, however, interpret Carol's temper as rude and insensitive. The difference between Jeremy and the coworkers illustrates the power of love to influence our

construals.

Last weekend, Paula smiled politely while Lance talked to her for several hours. Wow, thinks Lance to himself, she was so nice; Paula must really like me-she was so friendly! Lance's interpretation of Paula's behavior is an example of

construals.

The "father" of modern experimental social psychology, Kurt Lewin, pointed out the importance of understanding how people ________ their social environments.

construe

Don is told that he is being laid off from his job of five years. His employer explains that there is nothing Don or anyone in management at his office can do to change this decision. Based on information from your text, Don is experiencing a loss of _____________ in this situation.

control

You stayed up all night cramming for this examination and didn't do as well as you had hoped. "If only I had started studying sooner and gotten a good night's rest, I'd have done much better," you think to yourself. You have just engaged in

counterfactual thinking.

The attributional perspective that addresses behaviors that occur over time, place, and different actors and targets is the ________ model.

covariation

If you were a researcher studying self-fulfilling prophecies, when would you warn students that they are most likely to occur? a. when we have positive expectations rather than negative ones b. when we are children more so than for adults c. later rather than earlier in relationships d. when we are distracted more so than when we are not distracted

d.

In trying to decide which of two classmates is smarter, you use the rule, "the faster people talk, the smarter they are," so you choose the classmate who talks faster. You have applied a ________ to decide who is smarter. a. systematic judgment b. self-fulfilling prophecy c. cultural truism d. judgmental heuristic

d.

Karen is so used to driving to her boyfriend's house that when she is taking her little brother to softball practice early one morning, before she is fully awake, she finds herself driving to her boyfriend's house instead. This is an example of a. judgmental heuristics. b. counterfactual thinking. c. controlled thinking. d. automatic thinking.

d.

Nao was raised in Japan, and is walking through Times Square in New York City. Based on results from Nisbett' s studies on cultural differences in thinking styles, which of the following would Nao be most likely to notice relative to people raised in the U.S.? a. the number of people b. the advertisements c. the number of homeless people d. how the buildings are arranged relative to each other

d.

Express is to ________ as interpret is to ________.

encode; decode

You have worked extremely hard to attain a goal, but soon realize that the goal is not as exciting as you expected. You will probably

exaggerate the positive qualities of the goal in order to justify your effort.

In the self-fulfilling prophecy, a perceiver has a(n) ________ that affects the ________ of the target of perception.

expectation; behavior

According to research presented in this chapter, we typically use two steps in making attributions. The first step involves ________, and the second step involves ________.

forming an internal attribution; adjusting for the situation

At the grocery store, mothers often keep children busy by letting them "drive" the cart. Little do the children realize that the steering wheel they are playing with does not control anything. This example demonstrates our sometimes misplaced sense of control over

free will and our own behavior

Consider the following pieces of folk wisdom: "Out of sight, out of mind" versus "Absence makes the heart grow fonder." These statements represent which of the following shortcomings of relying on common sense? The common sense of folk wisdom is often

full of contradictory assumptions.

The tendency most people have to discount situational explanations of behavior in favor of personality characteristics or traits is called the

fundamental attribution error.

Research by Kahneman and Tversky (1973) presented by the authors of your text found that people do not use base rate information sufficiently, and pay more attention to

how well the information they have about a person represents a certain category.

Biologically based drives that affect human behavior include

hunger and thirst.

Schemas are most like which of the following?

hypothesis

a self-fulfilling prophecy is like a(n) _________ you want to come true.

hypothesis

Explicit attitudes are ________, while implicit attitudes are ________.

in conscious awareness; involuntary and sometimes unconscious

Self-fulfilling prophecies most often are a result of

inadvert/unconscious

"My happiness depends on the happiness of those around me" and "I enjoy being unique and different from others in many respects" are two items from Singelis's (1994) scale, which was designed to measure individual differences in

independence and interdependence.

Experimental Research- types of variables

independent variable dependent variable random assignment

Face-ism refers to the media's tendency to

indirectly perpetuate stereotypes by focussing on men's faces and women's bodies.

Compared to social psychologists, personality psychologists are more likely to focus their attention on

individual differences.

Complete the analogy about culture differences: Western culture: ________ :: ________: social group.

individualist; Eastern culture

One potential problem of justifying past behavior in order to protect our self-esteem is that

it hinders people from learning from their past mistakes.

In trying to decide which of two classmates is smarter, you use the rule, "the faster people talk, the smarter they are," so you choose the classmate who talks faster. You have applied a ________ to decide who is smarter.

judgmental heuristic

Rashim is viewed by most people he knows as rude, brusque, and completely unconcerned with other people's feelings. He, in contrast, describes himself as efficient and task-oriented. Rahim's self-description best reflects the motive to

justify our troublesome behaviors.

Based on your reading of Chapter 3 (Social Cognition), who would you predict would be more frustrated: Katie, who missed an "A" by one point, or James, who missed an "A" by five points? Correct!

katie, easier for her to imagine

According to social cognition researchers, we sometimes have difficulty making sense of the social world because we ___.

lack the stamina to seek out all the relevant facts.

Thomas is scrupulously honest when it comes to not cheating on his tests and papers, but when a cashier accidentally gives him back too much change, he is likely to keep the extra money. A social psychologist would most likely say that Thomas's behavior ________.

must be responding to factors in the situation that affect his honesty

There is a moderately strong correlation between the number of bars in a town and the number of churches in a town. As it turns out, this correlation is due to the correlation of both numbers of bars and number of churches with town population. This illustrates which of the following? Correlations are

no guarantee of causation.

During the 1990s, the MTV program The Real World aired, and currently there are several "reality TV shows" in which cameras record the activities of people living together. These shows are most like ________ research in social psychology.

observational

A researcher discovers a strong positive correlation between ice cream sales and the incidence of violent crimes. Based on this, we can conclude that

on a day when ice cream sales are high, there are likely to be more violent crimes.

The available heuristic is used when making judgments about

other people

When Rhianna left Chris Brown after he abused her, she was demonstrating a ___________ of _____________.

reduction: cognitive dissonance

In a study by Correll et al. (2002) described in the text, people played a video game in which they saw white and black people holding either a gun or a cell phone, and had to press either a "shoot" button or a "don't shoot" button. The results of the study showed a "shooter bias." This meant that people were more likely to press the "shoot" button

regardless of what the target held, but only if the target was black.

Behaviorists believe that all learning is a result of

reinforcement and punishment.

According to the authors, a particularly dangerous effect of advertising is that it

reinforces and perpetuates stereotypes.

Social psychologists differ from philosophers in that philosophers

rely on logical arguments, and social psychologists rely on research.

In the introduction to Chapter 1, you read about a number of social phenomena: cries for help were ignored by neighbors; a father and son disagreed on the attractiveness of the same fraternity; and more than 800 people committed mass suicide in Jonestown, Guyana. What do these examples have in common? They

reveal the power of social influence.

In trying to make sense of the mass suicide in Jonestown, a behaviorist would probably examine the

rewards and punishments that Jim Jones used to influence his followers.

Which of the following is most likely to be studied by a social psychologist?

sex differences in self-concepts

In general, when people first encounter a novel situation, they tend to

size up the situation quickly and effortlessly and, for the most part, correctly.

Human beings are often motivated to construe themselves and the social world accurately. The ________ approach is based on this assumption.

social cognition

Della is interested in the ways in which romantic partners think about each other. Her research most neatly fits the realm of

social cognition.

Juan thinks that the idea "birds of a feather flock together" has more merit than "opposites attract." So he designs an experiment to test his hypothesis. Juan is most likely a

social psychologist.

Consider the following research question: "Have no-fault divorce laws increased the rate of divorce in the United States?" This question is most likely to be asked by

sociologist

we have limited cognitive resources to....

solve problems direct attention and behavior

In Kelley's 1950 research about the guest lecturer, the dependent variable was the

students' impressions

Many fraternities, sororities, and sports teams have initiation rituals. For example, at one school, all the members of the soccer team must wear unusual clothing everywhere on campus for an entire day and then have embarrassing haircuts. These rituals try to capitalize on the role of ________ in increasing commitment via self-justification.

suffering

The authors of your text explain that people tend to overestimate how bad they will feel if a negative event were to happen to them. This tendency is called

the impact bias.

One common goal of sociology and social psychology is to understand

the influence of social factors on human behavior.

Social psychologists tend to be more focused on ________, and personality psychologists tend to focus more on ________.

the influence of the situation; individual differences

Tom saw a fad weight-loss program advertised on late-night television. The ad claimed that deep breathing exercises would speed up metabolism, causing people to lose weight. He mailed a check for $99.95. When the plan arrived, Tom rearranged his day so that he could complete the breathing exercises as scheduled. They made him nauseated and dizzy, and before long, Tom's friends began to think that he was going overboard with the program, and they began to avoid him. After the end of the ten-week program, Tom got on the scale and saw that he hadn't lost an ounce! Still, he remained enthusiastic and thought, "I may nothave lost any weight, but I feel so much healthier!" This situation is best explained by

the justification of suffering.

social cognition

the processes by which people think about and make sense of their social environments

Social psych uses _____________________________- to answer questions about human nature

the scientific method

People literally create their own realities" is the basic idea behind ________.

the self-fulfilling prophecy

After the mass suicides related to the cults at Jonestown, Waco, and Heaven's Gate, people tended to blame the victims and accuse them of being psychologically unstable, or deranged. Social psychologists are more likely to explain these mass suicides as being due to

the social influence of cult leaders.

According to the authors of your text, archival research can tell researchers about

the values and beliefs of a culture.

the crucial role of cognitive resources

these results suggest that it is only when a desire for accuracy is combined with sufficient cognitive resources that people can move beyond their tendency to simplify

Recall that Harold Kelley (1950) told some college students that their guest lecturer was a warm person, and others that he was a cold person. The visitor then lectured for twenty minutes. After the lecture,

those students who expected the lecturer to be warm rated him higher and participated more in class discussion than those who expected him to be cold.

Which of the following is not a function of schemas?

to make logical inferences based on an objective set of facts

When Robin meets her friend's new roommate, she sees that she has easels and paints strewn about, and has decorated her side of the room with lots of hand-made ceramic pots and paintings. At first, Robin is most likely

to use her "artistic" person schema and assume that her friend's roommate is free-thinking and creative.

A goal of ethnography that makes it different from other methods of testing theories is that in ethnography, the researcher

tries to understand the group or culture without imposing his or her own ideas on it.

Social and personality psychologists share which common goal?

understanding causes of human behavior

independent variable

variables that are often controlled or manipulated ie color of tshirt participants wore

Complete the following analogy: independent variable is to ________ as dependent variable is to ________.

varied; measured

Which of the following questions would be of most interest to a social cognition researcher?

What factors prevent people from reasoning accurately?

Carlos, the president of a fraternity on campus, randomly sampled 500 students, asked them if they belonged to a fraternity or sorority, and asked them about their current GPA. He discovered that the GPAs of those people in fraternities and sororities are higher than those of people who are not involved in the Greek system. Gleefully, he presented his findings to the dean, saying that being involved in a fraternity or sorority leads to higher grades. What rule of research methods is Carlos breaking? a. Correlation does not imply causation. b. College students are not representative of the whole sample of people in the world. c. His sample size of 500 is too small to make such a generalization. d. Correlational data do not provide any practical information on a topic.

A

Ethan and Heather see a man stumbling around as he walks down the street. Ethan, who belongs to Alcoholics Anonymous, thinks the man is drunk, but Heather, who just watched a TV special on Michael J. Fox, thinks the man has Parkinson's disease. These differing interpretations of the same behavior seem to be caused by Ethan and Heather's differences in a. accessibility. b. current goals. c. stereotypes. d. self-fulfilling prophecies.

A

One problem with informed consent is that a. full knowledge of the experiment's purpose can alter participants' behavior. b. its use tends to undermine the external validity of the experiment. c. its use is limited to deception experiments. d. it is time-consuming.

A

Consider the following survey item: "Most people agree with the following statement. Do you agree or disagree that parents should be held financially responsible for their children's vandalism?" Most social psychologists would not include such an item on their surveys because A) such survey questions can bias people's responses. B) opinion questions on surveys are seldom straightforward. C) adults without children will give biased responses. D) this is not a topic social psychologists would be interested in. E) parents of children do not represent the entire population.

A) such survey questions can bias people's responses.

Whereas observational research is designed to ________ behaviour, correlational research is designed to ________. A) systematically describe; assess relations between variables B) explain; systematically describe C) explain; assess relations between variables D) systematically describe; determine what causes behaviour E) systematically describe; explain behaviour

A) systematically describe; assess relations between variables

Which of the following people have fallen prey to the fundamental attribution error? A. Tim, who points to a person who fell down and says, "What a clumsy oaf!" B. Cindy, who explains her poor exam performance by pointing out how hard the questions were. C. Guillermo, who explains his girlfriend's tears by saying, "She didn't get enough sleep last night." D. Dien, who points to an erratic driver and says, "Look at that! The roads are slick tonight."

A. Tim, who points to a person who fell down and says, "What a clumsy oaf!"

Ethan and Heather see a man stumbling around as he walks down the street. Ethan, who belongs to Alcoholics Anonymous, thinks the man is drunk, but Heather, who just watched a TV special on Michael J. Fox, thinks the man has Parkinson's disease. These differing interpretations of the same behavior seem to be caused by Ethan and Heather's differences in A. accessibility. B. self-fulfilling prophecies. C. current goals. D. stereotypes.

A. accessibility.

When we make self-serving attributions, we tend to attribute our successes to ________ factors and our failures to ________ factors. A. internal; external B. uncontrollable; controllable C. unstable; stable D. inconsistent; consistent

A. internal; external

Imagine traveling to a country where you don't speak the native language. You just had a great meal. You don't want to be misunderstood, so how can you communicate to your host that the meal was great, and that you're happy? A. smile B. give a "thumbs up" sign C. give the "ok" sign D. frown

A. smile

Sebastian sits behind Mark in economics class. One day, an argument starts between Mark and the professor. When asked who he thought was responsible for the argument, who will Sebastian say is more responsible for the argument? A. the professor B. Mark C. both Mark and the professor equally

A. the professor

The six major facial expressions - anger, fear, disgust, happiness, surprise, and sadness - are widely considered to be A. universal. B. culture specific. C. developmentally determined. D. primate-specific.

A. universal.

Recall that a number of researchers have found that participants from Asian cultures are less likely than participants from Western cultures to make the fundamental attribution error. These findings suggest that A. values and norms can influence cognitive processes. B. experimental stimuli were poorly translated. C. it is relatively easy to teach people to reason more logically. D. Eastern cultures are more advanced than Western cultures.

A. values and norms can influence cognitive processes.

Korsakov's syndrome is a neurological disorder more closely associated with abnormal psychology than with social psychology. Why would the authors of your text describe the symptoms of Korsakov's syndrome and provide Oliver Sacks's description of his encounter with a patient suffering from that disease? This discussion illustrates A) the dangers of relying on heuristics to understand medical problems. B) the importance of schemas in helping humans make sense of their experiences. C) the power of schemas to create reality. D) how unreliable our schemas can be. E) the hazards of holding rigid, inflexible schemas.

B) the importance of schemas in helping humans make sense of their experiences.

Replications are designed to assure that A) ethical procedures are always followed. B) the original results can be obtained in different populations and in different settings. C) the original study was internally valid. D) the original study was high in psychological realism. E) the original study was high in mundane realism.

B) the original results can be obtained in different populations and in different settings.

According to the authors of your text, people raised in which of the following countries would be most likely to have a holistic thinking style? A. United States B. China C. Great Britain D. Italy

B. China

Which of the following relatively "famous" psychologists is a behaviorist?

B. F. Skinner

Isabella goes to a fortune teller who tells her, "You are the type of person who is very outgoing and sociable, yet at times you enjoy being alone." Isabella thinks this captures her personality amazingly well, because she finds many instances in her memories of such behaviors. Isabella has just fallen prey to the A: base rate heuristic. B: Barnum effect. C: priming effect. D: self-fulfilling prophecy.

B: Barnum effect.

After a controversial sports play, referees meet in the middle of the field to discuss what each saw and explain his/her ruling before coming to a final decision. This most closely resembles which of the following: A: Archival analysis B: Interjudge reliability C: External validity D: Dependent variables

B: Interjudge reliability

One application of Ross's idea of naïve realism is to assist the negotiations between long-time adversaries: Palestinians and Israelis. How would understanding naïve realism potentially help these parties negotiate more successfully?

Both sides would understand that their perceptions are biased, and try to be more objective.

After a controversial play, referees meet in the middle of the field to discuss what each saw and explain his/her ruling before coming to a final decision. This most closely resembles which of the following? a. archival analysis b. dependent variables c. interjudge reliability d. external validity

C

Christopher is considering breaking up with his high-school sweetheart. He spends many hours weighing the benefits and drawbacks, he asks his friends for advice, and he thinks about it further before coming to his decision. In this case, Christopher is using a. automatic thinking. b. the representativeness heuristic. c. controlled thinking. d. self-fulfilling prophecies.

C

Mental structures that organize information in our social world are called A) cognitive filters. B) counterfactuals. C) schemas. D) affect blends. E) heuristics.

C) schemas.

Priming and accessibility have greater impact when we interpret a(n) _______ event. A) explicit B) unusual C) certain D) ambiguous E) improbable

D) ambiguous

According to research described in Chapter 2 of the text regarding body-ideals and body sizes of young women and men, A) body sizes of young men have decreased in order to match the depiction of male models in the media. B) the body sizes of young women have decreased to a dangerous point because the ideal body for women, as portrayed in the media, is very slim. C) body sizes of young women have decreased slightly over the past decade because the ideal for women, as portrayed by the media, is very slim. D) body sizes of young men have increased due to more musculature and decreased fat. E) the average woman's body is further from the cultural ideal than it was 40 years ago.

E) the average woman's body is further from the cultural ideal than it was 40 years ago.

Adele is from France, where rabbits are eaten regularly. She can tell you the most tender part of the rabbit, how long to cook one, and how big a rabbit you would need to feed five people. Marsha is from the United States, where rabbits are pets, and bring coloured eggs to children in the spring. She hasn't a clue about how to cook one, even if she wanted to. This example best illustrates A) the representativeness. B) that rabbits are ambiguous stimuli. C) individual differences in the contents of schemas. D) the universality of schemas. E) the cultural determinants of schemas.

E) the cultural determinants of schemas.

A researcher investigated the relationship between self-esteem and choice of a partner in a "getting acquainted" situation. To do this, she had students take a personality test and then she gave them feedback (based on random assignment) indicating either that the test found them to be "mature, insightful, and socially skilled" or "immature, inflexible, and overly critical." After receiving this feedback, students chose a partner to interact with by looking at photos of people of the opposite sex who were either high or low in physical attractiveness. The study just described is a(n) ___.

Experiment

A researcher hired by a department store wants to examine whether manipulating the room temperature to be hotter as opposed to cooler affects sales of air conditioners. The best method for answering this question would be ___.

Experimental

Suppose the research by Donnerstein and Berkowitz (1981) regarding the role of violent pornography on aggression could be generalized to women aggressors. What could you conclude?

Exposure to violent pornography increases aggression toward men.

Rafael has always hated math courses. However, he likes most other courses at the university. Most other students tend to avoid math courses whenever possible, too. Given this information about consistency, distinctiveness, and consensus, most people would make a(n) ___ attribution for Rafael's attitude.

External

Researchers have found that couples who are happy in their marriages make very different attributions for their partners' behaviors than couples who are unhappy. Specifically, couples in happy marriages make ___, while couples in unhappy marriages make ___.

External attributions for negative behavior and internal attributions for positive behavior; internal attributions for negative behavior and external for positive.

Using a cover story to enhance the psychological realism of an experiment is one way to increase the ___ of a study.

External validity

Based on Nisbett and Wilson's (1977) work (as presented by the authors of your text) on telling more than you can know, if a survey respondent is asked about a hypothetical situation, he or she is probably going to respond ___.

In line with their theories and beliefs about what should have influenced them.

A social psychologist is perplexed. Is it true that birds of a feather flock together, or do opposites attract? Confronted with these two contradictory pieces of folk wisdom, what is this social psychologist most likely to do?

Investigate the different situations in which each bit of common sense holds true.

If all humans have access to the same cognitive tools, what is the role of culture?

It determines which tools you use most.

Although fields such as anthropology, sociology, and social psychology are related, what distinguishes social psychology from the others?

It examines how construal of social situations impact people's lives..

Even if an observer has information about an actor's situation available, why might the observer still be likely to commit the fundamental attribution error?

It is difficult to know how an actor interprets a situation

Even if an observer has information about an actor's situation available, why might the observer still be likely to commit the fundamental attribution error?

It is difficult to know how an actor interprets a situation.

Suppose a certain student, Nathan, falls asleep during every chemistry class. Suppose further that Nathan is the only one who falls asleep in this class, and that he falls asleep in all of his other classes. According to Kelley's covariation model of attribution, how will people explain Nathan's behavior (falling asleep)?

It is due to something unusual about Nathan, because his behavior is low in consensus, low in distinctiveness, and high in consistency.

Tom and Julius are both in line for a movie, but it sells out before either of them can get tickets. Tom is the last person in line, but Julius is directly behind the person who got the last ticket. Based on the idea of counterfactual reasoning, why will Julius be more upset than Tom?

It was easier for Julius to imagine how he could have gotten a ticket.

Your roommate questions how charitable you really are. She says to you, "Give me just one example of the last time you gave money to a homeless person." Odds are that her challenge will convince you that you really are a generous person. Based on research by Schwarz and his colleagues (1991), why is that?

It's easy to bring to mind one example that supports your self-schema.

Jill is eating dinner at her fiancé Rob's parents' house, and barely touches her food. Jill is aware that his parents could make different construals of this behavior, such as

Jill must not like the food served at dinner.

Which of the following is the best example of a construal?

John, who believes people born under the sign of Taurus are stubborn

Which of the following psychologists is considered the founding "father" of modern experimental social psychology?

Kurt Lewin

Recall that Harold Kelley (1950) told some college students that their guest lecturer was a warm person, and others that he was a cold person. The visitor then lectured for twenty minutes, and behaved in a very cocky, arrogant way during the lecture. All students viewed the lecturer as arrogant, but differed in their assessments of how humorous he was. These results suggest that schemas are ___.

More likely to be used to interpret ambiguous behaviors.

Which of the following is not a problem with using common sense or folk wisdom explanations for social behavior?

Most people do not have common sense anymore.

Recall that even when participants in an experiment conducted by Jones and Harris (1967) were told that people were assigned to write an essay sympathetic to Cuban dictator Fidel Castro, they still were willing to assume that the essay reflected the writer's true "pro-Castro" attitudes. In this experiment, how could people have avoided the fundamental attribution error? They could have assumed ___.

Nothing, and rated the essay writer's attitude in the midpoint of the pro- and anti-Castro scale.

Recall that Latané and Darley observed the number of participants in each experimental condition who left their cubicles to help the alleged victim of a seizure. The ___ was the independent variable in their experiment.

Number of other participants present

A researcher discovers a strong positive correlation between ice cream sales and the incidence of violent crimes. Based on this, we can conclude that ___.

On a day when ice cream sales are high, there are likely to be more violent crimes.

Why do people often believe that they could have predicted social psychological findings in advance?

Only when looking back to findings seem like common sense; people often cannot predict them ahead of time

How would a social psychologist explain the results of Rosenthal and Jacobson's (1968) study, in which erroneous teacher expectations changed the behaviors of their students?

Our social expectations influence our own and others' behaviors.

How are implicit personality theories developed?

Over time and with experience

In an effort to convince more women to get mammograms, public service ads downplay fear of breast cancer and the discomfort of this cancer detection procedure. Instead such ads point to the benefits of early detection and consequent cure rate. According to the authors of your text, an ad could be designed this way because ___.

Overly frightened women might engage in denial about the odds that they would contract breast cancer.

Recall that Harold Kelley (1950) told some college students that their guest lecturer was a warm person, and others that he was a cold person. The visitor then lectured for twenty minutes, and behaved in a very arrogant way during the lecture. When students evaluated the lecturer, ___.

Students expecting a warm person rated the lecturer as funnier, but no less arrogant than students who expected a cold person.

Many fraternities, sororities, and sports teams have initiation rituals. For example, at one school, all the members of the soccer team must wear unusual clothing everywhere on campus for an entire day and then have embarrassing haircuts. These rituals try to capitalize on the role of ___ in increasing commitment via self-justification.

Suffering

If height and weight have a positive correlation, what does this tell you about the relationship between these variables?

Taller people are usually heavier, but there are exceptions to the rule.

According to Jones's "correspondent inference theory," we can make strong dispositional attributions about a politician who is vulgar, inarticulate, and crude because...

That kind of behavior is counter-normative for politicians

Wegner (2002, 2004) compared the illusion of free will to which of the following?

The "correlation does not equal causation" problem

Adele is from France, where rabbits are eaten regularly. She can tell you the most tender part of the rabbit, how long to cook one, and how big a rabbit you would need to feed five people. Marsha is from the United States, where rabbits are pets, and bring colored eggs to children in the spring. She hasn't a clue about how to cook one, even if she wanted to. This example best illustrates ___.

The On Automatic Pilot: Low-Effort Thinking.

Lee Ross describes the idea of "naïve realism" as

The conviction that all of us have that we perceive things "as they really are."

Which of the following is a consequence of underestimating the power of social influence?

We tend to oversimplify complex situations.

Julia reads a research study which shows that when children have a mother who talks about emotions with them, the children tend to be more empathetic. Julia scoffs, "This is obvious; I could have told you that!" Julia's reaction to the study is an example of ___.

The hindsight bias

Design an experiment to test a hypothesis that you make up (ex. social media use increases depression, males will behave more aggressively when frustrated than females, etc.). Name and describe the independent and dependent variables within your control and experimental groups. What ethical issues will need to be addressed in your study to protect the participants? Be creative! Then reply to one other student's posting with a question or comment about their study.

The hypothesis of my experiment would be: People who meditate are less stressed. The independent variable is meditating/not meditating. The dependent variable is stress level. The control group would consist of people who do not meditate. The experimental group would consist of people who do meditate. I would begin by recording both groups stress levels (for example with a survey or interview), then have the experimental group participate in meditation (while the control does not), and then record stress levels afterward. Some factors I need to keep in mind are: time meditating, place of meditation, whether meditation is with people, and the overall environment of the participants (say number of classes enrolled in, or job/home conditions). Some ethical issues that may need to be addressed in my study to protect the participants would include informing them of that the study is taking place, getting their consent, and privacy issues.

What is the take-home message from the research regarding the effects of generating different numbers of examples of behavior?

The more examples you have to think of, the less the behavior describes you.

In many respects, reality television shows are similar to observational research in social psychology. In what crucial respect are reality TV shows most different from observational research?

The observations were not conducted in a systematic, scientific manner.

The news often points out girls' relatively poor academic performance on standardized tests compared to boys' performance on those tests. Based on the research presented in this chapter, these gender differences may be due in part to ___.

The power of teachers' self-fulfilling prophecies.

Sebastian sits behind Mark in economics class. One day, an argument starts between Mark and the professor. When asked who he thought was responsible for the argument, who will Sebastian say is more responsible for the argument?

The professor

Although there are only a few runners on your college track team, you see a woman on campus who has the physique of a runner. You decide that she's a member of the track team. Your conclusion is based on ___.

The representativeness heuristic.

The problem with a survey that is not representative is that ___.

The researcher will not be able to draw valid conclusions about the population.

You've heard that members of a certain fraternity or sorority are snobs. Whenever you see members of that fraternity or sorority, you look down and hurry past them. When they don't greet you, you say to yourself, "Just like I thought---they're all arrogant snobs." You have experienced a phenomenon known as ___.

The self-fulfilling prophecy

According to the authors of your text, why is it that there may be conflicting philosophical positions about human nature?

The world is a complex place, and situational differences impact behavior.

The authors of your text discuss the idea that social psychology progresses through the process of theory refinement. Which of the following is the best summary of theory refinement?

Theories are developed, hypotheses are proposed and tested, and then theories are revised.

According to the authors of your text, why did behaviorists not incorporate cognition, thinking, and feeling into their theory?

They consider the concepts too vague and difficult to observe.

Which of the following people have fallen prey to the fundamental attribution error?

Tim, who points to a person who fell down and says, "What a clumsy oaf!"

In their experiment, why did Bibb Latané and John Darley (1968) expose all participants to the same audiotaped seizure? They wanted ___.

To control extraneous variables, such as the quality of the fake seizure.

When making inferences about other people's mental states, we erroneously infer ___ from ___.

Traits; Behavior

T/F Hypotheses are used to test theories

True

You have just arrived for your first year of college, but your roommate, Leah, is nowhere to be found. Your next-door neighbor, Tim, tells you that he's met Leah, and that she's really shy and introverted. If you wanted to counteract the self-fulfilling prophecy, what might you do when you meet Leah?

Try to get her to talk about fun activities she did in high school.

You are taking a university course with a combined graduate and undergraduate enrollment. You know that there are many more undergraduate than graduate students enrolled in this course. There is a woman who looks to be about thirty sitting next to you. If you use only the base rate to guess her student status, you would guess that she is a(n) ___ student because ___.

Undergraduate; there are more of them enrolled in the class

The textbook discusses facilitated communication, a technique that has been touted as allowing communication-impaired people to express themselves via a keyboard operated with the help of a facilitator. In fact, the technique ___.

Was discredited since scientific evidence indicated that in fact, the facilitators unknowingly communicated their own responses rather than those of the other.

If accurate judgments are our goal, we should use base rate information instead of the representativeness heuristic when ___.

We lack specific information about the person.

If you were a researcher studying self-fulfilling prophecies, when would you warn students that they are most likely to occur?

When we are distracted more so than when we are not distracted

What is the take-home message of the authors' description of President Lyndon B. Johnson's decision to escalate the bombing during the Vietnam War?

When we forego accuracy in the interest of feeling good about ourselves, there are sometimes serious consequences.

At a dinner party, Marcia spilled wine on her boss's Persian rug. If Marcia is like most social psychologists who study construals, what will she ask herself in predicting how her boss will respond?

Will my boss think I am drunk?

If research by Pronin and colleagues regarding the bias blind spot applies to you, who would you think is least susceptible to making self-serving attributions?

YOU

Assume that the winning lottery ticket was purchased at the convenience store just around the corner from your house. According to research on counterfactual thinking, you would probably feel worse than if the winning ticket had been purchased all the way across the state, because ___.

You could think back on a dozen opportunities that you had to buy the winning ticket, but didn't.

Which of the following is an example of social influence?

You feel guilty because you lied to your trusting professor about your assignment.

Based on a study by Lassiter and his colleagues (2007) examining how judges and police officers viewed tapes of interrogations, if you were going to be interrogated by a detective, you would not want the camera pointed at ___.

You only.

If your professor asks you to list ten ways to improve the course and you cannot do it, how will that affect your course ratings overall?

You will feel positively about the course.

Assume that you want to avoid the self-fulfilling prophecy the next time you meet someone who fits a schema that you hold. According to work on the On Automatic Pilot: Low-Effort Thinking described in the text, what should you do? You should make sure that ___.

You're not preoccupied with other things.

The ________ approach portrays humans as akin to Sherlock Holmes, who tries his best to accurately make sense of the facts. a.) social cognition b.)self-esteem c.) esteem enhancement d.)rational-economic

a

When asked why she is always losing things, Veronique replies, "I'm not careless. It's just that I have more important things to think about." This response would be predicted by a social psychologist who advocates ________ approach to understanding human behavior.

a self-esteem

Adele is from France, where rabbits are eaten regularly. She can tell you the most tender part of the rabbit, how long to cook one, and how big a rabbit you would need to feed five people. Marsha is from the United States, where rabbits are pets, and bring colored eggs to children in the spring. She hasn't a clue about how to cook one, even if she wanted to. This example best illustrates a. the On Automatic Pilot: Low-Effort Thinking. b. the universality of schemas. c. that rabbits are ambiguous stimuli. d. personality differences in the contents of schemas.

a.

A mother who has tragically lost a daughter in a car accident spends her days rehearsing "if onlys." "If only I had made her stay home that night"; "If only she had taken Route 78 instead of Route 22"; "If only I had offered to chauffeur her instead of letting her drive herself." The mother is engaging in the process known as a. counterfactual thinking. b. blaming the victim. c. the availability heuristic. d. the self-fulfilling prophecy.

a.

The behaviorist approach

a. has its historical roots in Gestalt psychology. b. revolutionized psychology by introducing cognitive concepts. *c. claims that all learning occurs through reinforcement and punishment. d. claims that although thinking and feeling cannot be directly observed, such concepts are essential for a complete understanding of human behavior.

Researchers asked some participants to think of six times they had behaved assertively and asked other participants to think of twelve times they had behaved assertively, and then asked all participants to rate how assertive they thought they really were. Participants who tried to generate six examples of assertive behaviors rated themselves as more assertive than did participants who tried to generate twelve examples. Why? Schwarz suggests that participants

asked to generate six examples could do so more easily than participants who were asked to generate twelve.

When a psychologist discusses individual differences, she is discussing

aspects of personality that make people differ from one another.

2 types of thinking

autmatic controlled

The processes of decoding nonverbal communications and using schemas during impression formation are both examples of ________ processes.

automatic

Karen is so used to driving to her boyfriend's house that when she is taking her little brother to softball practice early one morning, before she is fully awake, she finds herself driving to her boyfriend's house instead. This is an example of

automatic processing.

Parents seldom spend a lot of time explaining to their children how to ride a bicycle. Instead, they often get a bike with training wheels and let the children learn for themselves how to ride a bike. This is because our understanding about how to ride a bicycle is based on

automatic thinking, which is difficult to describe to someone else.

Reasoning based on the ease with which we can bring something to mind involves the use of the ________________ heuristic.

availability

According to the authors of your text, why is it that there may be conflicting philosophical positions about human nature? a.)Philosophy relies purely on layperson observation. b.)The world is a complex place, and situational differences impact behavior. c.) Philosophers make their living through debate. d.)It is likely that philosophers build incorrect arguments about human nature.

b

Research by Lee Ross suggests that even when people recognize that others perceive information differently,they persist in thinking that others are biased while they themselves are objective. This illustrates the idea of a.)implicit values b.)naïve realism c.) individual differences d.)social psychology

b

The major difference between sociology and social psychology is a.) the kinds of topics studied. b.) the level of analysis used. c.)the methods of research used. d.)the ability to apply knowledge to address social problems.

b

Which of the following is an example of social influence? a.)When you get hungry, you have trouble concentrating. b.)You feel guilty because you lied to your trusting professor about your assignment c.)You almost fall asleep at the wheel, so you pull off the road to take a short nap. d.)You didn't do well on the test because you stayed up all night cramming.

b

When Becky meets her new roommate, she sees that she has her hair dyed blue and has decorated her side of the room with lots of handmade ceramic pots and paintings. She then assumes that her roommate has multiple piercings and listens to punk rock. She has used the ________ heuristic in making this assumption. a. base rate b. availability c. representativeness d. anchoring and adjustment

c.

The fact that students "instantly" know the difference between a classroom and a party is an example of a(n) a. controlled thinking. b. automatic thinking. c. schema. d. social cognition.

b.

The social cognition approach is based on the notion that humans are often motivated to

be accurate in their perceptions and inferences.

Social psychologists have identified two motives that are of primary importance in explaining our thoughts and behaviors. According to your textbook authors, these are the need to ________ and the need to ________.

be as accurate as possible; feel good about ourselves

Professor Jaffrey believes that children learn to be polite when they are rewarded for saying things like "Please" and "Thank you." Professor Jaffrey is most likely a ________ psychologist.

behavioral

Amani raises her hand in class. Her answer is wrong and the teacher scolds her. After that, Amani doesn't participate much in class. This situation is most compatible with a ________ approach to understanding and predicting behavior.

behaviorist

Assume that, contrary to research findings, people who undergo a severe initiation to join a group actually like the group less than do people who undergo a mild initiation. If these findings were true, they would provide support for a ________ approach to social influence.

behaviorist

In discussing the issue of parental discipline, which of the following professionals would be least likely to remind parents that it's important to consider how the child views being punished?

behaviorist

Linda tells her professor that her dog is very smart. Every time he hears the word "walkies," he runs to get his leash and stands in front of the door. Linda's professor tells her that her dog has learned to do this because in the past, pleasant walks always followed the word "walkies." Linda's professor is most likely endorsing a ________ explanation.

behaviorist

Assume that the winning lottery ticket was purchased at the convenience store just around the corner from your house. According to research on counterfactual thinking, you would probably feel worse than if the winning ticket had been purchased all the way across the state, because a. you could easily remember what it was like to struggle to make every penny count. b. it would be easy to imagine all the wonderful things you could do with the prize. c. you could think back on a dozen opportunities that you had to buy the winning ticket, but didn't. d. you would assume that the local winner is similar to you.

c.

Ava is at a party, deep in conversation with her friends. Suddenly, she hears her name mentioned in another conversation and her attention turns to monitor that conversation. Clearly, Ava had been unconsciously monitoring the other conversations. Such experiences demonstrate the a. representativeness heuristic. b. effects of priming. c. cocktail party effect. d. availability heuristic.

c.

Based on the study by Harold Kelley (1950) presented in your text, if you were to describe yourself to a blind date as being witty, how would she be most likely to view a slightly sarcastic comment you make? a. as rude b. as insensitive c. as being funny and witty d. as extremely sarcastic

c.

Priming is similar to which of the following since they both influence our behavior, even without our awareness? a. self-esteem b. social influence c. behaviorism d. Gestalt psychology

c. behaviorism

Which one of the following involves the least amount of automatic thinking? a. acting according to goals that have been primed b. using metaphors about the body to make judgments c. counterfactual reasoning d. self-fulfilling prophecies

c. counterfactual reasoning

Ross and Samuels (1993) randomly assigned participants previously identified as cooperative and participants identified as competitive to play a game that was labeled either the "Wall Street Game" or the "Community Game." If their results had indicated that, no matter what the game, participants who were identified as competitive behaved more competitively in both groups than did participants who were identified as cooperative, these findings would have suggested that

cooperation and competition are based on personality characteristics that are consistent across social situations.

Surveys are instruments most often used by social psychologists who conduct ________ research.

correlational

Are people who are better educated more or less prejudiced than people who have less education? This question is most appropriately answered by using

correlational research.

A researcher wants to see if there is a relationship between a person's birth order and his or her leadership ability. The best method for answering this question would be

correlational.

Which of the following factors was not necessary to ensure the internal validity of the Latané and Darley "seizure" study described in the text? They ___. a. made sure their conditions varied on only the key variable. b. made sure that the seizure overheard by participants was identical for everyone. c. randomly assigned participants to groups or conditions. d. asked participants about their knowledge of epilepsy before beginning the experiment.

d. asked participants about their knowledge of epilepsy before beginning the experiment.

All of the following are important to replication across different situations and people except to a. test if results are true across different types of people. b. test if results are true for different situations. c. test if results are externally valid across people and situations. d. enhance the internal validity of all experiments through random selection and assignment.

d. enhance the internal validity of all experiments through random selection and assignment.

Lars has been blind since birth. Which negative emotion is he unlikely to be able to express on his face? a. anger b. fear c. sadness d. frustration

d. frustration

What is one likely, undesirable consequence of foregoing accuracy in favor of justifying our past behaviors? Justifying past behavior can

decrease the probability of learning from past experiences.

Because the knowledge that we are mortal and that bad things can happen to us is very difficult to accept, we often make ___________ attributions

defensive

Biologically based drives affect our behavior most when we are in a state of

deprivation.

Larry is a physician and is convinced that his patient, Mr. Mraz, has cancer. However, after several diagnostic tests and a biopsy that came back negative, Larry rethinks his original diagnosis. He admits that he was wrong and does more research to find the correct diagnosis. In this case, which basic motive did Larry give into?

the need for accuracy

There are many advantages to using experiments; however, a major drawback is

the need to ensure psychological realism to maintain external validity.

From across the room, J.T. sees his mother sigh, and he approaches to give her a hug in the hopes of cheering her up. In this case, J.T.'s behavior is an example of a(n) ________ social influence attempt.

direct

Jada gives William her dessert at lunch in the hopes that he will like her. Jada's behavior is an example of

direct social influence attempt.

through cafeully designed experiments, social psych are able to _______________________

distinguish between 'common sense' and the truth

According to information presented in the textbook, researchers found that in order to feel more optimistic about their own futures, cancer patients engage in

downward social comparison

According to the perspective presented in Chapter 3 (Social Cognition), a person who grew up in a home with an alcoholic parent who sees a man acting in a somewhat strange manner may be ________ likely to interpret this behavior as due to alcohol because of ________.

more; chronic accessibility of an alcoholic schema

Ross and Samuels (1993) found that people playing the "Wall Street Game" were ________ competitive than people playing the "Community Game," ________ of individual differences in competitiveness and cooperativeness.

more; regardless

From an evolutionary perspective, failing to reproduce is the ................

most costly error

In which of the following examples does the need for accurate information most conflict with the need for self-esteem?

*a. Carla has to decide whether to read the detailed red ink comments on the "D" paper she just spent weeks writing. b. Joy has to decide whether to examine her job review file after receiving a promotion. c. Isabel has to decide whether to pay attention to her coach's evaluation of her performance after they have won the semi-final. d. Troy has to decide whether to read his opening night reviews after the play's producer has promised that the play will have at least a six-week run.

Many fraternities, sororities, and sports teams have initiation rituals. For example, at one school, all the members of the soccer team must wear unusual clothing everywhere on campus for an entire day and then have embarrassing haircuts. These rituals try to capitalize on the role of ________ in increasing commitment via self-justification.

*a. suffering b. the affiliation motive c. reinforcement d. social power

Main Effect

- is the effect of one independent variable on the dependent variable, ignoring the effects of other independent variables - In general, there is one main effect for every independent variable in a study -when an IV has an effect of similar magnitude and direction across levels of the other IV -ie appearance is not significantly in this case= 'no effect'= no main effect -people like kind person better

Interaction

- the effect of one independent variable on the dependent variable changes depending on the level of another independent variable -when the effect of one IV on the DV, depends on the value (or level) of the other IV -ie attractiveness effects behavior; effect of kindness effects attractiveness -effect of attractiveness depends on behaviors (vice versa)

Dick is a 30 yr old man who enjoys chess, classical music and wine. What is the probability that Dick is: a. a truck driver b. a college professor

-Dick matches professor stereotype, but do not consider relative number of people in those professionals (based on base-rates). ie there are many many more truck drivers than professors in the world -Takes too much effort to calculate hard probabilities... More efficient to go with what seems true or representative -answer B is correct only if circles/rates were same size=errors/info over looked in Representative Heuristics

2 types of Heuristics

-Representativeness -Availability

self-fulfilling prophecy

-behave towards others in ways that leads them to confirm our expectations -ie blind dates

Experimental Research

-can determine causation -experimental control: manipulation of independent variable (determined by researchers) and all other variables kept constant -random assignment of participants or conditions -tests whether or not variable A has an influence on variable B

Correlational Approach

-can determine variables that are related, but cannot determine that one variable causes another -compares 2 variables -doesn't tell all answers or 'why' 2 variables may/may not be related -Correlation DOES NOT equal causation

Benefit of random assignment

-choosing groups to compare that are equal on all other factors except the independent variable (ie tshirt color) -(people whoa re naturally aggressive have an equal chance of being in each group)

how social psych differs from its relative sciences

-different levels of analysis -main focus of social psych is "why" answers -level of analysis= the main focus of research -the main level of analysis for social psych is to seek and identify universal properties of human nature that make everyone susceptible to social influence regardless of social class or culture

Causes of AOE and FAE

-different visual perspectives on our own vs. others' behavior -resources (cognitive misers!)- dispositional attribute happens automatically; situational attributions acquire overriding first reaction (ie Hilda study)

According to Scopelleti et al (2015), as stated in the article Blindspot Bias: Are You Blind to Your Own Bias?, _____% of people believe they are less bias than their peers.

85

If a researcher were to observe women exercising and then men exercising and compare these groups, why would this not be considered an experiment? a. The experimenter is not manipulating an independent variable. b. The study is biased. c. There is no prior evidence that gender is related to one's preferred type of exercise. d. An experiment must involve at least three different conditions.

A

Nutz and Boltz, Inc. employs 100 people; 80 are lawyers and 20 are engineers. Albert, a lawyer at the company, is quiet, likes puzzles, and carries a calculator. Most people, when asked whether Albert is a lawyer or an engineer, will guess that he is an engineer. These people are using ________ to make this faulty guess. A correct guess would have been made if they had used ________. a. the representativeness heuristic; base rate information b. base rate information; the representativeness heuristic c. the representativeness heuristic; the availability heuristic d. base rate information; anchoring and adjustment

A

Professor Swenson is interested in university students' reactions to the death of a popular rock star. For two weeks, Professor Swenson spends one hour a day in a popular cafeteria, inconspicuously listening to students, joining in their conversations when the topic of the dead rock star comes up, and recording what the students have to say. Professor Swenson is conducting ________ research. a. observational b. experimental c. correlational d. archival

A

Recall that Harold Kelley (1950) told some college students that their guest lecturer was a warm person, and others that he was a cold person. The visitor then lectured for twenty minutes. After the lecture, those students who expected the lecturer to be warm a. rated him higher and participated more in class discussion than those who expected him to be cold. b. rated him higher but did not talk in class any more than those who expected him to be cold. c. participated in class more than those who expected him to be cold, but did not rate him any differently. d. and those who expected him to be cold did not differ in their assessments, since they both saw the same person.

A

When you conduct your own research, what size p-value indicates that your research has been successful? a. small, such as less than 5% b. medium, such as between 30 and 70% c. large, such as greater than 70% d. It doesn't matter the size. The p-value does not indicate anything about research.

A

Which of the following is one of the ethical principles that psychologists must follow when using human participants? All participants must be a. informed that they can withdraw from the study at any time. b. informed of the true nature of the study upon arriving at the laboratory. c. contacted six months after the study to assure that no psychological harm resulted. d. compensated in some way for their participation in research.

A

Which of the following is an example of a direct persuasion attempt?

A bully threatens Billy and steals his lunch money.

_______ research involves systematic examination of the documents or records of a culture. A) Archival B) Participant observational C) Cross-sectional D) Experimental E) Systematic

A) Archival

Carlos, the president of a fraternity on campus, randomly sampled 500 students, asked them if they belonged to a fraternity or sorority, and asked them about their current GPA. He discovered that the GPAs of those people in fraternities and sororities are higher than those of people who are not involved in the Greek system. Gleefully, he presented his findings to the Dean, saying that being involved in a fraternity or sorority leads to higher grades. What rule of research methods is Carlos breaking? A) Correlation does not equal causation. B) His sample size of 500 is too small to make such a generalization. C) Correlational data do not have internal validity. D) College students are not representative of the whole sample of people in the world. E) Correlational data do not provide any practical information on a topic.

A) Correlation does not equal causation.

Why do schemas differ from culture to culture? A) Cultures differ in terms of what is important and relevant to the people who live in them. B) In some cultures, it is more important to make quick decisions. C) Life is simpler in nonindustrialized cultures, making schemas less important. D) A good memory is more important in some cultures than in others. E) Survival is more difficult in nomadic cultures, making schemas more important.

A) Cultures differ in terms of what is important and relevant to the people who live in them.

Iain is from Australia, where people drive on the left-hand side of the road. When he moved to Canada, his accident rate was higher than any of his friends'. Why might this be? A) Driving schemas differ between Canada and Australia. B) There is less traffic in Australia. C) Iain had no schema for driving behaviours. D) They don't have Driver Education courses in Australia. E) There is no speed limit in Australia.

A) Driving schemas differ between Canada and Australia.

If a researcher were to use deception as part of her experimental procedure, when would she explain the purpose of this deception to her participants? A) During the debriefing. B) Immediately preceding the experimental manipulation. C) Before obtaining informed consent. D) After she has analyzed the data. E) Before collecting any dependent measures.

A) During the debriefing.

Your roommate questions how charitable you really are. She says to you, "Give me just one example of the last time you gave money to a homeless person." Odds are that her challenge will convince you that you really are a generous person. Based on research by Norbert Schwarz and his colleagues (1991), why is that? A) It's easy to bring to mind one example that supports your self-schema. B) It's easier to think of "factuals" than "counterfactuals." C) Your roommate activated your "generosity" schema. D) You will believe that you donated more money than the average person. E) Homeless people are representative of people who need our help.

A) It's easy to bring to mind one example that supports your self-schema.

Imagine that researchers have found a correlation of -.72 between the frequency of disagreements that couples have and how long they stay together. Based on this correlation, would you start arguments with your significant other in order to sustain your relationship? A) No, because although the two may be correlated, causation has not been proved. B) No, because the correlation is positive. C) Yes, because the correlation is positive. D) No, because in your group of friends, the correlation is negative. E) Yes, because the correlation is relatively strong.

A) No, because although the two may be correlated, causation has not been proved.

The introduction to Chapter 2 began with descriptions of disagreements over the relation between television violence and aggressive behaviour, and of situations in which people won't intervene to stop violence. Why would authors begin Chapter 2 by describing these phenomena? A) There are competing explanations for both phenomena, and research methods can be used to establish the best explanation. B) These topics address the most difficult issues that social psychologists will ever study. C) Definitive explanations for both phenomena have been provided by social- psychological research. D) These topics are a good example of what has yet to be studied in social psychology. E) There is more literature on violence and aggression than on any other topic of interest to social psychologists.

A) There are competing explanations for both phenomena, and research methods can be used to establish the best explanation.

You have just arrived for your first year of university, but your roommate, Leah, is nowhere to be found. Your next-door neighbour, Tim, tells you that he's met Leah, and that she's really shy and introverted. If you wanted to counteract the self-fulfilling prophecy, what might you do when you meet Leah? A) Try to get her to talk about fun activities she did in high school. B) Ask her if she is an introvert. C) Be quiet and shy yourself, so as not to intimidate her. D) Ask her what books she's read lately. E) Ask her if she likes to spend a lot of time alone.

A) Try to get her to talk about fun activities she did in high school.

Cross-cultural research has revealed that A) Western cultures emphasize individualism and independence. B) Westerners are more likely to intervene in an emergency than are people in the East. C) American researchers have historically imposed their own values on other cultures. D) many Eastern cultures have no concept of "emergency." E) there are few differences between Western and Eastern culture.

A) Western cultures emphasize individualism and independence.

Austin cannot stop thinking about a scary accident in which he was involved. These intrusive thoughts are both unpleasant and distracting. When is Austin least likely to think about the accident? When his A) When his monitoring and operating processes work in tandem. B) When his operating process continues, and his monitoring process shuts down. C) When his operating and controlled processes work in tandem. D) When his monitoring process continues, and his operating process shuts down. E) When his automatic and monitoring processes work in tandem.

A) When his monitoring and operating processes work in tandem.

Which of the following questions would be least likely to yield accurate responses? A) Would you ever consider physician-assisted suicide? B) How many cars do you own? C) How many different magazines do you subscribe to? D) In your opinion, who performed better in the last election debate? E) Have you ever been the victim of a violent crime?

A) Would you ever consider physician-assisted suicide?

A cover story is A) a "false" description of the purposes of a study. B) an overview of the study for participants. C) the result of experimental findings of interest to the media. D) a way to make sure that an experiment is ethical. E) an explanation provided to participants when the study is over.

A) a "false" description of the purposes of a study.

Among the features of an analytic thinking style is A) a focus on objects. B) a focus on context. C) concern with the relationship between objects. D) greater cortical activity when paying attention to objects. E) efficient information searching strategies.

A) a focus on objects.

Warren believes that Kurt is an outgoing, gregarious person. "How many parties did you go to this weekend?" Warren asks Kurt. "Tell me about all of the fun and crazy things that you have planned for the summer," Warren continues. Although Kurt is usually rather quiet and reserved, he responds to Warren in an outgoing, friendly manner. This is an example of A) a self-fulfilling prophecy. B) false memory. C) the perseverance effect. D) the primacy effect. E) reconstructive memory.

A) a self-fulfilling prophecy.

Why is the experiment the method of choice for many social psychologists? Experiments A) afford cause-and-effect conclusions. B) ensure random sampling. C) are the only way to test hypotheses. D) are easy to conduct. E) are inexpensive to conduct.

A) afford cause-and-effect conclusions.

Social scientists who conduct survey research use a process of random selection to A) allow them to apply the results from their survey sample to the general population. B) allow them to increase the strength of the relationships found. C) allow them to increase the psychological realism of their research. D) allow them to come to causal conclusions on the basis of their survey results. E) allow them to increase the mundane realism of their research.

A) allow them to apply the results from their survey sample to the general population.

An experimenter conducts a program of research to determine what types of messages are most effective in promoting condom use among adolescents. How would you classify this type of research? A) applied research B) secondary research C) basic research D) descriptive research E) primary research

A) applied research

Barbara is distracted at work because she cannot stop thinking about her upcoming wedding. She decides that the best way to avoid those distracting thoughts is to immerse herself in the multiple demands of her job. Why is this strategy actually counterproductive in suppressing those thoughts of her wedding? Too many cognitive demands A) are placed on her operating process. B) increase controlled processing. C) increase monitoring processing. D) are placed on her monitoring process. E) halt automatic processing.

A) are placed on her operating process.

One of the greatest advantages of surveys is that researchers A) can sample representative segments of the population of interest. B) can show causation. C) can be biased, but still collect objective information. D) need not concern themselves with interjudge reliability. E) can be confident that respondents answered honestly.

A) can sample representative segments of the population of interest.

The Kitty Genovese murder inspired research on bystander apathy. This example illustrates the usefulness of relying on ________ in formulating research hypotheses. A) casual observations of everyday life B) folk wisdom C) common sense D) social-psychological theory E) personal experience

A) casual observations of everyday life B) folk wisdom

Which of the following pairs of variables are most likely to be positively correlated? A) consumption of fatty foods; risk of heart disease B) age; strength C) room temperature; comfort level D) flossing; cavities E) cigarettes smoked; increased life expectancy

A) consumption of fatty foods; risk of heart disease

New professors often have to concentrate very hard to give a lecture, and monitor themselves carefully to make sure they're not talking too fast and that they are using appropriate and clear examples. More seasoned professors don't have to concentrate as much, and can talk clearly and engagingly, reorder the overheads, and keep an eye on students all at the same time. This example represents the transition from _______ processing to _______ processing as professors gain experience. A) controlled; automatic B) random; systematic C) automatic; controlled D) unconscious; conscious E) controlled; random

A) controlled; automatic

A researcher is interested in the relation between the number of a person's past sexual partners and the person's decision to have an HIV test. To determine this, the researcher should use the ________ method. A) correlational B) longitudinal C) experimental D) observational E) participant observation

A) correlational

You stayed up all night cramming for an examination and didn't do as well as you had hoped. "If only I had started studying sooner and gotten a good night's rest, I'd have done much better," you think to yourself. You have just engaged in A) counterfactual thinking. B) self-justification. C) wish fulfillment. D) the hindsight bias. E) unrealistic fantasy.

A) counterfactual thinking.

Observational research allows a researcher to A) describe the nature of a phenomenon. B) match participants to conditions of an experiment. C) make predictions about one variable based on knowledge of another. D) randomly assign participants to conditions of an experiment. E) make statements about causality.

A) describe the nature of a phenomenon.

What is one of the major advantages of archival research? This research method A) enables researchers to detect changes across time and cultures. B) takes less time than systematic observation. C) can show causal relationships. D) does not require trained observers. E) generates information that is easier to code reliably.

A) enables researchers to detect changes across time and cultures.

Social psychologists seldom address the question of external validity by conducting their experiments on random samples of people from all over. This is because A) human beings are incredibly complex, and social psychologists are content with doing inexact science. B) the research methods that social psychologists use cannot be applied to a random sample. C) it is difficult to get a random sample of people to participate in research. D) social psychology is poorly funded, and experimenters are forced to do inexpensive research. E) experiments capture general psychological processes experienced by people all over.

A) human beings are incredibly complex, and social psychologists are content with doing inexact science.

Self-fulfilling prophecies most often are a result of A) inadvertent and unconscious influences of individuals' schemas. B) deliberate attempts by people to confirm their schemas. C) deliberate attempts to treat people in an unbiased manner. D) the immediate situation. E) priming.

A) inadvertent and unconscious influences of individuals' schemas.

The procedure whereby the researcher explains the nature of the study and participants then decide whether or not to participate is called A) informed consent. B) experimental debriefing. C) experimental review. D) a truthful cover story. E) pre-experimental briefing.

A) informed consent.

The shortcuts or "rules of thumb" that people use to make judgments and decisions quickly and efficiently are called A) judgmental heuristics. B) functional cognitions. C) cultural truisms. D) structural equations. E) rationalizations.

A) judgmental heuristics.

In short, counterfactual thinking can be described as A) mentally reconstructing the past. B) one of the most rational responses to an undesirable outcome. C) putting a positive spin on things to enhance self-esteem. D) thinking about how things couldn't have turned out differently. E) avoiding thinking of things that may damage self-esteem.

A) mentally reconstructing the past.

Suppose a researcher wants to know whether frustration really does cause aggression. She collects a large number of experimental studies that involve both children and adults, and that are conducted both in the laboratory and in the "real world" in both Canada and other cultures. She then conducts a(n) ________ to determine whether there is enough consistency in findings across studies to determine the generalizability of the relation between frustration and aggression. A) meta-analysis B) case study C) archival study D) systematic replication E) direct replication

A) meta-analysis

The correlational method most often makes use of ________ and ________ data. A) observational; survey B) experimental; archival C) archival; ethnographic D) survey; experimental E) archival; experimental

A) observational; survey

You and your friend have a big assignment due today in your English class. You ask how your friend's paper turned out, and he mentions that he hasn't yet gotten around to writing it. Shocked, you ask him what he's going to tell the professor. He replies, "Oh, it won't be a problem. All female professors are pushovers; she'll let me turn the paper in next week." You tell your friend that you don't think all female professors are pushovers, but he doesn't seem worried. His conviction that his reasoning is correct is called the A) overconfidence barrier. B) motivational effect. C) arrogance barrier. D) perseverance effect. E) testosterone effect.

A) overconfidence barrier.

In a study of frustration and aggression, some participants were exposed to a confederate who insulted them, and others were exposed to no such insult. Participants were then allowed to recommend whether the confederate should be fired. Those who were insulted were more likely to retaliate by recommending that the confederate lose his job. In this experiment, the ________ was the dependent variable. A) participants' recommendations B) confederate C) presence or absence of an insult D) difference between the groups E) participants' condition

A) participants' recommendations

As an empirical science devoted to understanding human social behaviour, social psychology is most like A) physics. B) theology. C) linguistics. D) moral philosophy. E) literary criticism.

A) physics.

Before Jonas Salk discovered a vaccine to prevent polio, people noticed a correlation between outside temperature and the incidence of polio: The warmer the temperature over the course of the year, the more outbreaks of polio. This relationship is an example of a(n) ________ correlation. A) positive B) causal C) illusory D) negative E) spurious

A) positive

You do not know many people with disabilities, but view a telethon in which people with disabilities are shown needing and appreciating help. The next day, you see a blind person walking down the street, judge her to be in need of help, and ask her if you can help her find her destination. The notion of helping people with disabilities was ________ by the telethon, and became accessible in this situation. A) primed B) clarified C) validated D) reinforced E) biased

A) primed

To avoid the use of deception in his experiment, a researcher tells participants ahead of time that they will be witnessing a staged fight between two people who were actually confederates of the experimenter. By so informing the participants, this researcher has decreased the ________ realism of his experiment. A) psychological B) internal C) mundane D) ethical E) experimental

A) psychological

Did the participants in Latané and Darley's (1968) bystander intervention experiment think what Kitty Genovese's neighbours thought? Did they feel what her neighbours felt? These questions address the ________ of their experiment. A) psychological realism B) mundane realism C) cover story D) internal validity E) construct validity

A) psychological realism

The only way to be certain that the results of a survey represent the behaviour of a particular population is to ensure that the respondents are ________ that population. A) randomly selected from B) normally distributed in C) randomly assigned to D) equally distributed in E) matched within

A) randomly selected from

Professor Rothman is interested in tracking changes in racial stereotypes in CanadA) If he decides to conduct an archival analysis, he should A) record how minorities are portrayed in newspaper cartoons from 1940 to 2000. B) interview multiple generations in families of different race and ethnicities. C) interview one generation of different races and ethnicities. D) record his subjective impressions of the racial stereotypes contained in newspaper humor columns from 1940 to 2000. E) ask his students to watch television every night for a week and tell him what they saw.

A) record how minorities are portrayed in newspaper cartoons from 1940 to 2000.

A particular company employs 100 people; 80 are lawyers; and 20 are engineers. Albert, an employee at the company, is quiet, enjoys puzzles, and carries a calculator. Most people, when asked whether Albert is a lawyer or an engineer, will guess that he is an engineer. People are using the _______ heuristic to make this faulty guess. A) representativeness B) availability C) anchoring and adjustment D) base rate E) counterfactual

A) representativeness

In general, _______ represent knowledge to which we often apply _______. A) schemas; heuristics B) generalizations; logic C) base rates; personal experience D) personal experience; base rates E) biases; corrections

A) schemas; heuristics

Why might taking a statistics course help you become more complex and accurate in your social judgments? Such a course teaches the skills needed by a(n) A) scientist. B) social worker. C) cognitive miser. D) motivated tactician. E) accurate tactician.

A) scientist.

What is the major difference between ethnography and other kinds of systematic observation used by social scientists? In ethnography A) scientists interact with the people they are observing. B) the people who are observed are paid for their part in the research study. C) scientists randomly assign people to conditions. D) scientists observe anything that seems surprising or interesting. E) scientists record their own behaviours, as well as the behaviours of others.

A) scientists interact with the people they are observing.

Which of the following can be considered good examples of schemas? A) stereotypes B) counterfactuals C) base rate information D) heuristics E) priming

A) stereotypes

Nutz and Boltz, Inc. employs 100 people; 80 are lawyers and 20 are engineers. Albert, a lawyer at the company, is quiet, likes puzzles, and carries a calculator. Most people, when asked whether Albert is a lawyer or an engineer, will guess that he is an engineer. These people are using ________ to make this faulty guess. A correct guess would have been made if they had used ________. A: the representativeness heuristic; base rate information B: base rate information; the representativeness heuristic C: base rate information; anchoring and adjustment D: the representativeness heuristic; the availability heuristic

A: the representativeness heuristic; base rate information

The "basic dilemma of the social psychologist" refers to: A: the trade-off between internal and external validity. B: whether or not to use deception in their experiments. C: the trade-off between independent and dependent variables. D: whether or not to conduct ethically questionable research.

A: the trade-off between internal and external validity.

Dr. Nash wants to conduct research on spousal abuse. Why would Dr. Nash be unwise to use observational methods for this topic?

Abuse is usually something that occurs privately.

Ethan and Heather see a man stumbling around as he walks down the street. Ethan, who belongs to Alcoholics Anonymous, thinks the man is drunk, but Heather, who just watched a TV special on Michael J. Fox, thinks the man has Parkinson's disease. These differing interpretations of the same behavior seem to be caused by Ethan and Heather's differences in ___.

Accessibility

Ethan and Heather see a man stumbling around as he walks down the street. Ethan, who belongs to Alcoholics Anonymous, thinks the man is drunk, but Heather, who just watched a TV special on Michael J. Fox, thinks the man has Parkinson's disease. These differing interpretations of the same behavior seem to be caused by Ethan and Heather's differences in...

Accessibility

The fact that students "instantly" know the difference between a classroom and a party is an example of a(n) ___.

Automatic thinking

_______ thinking is non-conscious, effortless, and unintentional, whereas ________ thinking is conscious, effortful, and intentional.

Automatic; Controlled

A psychologist first conducted a study on helping behavior with a sample of college undergraduates. This psychologist then replicated the study using the same procedure with a different participant sample. If similar results were obtained with both samples, this experimenter has demonstrated a. internal validity. b. external validity. c. population consistency. d. chronological consistency.

B

A researcher conducted a study and found a positive correlation between age and income level. In other words, this researcher found that as age ________, income level tends to ________. a. increases; decrease b. increases; increase c. increases; stay the same d. decreases; increase

B

A researcher conducted a survey and found a negative correlation between education and the tendency to resort to violence during disputes. In other words, this researcher found that as education level ________, the tendency to use violence ________. a. decreases; also decreases b. increases; decreases c. increases; also increases d. increases; remains the same

B

According to information from the registrar's office, Lee has discovered that people who score higher on the SAT tend to have a higher GPA in their first year of college. Lee has used a(n) ________ research method. a. ethnographic b. correlational c. experimental d. field study

B

Did the participants in Latané and Darley's (1968) bystander intervention experiment think what Kitty Genovese's neighbors thought? Did they feel what her neighbors felt? These questions address the ________ of their experiment. a. internal validity b. psychological realism c. reliability d. cover story

B

Dr. Winstell is interested in studying prejudice, and devises a study in which Caucasian participants are led to believe that they are chatting electronically with an African-American participant. In reality, the participants are responding to another Caucasian participant, and she discloses this information to all participants at the end of the study. Dr. Winstell has used ________ in her study. a. informed consent b. deception c. replication d. cross-cultural research

B

In a study described in the textbook, participants memorized either words such as neat or disrespectful before reading an ambiguous passage about Donald in which he was described as engaging in a number of recreational activities (e.g., driving in a demolition derby, white-water rafting) in which he had risked injury and even death. How did these words influence the impressions of Donald? a. They made the impression of his behavior appear more extreme. b. They did not affect the impression of him because they were not related to his behavior. c. They helped make the impression of Donald more sympathetic. d. They made the impression of Donald more negative.

B

Professor Rothman is interested in tracking changes in racial stereotypes in the U.S. If he decides to conduct an archival analysis, he should a. interview multiple generations in families of different race and ethnicities. b. record how minorities are portrayed in cartoons in the New Yorker magazine from 1940-2010. c. ask his students to report their impressions of how different ethnic groups are portrayed on television. d. compare the results of opinion polls taken over five decades.

B

Research demonstrated that physical sensation can activate metaphors that influence our judgments about unrelated topics. If you wanted students to take your petition seriously, you should present the information about it a. on a piece of paper they can take with them. b. on a heavy tablet. c. on a light clipboard. d. verbally rather than written.

B

Why is random assignment important in an experiment? a. It assures that all participants take part in all conditions of a study. b. It allows researchers to rule out individual differences as an alternative explanation. c. It ensures that all participants see the same manipulation of the independent variable. d. It is an index of the probability that experimental results occurred by chance.

B

A teenager wants to borrow the family car to go to an out-of-town concert. Her parents resist the idea. The teenager in question is most likely to view her parents as stubborn and unreasonable when they say to her, A) "Tell us again what you would do if the car broke down." B) "Give us one reason why we shouldn't worry ourselves to death." C) "How many other friends are going with you?" D) "Give us five reasons why you should be allowed to borrow the car." E) "Give us ten reasons you should be allowed to borrow the car."

B) "Give us one reason why we shouldn't worry ourselves to death."

A correlation of ________ indicates that two variables are not correlated. A) 1.00 B) 0.00 C) -1.00 D) -2.00 E) 0.50

B) 0.00

Assume that you are a participant in Latané and Darley's (1968) experiment examining when people help. You believe that there are four other people participating with you in a discussion, although you are isolated in a booth to ensure privacy. When you are listening to one of the other participants talk, you hear him beginning to have a seizure. What are you most likely to do? A) Run out of the booth to find the experimenter for help. B) Anxiously remain in the booth and hope for the best. C) Leave the experiment because of psychological stress. D) Run out of the booth to help the person having the seizure. E) Try yelling through the walls to see if he is all right.

B) Anxiously remain in the booth and hope for the best.

_______ processing is nonconscious, effortless, and unintentional, whereas _______ processing is conscious, effortful, and intentional. A) Biased; accurate B) Automatic; controlled C) Logical; illogical D) Illogical; logical E) Controlled; automatic

B) Automatic; controlled

Which of the following is a serious shortcoming of the correlational approach? A) There is always some third variable that accounts for the correlation. B) Causal inferences based on correlational data are risky at best. C) Surveys are often poorly designed, leading to erroneous conclusions. D) It is difficult to generalize the results. E) It is difficult for social psychologists to secure a random sample.

B) Causal inferences based on correlational data are risky at best.

Which of the following is a drawback to the observational method? A) People's behaviours inevitably change when they are being observed. B) Certain behaviours only take place rarely or in private. C) No interesting or important questions can be answered using this method. D) The observational method is statistically unreliable. E) The observational method cannot impact a well-accepted theory.

B) Certain behaviours only take place rarely or in private.

Richard and his friend Henry are watching a movie. Henry's background is East Asian Canadian while Richard's background is European Canadian. Based on research in Chapter 3, which aspects of the movie is Richard likely to notice more than Henry? A) Background interaction between movie extras. B) Details of the hero's appearance. C) The scenery through which the hero is travelling. D) The storyline of the movie. E) The credits at the end of the movie.

B) Details of the hero's appearance.

Why didn't Latané and Darley (1968), in their study of bystander intervention in emergencies, use a manipulation more like events in the Kitty Genovese murder? A) It was impossible to assign participants on the street to experimental conditions. B) Ethically, it was impossible to expose unwitting participants to such a distressing manipulation. C) The New York City police refused to give the researchers permission to conduct their experiment. D) It was impossible to ensure that the murder sounded identical to all participants. E) It was impossible to have external validity.

B) Ethically, it was impossible to expose unwitting participants to such a distressing manipulation.

________ methods allow a researcher to make a valid cause-and-effect statement about the variables in her study. A) Observational B) Experimental C) Descriptive D) Cross-sectional E) Correlational

B) Experimental

In a study of group dynamics, participants were placed in groups consisting of either three or ten people. During the study, group members worked together trying to solve a puzzle. After completing the task, participants reported how satisfied they were with the other members of their group. ________ is the independent variable in this study. A) Gender B) Group size C) The puzzle D) Group dynamics E) Satisfaction

B) Group size

Recall Victoria Medvec and her colleagues' (1995) study of the emotions experienced by Olympic athletes who won bronze and silver medals. If silver medalists out-performed bronze medalists, why were they less happy? A) There is a positive correlation between talent and competitiveness. B) It was easier for silver medalists to imagine winning the gold medal. C) Bronze medalists were grateful simply to have the opportunity to compete. D) Silver medalists had their sights set on gold, as do most "favoured" competitors. E) Bronze medalists did not think they were going to win anything.

B) It was easier for silver medalists to imagine winning the gold medal.

Based on your reading of Chapter 3 (Social Cognition), who would you predict would be more frustrated: Katie, who missed an "A" by one point, or James, who missed an "A" by five points? A) Katie, because she and James studied together. B) Katie, because it's easier for her to imagine getting one more test item right. C) James, because he knew how close Katie was to an "A." D) James, because he and Katie studied together. E) James, because it's harder for him to imagine an alternative outcome.

B) Katie, because it's easier for her to imagine getting one more test item right.

A researcher concludes that frustrating people by giving them a task that is impossible to complete causes them to behave more aggressively. Only ________ warrants this type of conclusion. A) an archival analysis B) experimental research C) participant observation D) observational research E) correlational research

B) experimental research

Rosenberg and colleagues (1992) conducted a study that found that women who relied on the diaphragm or contraceptive sponges had fewer STDs than women who used condoms. The media jumped to the conclusion that condom use contributes to STDs. Why was this media conclusion inappropriate? A) No such relation between birth control device and STDs exists for men. B) Perhaps women who use condoms have more sexual partners and contract more STDs. C) The researchers did not have a large enough sample. D) The researchers did not study women who were on the pill. E) Women who use public clinics are not representative of the female population.

B) Perhaps women who use condoms have more sexual partners and contract more STDs.

Which of the following is NOT essential to conducting an internally valid experiment? A) Measurement of the dependent variable. B) Random selection of the sample from the population. C) Keeping everything in the experimental conditions the same except for the independent variable. D) The use of operational definitions. E) Random assignment to condition.

B) Random selection of the sample from the population.

________ are the cognitive structures we use to organize our knowledge of the social world. A) Social roles B) Schemas C) Heuristics D) Social cognitions E) Internal attributions

B) Schemas

What have a number of research studies on the self-fulfilling prophecy revealed? A) Schema-confirming behaviours are conscious and deliberate. B) Self-fulfilling prophecies are the result of unconscious, inadvertent processes. C) Self-fulfilling prophecies are a result of our desire to "be right." D) Because their job is to assess performance, educators are especially susceptible to the self-fulfilling prophecy. E) Self-fulfilling prophecies cannot have any real impact on our behaviour.

B) Self-fulfilling prophecies are the result of unconscious, inadvertent processes.

________ refer(s) to the way people select, interpret, remember, and use social information to make judgments and decisions about themselves and others. A) Counterfactual thinking B) Social cognition C) Automatic thinking D) Schemas E) Decision rules

B) Social cognition

Which metaphor best represents the view of people as good, but not infallible, thinkers? A) Social thinkers are cognitive misers. B) Social thinkers are flawed scientists. C) Social thinkers are biased tacticians. D) Social thinkers are accurate statisticians. E) Social thinkers are practical theorists.

B) Social thinkers are flawed scientists.

An experiment like Latané and Darley's (1968) demands creativity and ingenuity, and is not easy to conduct. Why didn't Latané and Darley simply describe the hypothesis and procedures to participants and ask them what they would do in that situation? A) It would be too difficult to secure a random sample. B) The simpler alternative study would be low in psychological realism. C) Participants would lie in order to look good. D) It would be impossible to randomly assign participants in the simpler study. E) They would not be able to use operational definitions.

B) The simpler alternative study would be low in psychological realism.

Which of the following explanations for the failure of neighbours to come to Kitty Genovese's aid best reflects the idea of diffusion of responsibility? A) The cost of intervening was too high, so neighbours didn't help. B) There were so many witnesses that no single person felt responsible to intervene. C) The neighbours did not personally know Kitty Genovese. D) Urban dwellers are especially callous when it comes to giving aid. E) The neighbours did not interpret her cries as an emergency, so they didn't intervene.

B) There were so many witnesses that no single person felt responsible to intervene.

How did Latané and Darley (1968) vary the number of bystanders in their experimental study of the effects of the number of witnesses exposed to an emergency? A) Participants who arrived first were assigned to a large group condition, and those who arrived later were assigned to a small group condition. B) They randomly assigned participants to one of three conditions: large discussion group, small discussion group, or one-on-one discussion group. C) They allowed participants to choose whether to participate in a group or in a one-on-one discussion. D) They asked participants to wait in a hallway alone, or else with a group of others. E) They randomly choose a number of different bystanders for each participant.

B) They randomly assigned participants to one of three conditions: large discussion group, small discussion group, or one-on-one discussion group.

Service agencies seeking donations are more likely to run television ads that show hungry or sick children than they are to show statistics, charts, or graphs that more accurately reflect the plight of children all over the world. Why? A) Research has shown that most viewers believe that it's easy to "lie with statistics." B) Vivid visual images are more available in memory and are more likely to influence decisions about whether to contribute. C) Most people do not understand statistics very well. D) Representative statistics are not available, so producers must rely on unreliable visual samples. E) Nielsen research has revealed that viewers will leave the room unless pictures of children are used.

B) Vivid visual images are more available in memory and are more likely to influence decisions about whether to contribute.

Based on the 2010 study by Callan and colleagues, which of the following responses would be expected if you had been subliminally primed with a legal term (such as "judge" or "lawyer")? A) You would be more likely to worry about expenses. B) You would show more opposition to government policies that were unfavourable to you. C) You would show less competitiveness in games. D) You would be more likely to interpret an ambiguous scenario as having cooperative qualities. E) You would be more likely to follow the rules of the road when driving.

B) You would show more opposition to government policies that were unfavourable to you.

When Latané and Darley (1970) later tested the effects of the number of bystanders on whether people intervene in an emergency, they left the laboratory and staged a fake theft at a convenience store. This new study was A) covered in the New York Times. B) a non-experimental field study that supported experimental results. C) low in external validity. D) unsuccessful, demonstrating the low external validity of their lab experiments. E) a field experiment that replicated laboratory results.

B) a non-experimental field study that supported experimental results.

You were supposed to make an important presentation at work, but you overslept and the bus was late, so you missed the meeting. If you are able to generate many ways in which to "undo" this terrible outcome you will experience _______ emotional reaction. A) a mild B) a strong C) an incapacitating D) no E) an irrational

B) a strong

Chapter 2 of the text discusses Tillie the Rainmaker, a woman who believes she has the power to cause rain in drought-stricken areas. According to your text, this belief was probably caused by A) a lack of random assignment. B) a tendency to infer causality from correlational events. C) the lack of precise operational definitions. D) a lack of random selection. E) such behaviours becoming a self-fulfilling prophecy.

B) a tendency to infer causality from correlational events.

Sometimes when forming impressions of others, we are influenced by the schemas that are in the forefront of our mind. When this happens, social psychologists talk about the role of ________ in colouring our impressions of others. A) the unconscious B) accessibility C) memory D) past experience E) priming

B) accessibility

In Chapter 3, the lesson to be learned from accident victim Kevin Chappell's story is that we can automatically categorize objects and faces A) because of the mere exposure effect. B) allowing processing capacity for other important things. C) because we are particularly interested in faces and objects. D) because of our strong desire to do so. E) because of cognitive biases that influence our perception.

B) allowing processing capacity for other important things.

We are most likely to rely on schemas when the situation we confront is A) confusing. B) ambiguous. C) forgettable. D) interesting. E) arousing.

B) ambiguous.

In order to examine the prevalence of drug use in several different generations of North Americans, a researcher decides to collect the lyrics from the fifty most popular songs from each decade, from 1940 to 2000, and to code those lyrics for how often drug-related themes were present. Which of the following methods is this researcher using? A) correlational B) archival C) observational D) cross-sectional E) experimental

B) archival

Reasoning based on the ease with which we can bring something to mind involves the use of the _______ heuristic. A) representativeness B) availability C) confirmatory D) counterfactual E) base rate

B) availability

Your roommate is interested in dating Carlotta, a woman you know from a history class. Your roommate asks if you think Carlotta is generally a considerate person. You remember how Carlotta lent you her notes when you missed class last week, and tell your roommate you think Carlotta is very considerate. In making your judgment, you have relied upon the _______ heuristic. A) anchoring and adjustment B) availability C) representativeness D) descriptive E) base rate

B) availability

When asked to guess whether Mark is from Winnipeg or Toronto, you guess Toronto because more people live in Toronto. You have used _______ in making your decision. A) the availability heuristic B) base rate information C) the representativeness heuristic D) counterfactual thinking E) anchoring and adjustment

B) base rate information

A researcher interested in testing a(n) ________ hypothesis would be most likely to conduct an experiment. A) correlational B) causal C) descriptive D) theoretical E) observational

B) causal

Counterfactual thinking is A) unrealistic. B) conscious and effortful. C) voluntary and intentional. D) realistic. E) unconscious.

B) conscious and effortful.

Surveys are instruments most often used by social psychologists who conduct ________ research. A) experimental B) correlational C) descriptive D) observational E) archival

B) correlational

Researchers interested in determining how people's responses to one question can predict their other responses use the A) archival method. B) correlational method. C) descriptive method. D) experimental method. E) observational method.

B) correlational method.

Are people who are better educated more or less prejudiced than people who have less education? This question is best answered by A) archival analysis. B) correlational research. C) systematic observation. D) participant observation. E) structured interviews.

B) correlational research.

Feeling lonely and sorry for himself, Jerry moans, "If only I hadn't gotten drunk and made a fool of myself on my date with Mary, she and I would now be happily married." According to the text, Jerry is engaging in A) ironic processing. B) counterfactual thinking. C) delusions of adequacy. D) self-improvement. E) wishful thinking.

B) counterfactual thinking.

Getting trapped in one's thoughts about unpleasant events and an inability to move on from them is a risk of A) the accessibility heuristic. B) counterfactual thinking. C) the availability heuristic. D) the self-fulfilling prophecy. E) the representativeness heuristic.

B) counterfactual thinking.

A ________ is a description of the purpose of a study that is different from the true purpose. A) consent form B) cover story C) covert agenda D) construal E) debriefing

B) cover story

Latané and Darley (1968) told participants that they were interested in the kind of problems that college students experience. This explanation is the A) experimental debriefing. B) cover story. C) independent variable. D) dependent variable. E) operational definition.

B) cover story.

The two main goals of cross-cultural research are to ________ and to ________. A) increase internal validity; replicate questionable findings B) demonstrate human universals; explore cultural differences C) increase external validity; secure unbiased samples D) explore cultural differences; replicate questionable findings E) demonstrate human universals; replicate questionable findings

B) demonstrate human universals; explore cultural differences

It is important to know the probability level for a given set of experimental findings because p-values A) greater than .10 indicate that there is no need to replicate the experiment. B) inform experimenters whether their results might have happened by chance. C) alert experimenters to poor dependent variable measures. D) indicate that experimenters have used the correct manipulation of the independent variable. E) inform experimenters about the external validity of their research.

B) inform experimenters whether their results might have happened by chance.

When three observers record children's behaviour in a park, it is essential to establish A) internal validity. B) interjudge reliability. C) extrinsic reinforcement. D) demand characteristics. E) a theory.

B) interjudge reliability.

By controlling all extraneous variables and by randomly assigning people to different experimental conditions, an experimenter can achieve high A) interjudge reliability. B) internal validity. C) replicability. D) generalizability. E) p-values.

B) internal validity.

A ________ enables researchers to decide whether variations in findings across different studies are large enough that scientists should be skeptical about their conclusions. A) correlation coefficient B) meta-analysis C) validity index D) replication E) reliability coefficient

B) meta-analysis

Recall that Latané and Darley observed the number of participants in each experimental condition who left their cubicles to help the alleged victim of a seizure. The ________ was the dependent variable in their experiment. A) severity of the seizure B) number of participants who tried to intervene C) number of participants D) number of personal problems reported by participants E) number of other discussants

B) number of participants who tried to intervene

A researcher has recorded that on the playground, boys are more likely to use physical aggression to get what they want, but girls are more likely to use verbal aggression to get what they want. This researcher most likely employed a(n) ________ research method. A) experimental B) observational C) clinical D) interview E) archival

B) observational

In reading descriptions of the behaviours of the university students who participated in Latané and Darley's (1968) experiment on bystander intervention, you wonder whether your parents and their friends would behave the same way as those participants from the university. In essence, your question is, "Will these results generalize across ________?" A) situations B) people C) other experiments D) manipulations E) time

B) people

Experimenters who value external validity want their findings to generalize across ________ and ________. A) experiments; time B) people; situations C) people; experiments D) observers; situations E) time; situations

B) people; situations

The ________ is a number that expresses the likelihood that a given experimental finding would have occurred by chance alone. A) chaos index B) probability level C) correlation D) uncertainty quotient E) chance index

B) probability level

A credible cover story ensures A) mundane realism. B) psychological realism. C) appropriate dependent variable measures. D) internal validity. E) appropriate independent variable measures.

B) psychological realism.

When thinking about the film Casablanca, many people erroneously remember the Humphrey Bogart line, "Play it again, Sam." In reality, Bogart never said that line. This example illustrates that memory is A) objective. B) reconstructive. C) random. D) regularly false. E) inconsistent with our schemas.

B) reconstructive.

The _______ heuristic refers to the mental shortcut whereby people classify a person or thing based on how similar it appears to the typical case. A) base-rate B) representativeness C) anchoring and adjustment D) availability E) counterfactual

B) representativeness

In Chapter 2, the authors included a brief quiz about research findings. This quiz was designed to illustrate that A) most research findings directly contradict folk wisdom. B) so-called "obvious" research findings are not all that easy to predict in advance. C) although people are not insightful "physicists," they are insightful "social psychologists." D) social psychology is really little more than common sense. E) the wording of a quiz can easily be manipulated to trick the reader.

B) so-called "obvious" research findings are not all that easy to predict in advance.

When the stakes are high and accuracy counts, people are more likely to use _______ strategies and thus to make _______. A) simplistic; more accurate judgments B) sophisticated; more accurate judgments C) sophisticated; slower judgments D) motivated; less accurate judgments E) simplistic; less accurate judgments

B) sophisticated; more accurate judgments

Pierre encounters his physics professor working on her laptop computer in a coffee shop. The professor has a stack of physics journals, along with a rap music CD on the table. If asked later to recall what he had seen, Pierre would be most likely to remember ________ because ________. A) the rap music CD; it is consistent with his schema for a coffee shop B) the computer and journals; they are consistent with his schema for physics professors C) the rap music CD; it is consistent with his schema for physics professors D) the rap music CD; it is consistent with his schema of a student E) the computer and journals; they are inconsistent with his schema for physics professors

B) the computer and journals; they are consistent with his schema for physics professors

Social neuroscience examines A) social behaviour among neuroscientists. B) the correlation of social information processing and brain activity. C) how social behaviour affects brain development. D) the correlation of social action and motor activity in the brain. E) brain activity in social scientists.

B) the correlation of social information processing and brain activity.

The notion of "the basic dilemma of the social psychologist" refers to A) whether or not to conduct ethically questionable research. B) the trade-off between internal and external validity. C) the trade-off between correlational and experimental research. D) the trade-off between independent and dependent variables. E) whether or not to use deception in their experiments.

B) the trade-off between internal and external validity.

In a study by Hedden and colleagues (2008) examining brain activity, East Asian and European Americans underwent an fMRI while focusing on either a target or the context of a picture. In which circumstances did European Americans show greater attention? A) when asked to attend to the target B) when asked to attend to the context C) when asked to ignore the context D) when in a room with many other people E) when in a room alone

B) when asked to attend to the context

Based on survey data, the Literary Digest erroneously predicted that Alf Landon would beat Franklin D. Roosevelt in the upcoming presidential election. What cardinal rule of survey methods did the Literary Digest violate? Make sure A) respondents understand the survey. B) you have sampled randomly from the population. C) respondents are given more than one response option. D) responses are made anonymously. E) the survey questions are straightforward.

B) you have sampled randomly from the population.

Which of the following statements is true? A. Some people are controlled processors while others are automatic processors. B. Both automatic and controlled processing can lead to consequential errors. C. People generally use controlled processing unless a domain is familiar enough that they can use automatic processing. D. A basic assumption of psychology that it is impossible to ever gain control over automatic processes.

B. Both automatic and controlled processing can lead to consequential errors.

Reasoning based on the ease with which we can bring something to mind involves the use of the ________ heuristic. A. counterfactual B. availability C. confirmatory D. representativeness

B. availability

If you use the Internet, you are probably familiar with emoticons: facial expressions displayed using a combination of typographic figures, e.g., :-) for a smile. Asian countries have their own set of emoticons; in these (which are viewed without having to turn one's head), there are different smiling facial figures for a man smiling (^_^) and a woman smiling (^ . ^), since, in Japan, women are not supposed to exhibit a wide, uninhibited smile. This difference between the men's and women's emoticons in Japan is due to a(n) A. emblem. B. display rule. C. affect blend. D. decoding difference.

B. display rule.

Paula's cubicle at work is filled with pictures of her family, comfy stuffed animals, her favorite foods, and posters of her favorite music groups. Based on the research of Gosling, Paula is most likely a(n) A. narcissist. B. extravert. C. depressed person. D. foreigner.

B. extravert.

The mental shortcuts that people use to make judgments quickly and efficiently are called: A. schemas. B. heuristics. C. controlled processes. D. self-fulfilling prophecies.

B. heuristics.

Anna usually doesn't like movies with violent scenes. Still, she saw Sin City-a violent movie-five times, and loved it. Everyone else-including critics-really liked Sin City, too. In this example, distinctiveness is ________ and consensus is ________. A. low; high B. high; high C. low; low D. high; low

B. high; high

Schemas are most like which of the following? A. tires on which we move about in the world B. lenses through which we look at and learn about the world C. food which helps us stay energized to explore the world D. pillows which help us recover from our time in the world

B. lenses through which we look at and learn about the world

Which of the following is the best illustration of a self-fulfilling prophecy? A: Justin wanted some candy, and bought some at the store the next day. B: Ryan heard that Grace is opinionated; upon meeting her, he strikes up a conversation about politics and finds that she is willing to express her opinion. C: Emily always wanted to be a physician when she grew up and is now finishing medical school. D: Olivia has heard that Dylan is a good singer. When she sees him at a coffee shop, she asks him to sing yet finds his singing atrocious.

B: Ryan heard that Grace is opinionated; upon meeting her, he strikes up a conversation about politics and finds that she is willing to express her opinion.

Professor Chadwick is young and looks even younger than his years. He dresses in jeans, grubby tennis shoes, and Hawaiian print shirts. One day in the hallway, he strikes up a conversation with a student. During the conversation, the student begins to complain about the poor teaching of the faculty in the department and about the bad things he's heard about the course that Professor Chadwick teaches. Needless to say, when the student finds out he was actually talking to a professor, he feels embarrassed and avoids Professor Chadwick. What is the moral of this story? A: Students should be seen and not heard. B: Schemas may make us efficient, but sometimes at a cost in accuracy. C: The world would be a better place if people abandoned their schemas altogether. D: Heuristics make us more efficient, but only if we apply them to schemas.

B: Schemas may make us efficient, but sometimes at a cost in accuracy.

Which of the following is a consequence of underestimating the power of social influence? A: We perceive people as more inconsistent and variable than they really are. B: We tend to oversimplify complex situations. C: We tend to over-complicate simple situations. D: We overestimate our vulnerability to social situations.

B: We tend to oversimplify complex situations.

Christina returns home from her first year of college at UW - La Crosse, and is very proud of her first-year GPA on an A/B average. She's unhappily surprised to discover that her parents are disappointed that she didn't perform better. This difference in interpretation illustrates the power of ________ in explaining social behavior. A: interpersonal conflict B: construals C: socialization D: achievement motivation

B: construals

When Ashley first starts typing, she uses the "hunt and peck" method and finds typing very effortful. Now, after taking a typing class, she feels like her papers practically type themselves once she figures out what she wants to say. This change in her typing reflects a shift from A: emotional to cognitive thinking. B: controlled to automatic thinking. C: cognitive to emotional thinking. D: automatic to controlled thinking.

B: controlled to automatic thinking.

The tendency most people have to discount situational explanations of behavior in favor of personality characteristics or traits is called the: A: blame assignment bias. B: fundamental attribution error. C: character bias. D: discounting effect.

B: fundamental attribution error.

Complete the analogy: hindsight bias is to _________ as hypothesis is to _________. A: prediction; an educated guess B: retrospection; prediction C: common sense; logical analysis D: an explanation; common sense

B: retrospection; prediction

Researchers had participants memorize a number of words before reading an ambiguous passage about a character named Donald. The results of this study showed that people: A: who memorized any positive words about Donald, whether or not they applied to the passage, viewed him more positively. B: who memorized positive words about Donald viewed him positively only when those words were also applicable to the passage. C: who memorized negative words about Donald made a concerted attempt to overcome their prejudices and viewed him more positively. D: was not affected by the positivity or negativity of the words they memorized.

B: who memorized positive words about Donald viewed him positively only when those words were also applicable to the passage.

Social psychologists are interested in cross-cultural research because it: A: helps to understand the differences between societies. B:demonstrates which aspects of human behavior are universal. C: ultimately will lead to a better understanding of the causes of mental illness. D: provides information about individual differences.

B:demonstrates which aspects of human behavior are universal.

All of the following are examples of social influence EXCEPT: A: a bully imitates another child on the school yard B: you perceive the bathwater as hot when you first get in, but don't notice the heat ten minutes later. C: a child refrains from stealing ten dollars from his mother's purse when he imagines her anger at him. D:you cover your nose when you sneeze because you don't want to spread germs.

B:you perceive the bathwater as hot when you first get in, but don't notice the heat ten minutes later.

Consider the results from a study by Miyamoto and colleagues in which participants viewed either photos of U.S. city scenes or Japanese city scenes, then were asked to detect differences between two similar pictures. Based on their findings, if an American was touring in Tokyo, Japan, which of the following aspects of a picture would she be most likely to pick up on? a. foreground b. objects c. background d. contrasting colors

Background

In a 1932 study, a Bantu herdsman readily recalled the details of a cattle transaction, but the Scotsman had to look up his written record of the event. According to your text, what can best explain this difference in memory ability?

Bantu culture emphasizes cattle herding so strongly that herdsmen have well-developed schemas.

Isabella goes to a fortune teller who tells her, "You are the type of person who is very outgoing and sociable, yet at times you enjoy being alone." Isabella thinks this captures her personality amazingly well, because she finds many instances in her memories of such behaviors. Isabella has just fallen prey to the ___.

Barnum effect

You are listening to a radio broadcast which describes a person who donated a kidney to a complete stranger in need of a transplant, when the phone rings and the fireman's association asks you to make a donation. The idea of priming suggests that you will ___.

Be more likely to make a donation because the schema of being charitable has been made more accessible

Amani raises her hand in class. Her answer is wrong, and the teacher scolds her. After that, Amani doesn't participate much in class. This situation is most compatible with a ___ approach to understanding and predicting behavior.

Behaviorist

Which of the following is a criticism of the behaviorist approach?

Behaviorist explanations are too simplistic to explain all human social behavior.

Larry believes that Chris contributed to getting raped by wearing a skirt and tank top to a party. He also believes that a battered woman in his hometown is probably responsible for the abuse. This way of thinking represents Larry's

Belief in a just world

Recall that Hoffman, Lau, and Johnson (1986) provided native English speakers and Chinese-English bilinguals with descriptions of people who fit either an "artistic" type or a "shi gú" type. Their findings revealed that ___.

Bilingual speakers' impressions were determined by the language in which they read the story.

In a study of frustration and aggression, some participants were exposed to an accomplice who insulted them, and others were exposed to no such insult. Participants were then allowed to recommend whether the accomplice should be fired. Those who were insulted were more likely to retaliate by recommending that the accomplice lose his job. In this experiment, the ________ was the dependent variable. a. presence or absence of an insult b. accomplice c. participants' recommendations d. difference between the groups

C

In a study of group dynamics, participants were placed in groups consisting of either three or ten people. During the study, group members worked together trying to solve a puzzle. After completing the task, participants reported how satisfied they were with the other members of their group. ________ is the independent variable in this study. a. Group dynamics b. The puzzle c. Group size d. Satisfaction

C

In the 2000 presidential election, the polls of upper-middle-class Americans tended to show Bush leading Gore by a landslide. When election night came, the final results were closer than had been predicted, and it turned out that Gore won the popular vote while Bush won the more critical Electoral College vote. One possible problem with the earlier survey results was that they relied primarily on polls of upper-middle-class people. This illustrates the finding that a major problem with surveys is a. responses are not anonymous. b. people's votes in the polling booth do not necessarily agree with their public statements. c. they are not predictive of behavior if the sample is not representative of the population about which the prediction is to be made. d. question order can affect survey results.

C

Latané and Darley (1968) told participants that they were interested in the kind of problems that college students experience. This explanation to their participants was the a. independent variable. b. dependent variable. c. cover story. d. experimental debriefing.

C

Recall that Rosenthal and Jacobson (1968) informed grade-school teachers that some of their students (called "bloomers") would show great academic improvement in the upcoming year. Also recall that students labeled "bloomers" actually showed greater increases on IQ tests compared to students who were not labeled "bloomers." These differences occurred because teachers a. decided to devote their attention to the students with the most promise. b. spent significantly more time interacting with "bloomers." c. challenged "bloomers" more and gave them more and better feedback. d. distracted "non-bloomers" by providing them too much personal attention.

C

Research that examines the effects of deception experiments on participants has found that a. most people find such experiences extremely upsetting. b. participating in such experiments causes people to develop an enduring distrust of researchers. c. people do not object to the mild discomfort and deceptions used in social psychological research. d. participating in such experiments has a delayed negative effect.

C

Researchers asked some participants to think of six times they had behaved assertively and asked other participants to think of twelve times they had behaved assertively, and then asked all participants to rate how assertive they thought they really were. Participants who tried to generate six examples of assertive behaviors rated themselves as more assertive than did participants who tried to generate twelve examples. Why? Schwarz suggests that participants a. who could generate twelve examples were really more assertive, or else they couldn't come up with the examples. b. asked to think of twelve examples didn't have time to complete the experimental task. c. asked to generate six examples could do so more easily than participants who were asked to generate twelve. d. who were asked to generate six examples were more likely to use the representativeness heuristic.

C

What is the function of an institutional review board (IRB)? a. to peer-review results from experiments and suggest appropriate replications b. to perform a meta-analysis on all studies about a given phenomenon c. to review proposals for research and decide whether they meet ethical guidelines d. to assess the psychological realism of proposed psychological research

C

Your roommate is interested in dating Chris, whom you know from a history class. Your roommate asks if you think Chris is generally a considerate person. You remember how Chris lent you notes when you missed class last week, and tell your roommate you think Chris is very considerate. In making your judgment, you have relied upon the ________ heuristic. a. representativeness b. base rate c. availability d. counterfactual thinking

C

Psychologists distinguish between two types of cognitive processing. _______ processing is nonconscious and unintentional, whereas _______ processing is conscious and intentional. A) Inefficient; efficient B) Controlled; automatic C) Automatic; controlled D) Accurate; inaccurate E) efficient; inefficient

C) Automatic; controlled

At an outdoor concert, Julian automatically applies his "pot-smoker" schema when he sees a young man with long hair, sandals, and a Grateful Dead T-shirt. He offers the man a joint, and is arrested by an undercover narcotics officer. From a social- psychological perspective, what is the moral to this story? A) It is very difficult to ignore atypical information completely. B) It's not advisable to use schemas in interpreting the social world. C) Efficient information processing often comes at a cost in accuracy. D) Never trust anyone wearing a Grateful Dead T-shirt. E) In everyday life, most people want their experiences to be atypical.

C) Efficient information processing often comes at a cost in accuracy.

Which of the following threatens the internal validity of an experiment? A) Using an independent variable with more than one level. B) Failing to generalize the results outside the lab. C) Failing to assign participants randomly to conditions. D) Failing to use a random sample. E) Using more than one dependent variable.

C) Failing to assign participants randomly to conditions.

Why are social psychologists concerned with the issue of interjudge reliability? A) Interjudge reliability makes causal explanations possible in archival research. B) Interjudge reliability helps researchers determine relationships between variables. C) Independent agreement reduces the possibility of bias or distortion. D) Without it, there is no hope of reforming the legal system. E) Coding criteria must be objective and determined before observation begins.

C) Independent agreement reduces the possibility of bias or distortion.

________ refers to the extent to which an experimental situation resembles real-life situations. A) Situational realism B) Internal validity C) Mundane realism D) Psychological realism E) Operational realism

C) Mundane realism

Which of the following is an ethical concern in social psychological research? A) It is often difficult to debrief participants in a study. B) Undoing the effects of a plausible cover story can be difficult. C) Participants are sometimes exposed to upsetting situations. D) Participants must sometimes be discouraged from withdrawing prematurely from the experiment. E) People do not have a choice about the experimental condition to which they are assigned.

C) Participants are sometimes exposed to upsetting situations.

You've rented the movie Sophie's Choice. There is a scene in which Sophie, a recent immigrant from Poland, is both surprised and amused that Americans have so many words for the concept "fast." She reports that in Polish, there is only one word. You've just read Chapter 3 (Social Cognition), and a friend asks you why the cultures differ in that regard. What would you say? A) Americans are generally better educated than Poles. B) Polish has different roots than does English. C) Poles don't have well-developed schemas for the concept "fast." D) English has borrowed more words from foreign languages than Polish has. E) Poles have a relatively limited vocabulary compared to Americans.

C) Poles don't have well-developed schemas for the concept "fast."

According to the text, which of the following statements about judgmental heuristics is false? A) They are examples of automatic rather than controlled processing. B) They are usually adaptive because they help us make decisions when dealing with large amounts of ambiguous information. C) They usually lead us to make faulty decisions. D) They may be most likely to show up in experiments when researchers don't follow rules regarding conversations. E) They allow us to make judgments quickly and efficiently.

C) They usually lead us to make faulty decisions.

What is one of the main problems faced by researchers who want to test their theories cross-culturally? A) It is difficult to find funding to study culture. B) Basic psychological processes inevitably differ from culture to culture. C) Variables may have to be changed so that their meaning is understood in the same way across cultures. D) Ethical laws often prohibit psychologists from studying phenomena in all cultures. E) Because experimental social psychology is historically an American enterprise, it is difficult to secure funding for cross-cultural research.

C) Variables may have to be changed so that their meaning is understood in the same way across cultures.

Why do we use schemas? A) Schemas are taught to us in our early childhood. B) Schemas enable us to interpret the world accurately. C) Without schemas, the world would seem inexplicable and confusing. D) Humans are born with schemas. E) Without schemas, we could not be accurate in our judgments.

C) Without schemas, the world would seem inexplicable and confusing.

Which of the following is NOT a step in the self-fulfilling prophecy? A) Your behaviour toward someone influences their behaviour toward you. B) The person you have an expectation about actually behaves in a manner that confirms your expectation. C) Your expectation of someone influences your interpretation of their behaviour. D) You ignore information that is not consistent with your expectations. E) Your expectation of someone influences how you behave toward them.

C) Your expectation of someone influences your interpretation of their behaviour.

Assume that instead of conducting experiments, Latané and Darley had used a correlational method to study the relation between the number of bystanders who witness an emergency and how quickly a victim receives help. Assume that the correlational data were compatible with results from experiments: the more bystanders, the longer it took bystanders to help. What type of correlation is this? A) a nonlinear correlation B) a zero correlation C) a positive correlation D) a spurious correlation E) a negative correlation

C) a positive correlation

Korsakov's syndrome is a neurological disorder that makes sufferers unable to form new memories. The world is disorienting and incoherent to patients with this disorder, so they often confabulate or invent fictions to make the world less scary and confusing. A social psychologist might say that these patients A) are not concerned with reducing ambiguity. B) invent schemas where none exist. C) are cursed with memory that is always reconstructive. D) have lost their schemas. E) cannot form schemas.

C) are cursed with memory that is always reconstructive.

Which of the following is the best example of participant observation? A researcher A) videotapes the kinds of complaints that shoppers make at a customer service desk. B) goes through medical records to investigate physician's errors. C) attends a meeting of Alcoholics Anonymous to observe how people respond to excuses. D) uses automobile insurance records to record how drivers explain their accidents. E) is rude to some people in line at a movie theater and polite to others, and observes how they react.

C) attends a meeting of Alcoholics Anonymous to observe how people respond to excuses.

Research on the impact of media representations of male and female body ideals finds that A) the male muscular ideal shown in fitness magazines has little impact on males' assessment of their own bodies. B) exposure to media representations of the thin female body type has most impact on females who show little interest in their own body image. C) both genders can suffer emotionally or physically if they are exposed to, and accept, media body ideals. D) the only males who respond to muscular ideals in fitness magazines are those who have low self-esteem. E) exposure to body ideals in the media has greater impact on male ratings of their own body image than on female ratings of their own body image.

C) both genders can suffer emotionally or physically if they are exposed to, and accept, media body ideals.

According to information from the registrar's office, Lee has discovered that people who achieve higher grades in their last year of high school tend to have higher GPAs in their first year of university. Lee has used a(n) ________ research method. A) observational B) field study C) correlational D) experimental E) participant observation

C) correlational

Of the heuristics below, the use of _______ is most likely to have negative emotional consequences. A) the representativeness heuristic B) the availability heuristic C) counterfactual thinking D) the base rate fallacy E) anchoring and adjustment heuristic

C) counterfactual thinking

Recall that Christopher Davis and his colleagues (1995) interviewed people who had suffered the loss of a spouse or child. They found that the more ____________, the more distress the survivors reported. A) recent B) unpreventable the deaths were perceived to be C) easily they could imagine how the death could have been prevented D) vivid the survivors' memories of the death E) violent the cause of death

C) easily they could imagine how the death could have been prevented

"Give me a break!" you say to yourself as you read the description of Latané and Darley's (1968) experiment, "How often in the real world do people on intercoms hear a seizure unfold? How similar is that situation to real life?" You are skeptical about the ________ validity of their experiment. A) construct B) criterion C) external D) internal E) conclusion

C) external

In a study by Hedden and colleagues (2008) examining brain activity, East Asian and European Americans underwent an fMRI while focusing on either a target or the context of a picture. When asked to focus on the context of a picture, European Americans' ________ regions of the brain were more active. A) occipital B) limbic C) frontal and parietal D) temporal E) lateral

C) frontal and parietal

Chapter 3 began with a description of accident victim Kevin Chappell who suffered brain damage that left him with a severe visual disability. His disability is such that A) he is blind due to damage to the optic nerve. B) he can recognize environmental context but not the objects or people in the scene. C) he can recognize faces but not things. D) he can recognize objects, but cannot read. E) he can recognize things but not faces.

C) he can recognize faces but not things.

Which of the following refers to the level of agreement between two or more people who independently observe and code the same information? A) archival analysis B) external validity C) interjudge reliability D) archival validity E) coding validity

C) interjudge reliability

Consider the following (fictitious) conclusion reached by a social psychologist: "After analyzing a sample of over 578 experiments, we have concluded that the question of whether men are more likely than women to help is difficult to answer. Based on the findings of this large number of studies, we have concluded that men are more likely to help when helping demands physical risk or 'chivalry;' otherwise, there are no reliable gender differences in helping." This conclusion about the reliability of the relation between gender and helping was most likely based on A) pure conjecture. B) surveys of researchers. C) meta-analytic techniques. D) correlational research. E) experimental procedures.

C) meta-analytic techniques.

It is _______ to miss a bus by five minutes than to miss it by an hour because _______. A) more distressing; images of the people waiting for you are more available in memory B) less distressing; you feel good that you almost made it C) more distressing; it's easier to imagine what you might have done to catch the bus D) less distressing; it's easier to imagine what you might have done to catch the bus E) less distressing; you don't have to wait as long for the next bus

C) more distressing; it's easier to imagine what you might have done to catch the bus

Suppose that I conduct an experiment on the effects of flattery on compliance. I bring women into the laboratory, tell them they are beautiful, and ask them to lend me $10. I bring men into the laboratory, I don't flatter them, and I ask them to lend me $10. Sure enough, more women than men lend me $10. My conclusion that flattery increases compliance is suspect because A) I failed to randomly select the sample from the population. B) my experiment lacks external validity. C) my experiment lacks internal validity. D) I have not operationally defined my variables. E) my experiment lacks generalizability.

C) my experiment lacks internal validity.

Television programs such as Survivor and Big Brother, in which television cameras record the activities of people as they interact, are most like ________ research in social psychology. A) cross-cultural B) experimental C) observational D) archival E) correlational

C) observational

What makes the observations conducted by social scientists different from the kinds of observations that anyone might make in the course of a day? Social scientists A) will only sample people from their own culture. B) make sure to observe a random sample of people. C) observe and code behaviours according to prearranged criteria. D) rely on technology (e.g., hidden cameras or tape recorders) to record behaviours. E) make it a point never to interact with the people they are observing.

C) observe and code behaviours according to prearranged criteria.

The importance of physical environment in triggering automatic thinking about racist stereotypes was shown in a study that found A) participants in a bright room responded with more positive evaluations of their own race compared to participants in a darkened room. B) participants who were in an open field gave more aggression responses to faces of black men, while those in a closed booth gave more flight responses. C) participants in a darkened room had faster access to negative stereotypes of black people compared to participants seated in a bright room. D) participants were more likely to "shoot" at a picture showing a white person than they were one showing a black person. E) tools were more likely to be misidentified as guns if a preceding photograph was of a white face.

C) participants in a darkened room had faster access to negative stereotypes of black people compared to participants seated in a bright room.

According to the authors, there are two ways that traits about others can become accessible. These are A) automatic activity and past experience. B) priming and attitude retrieval. C) past experience and priming. D) attitude retrieval and automatic activity. E) priming and attitude retrieval.

C) past experience and priming.

Sometimes traits are chronically accessible due to past experience. Other times, traits are accessible for more arbitrary reasons, such as what we're currently thinking. Social psychologists use the term ________ to describe the latter case. A) attributions B) implicit personality theories C) priming D) social retrieval E) accessibility

C) priming

The major difference between experiments and other research methods is that experiments involve A) mundane realism and psychological realism. B) both internal and external validity. C) random assignment to conditions and manipulation of the independent variable. D) the use of statistical analysis. E) random assignment and psychological realism.

C) random assignment to conditions and manipulation of the independent variable.

According to the research by John Bargh and his colleagues (2001), your professor might read the following words prior to an exam if she wants her students to do well on the exam: A) coward, timid, fear. B) sloppy, messy, wasteful. C) remember, integrate, comprehend. D) quiet, concentrate, obey. E) hockey, tennis, cricket.

C) remember, integrate, comprehend.

When people classify a stimulus based on how similar it appears to the typical case, they are using the _______ heuristic. A) correspondence B) availability C) representativeness D) exemplar E) hindsight

C) representativeness

Basic research is to ________ as applied research is to ________. A) correlation; causation B) experiments; surveys C) satisfying intellectual curiosity; solving social problems D) biological sciences; social sciences E) experiments; observational research

C) satisfying intellectual curiosity; solving social problems

We can conclude from the studies comparing motivated judgments to automatic processing that people who are A) motivated to produce judgments are more anxious and tense and this biases their judgments. B) not motivated to produce judgments tend to be proficient in their thinking and their judgments are less biased due to low stress. C) strongly motivated to produce unbiased judgments tend to do so. D) strongly motivated to produce judgments tend to produce biased ones due to the availability effect. E) strongly motivated to produce judgments tend to produce biased ones due to the perseverance effect.

C) strongly motivated to produce unbiased judgments tend to do so.

Recall that Harold Kelley (1950) told some college students that their guest lecturer was a warm person, and others that he was a cold person. The visitor then lectured for 20 minutes, and the students later evaluated him. Assume for the moment that students' evaluations and behaviours didn't differ in the two conditions. These findings would suggest that A) the contents of the lecture contradicted the warm versus cold descriptions. B) the guest lecture was the same in both conditions. C) students didn't have schemas for "warm" and "cold" personalities. D) schemas don't exist. E) the students did not pay attention to the lecturer.

C) students didn't have schemas for "warm" and "cold" personalities.

When confronted with "H H H H T T T T T T" as a summary of the results of a series of 10 flips of a coin, most people will view that series as "less random" than a summary of results that looks like "H H T H T T H T H T," even though the odds of a "head" on the next toss is 50/50 in each case. Why? Their estimates have been biased by A) counterfactual thinking. B) the availability heuristic. C) the representativeness heuristic. D) downward counterfactual thinking. E) anchoring and insufficient adjustment.

C) the representativeness heuristic.

In trying to understand new situations, we A) prefer to come up with brand new schemas to specifically fit the situation. B) tend to avoid the use of heuristics. C) use judgmental heuristics to make decisions quickly and efficiently. D) attempt to apply old schemas, which often lead to faulty decisions. E) exhaustively search our memories for all relevant information in order to ensure the correct decision.

C) use judgmental heuristics to make decisions quickly and efficiently.

Recall that Harold Kelley (1950) told some students that a guest instructor was warm, and others that a guest instructor was cold. When the instructor behaved in an arrogant way, he was rated as immodest, no matter what students were told about him at the outset. These findings suggest that A) schemas are specific to the situation in which we find ourselves. B) when we dislike someone, we don't give that person the benefit of the doubt. C) when behaviours are decidedly unambiguous, we rely less on our schemas. D) in tense situations, we seldom rely on our schemas. E) schemas are completely malleable.

C) when behaviours are decidedly unambiguous, we rely less on our schemas.

Conceptually, what distinguishes laboratory from field experiments is A) whether there is a dependent variable. B) whether there is an independent variable. C) where the experiment is conducted. D) the number of participants. E) whether there is random assignment.

C) where the experiment is conducted.

Even if an observer has information about an actor's situation available, why might the observer still be likely to commit the fundamental attribution error? A. People rely too much on the base rate heuristic. B. People tend to rely on stereotypes. C. It is difficult to know how an actor interprets a situation. D. People tend to rely on the availability and adjustment heuristic.

C. It is difficult to know how an actor interprets a situation.

Which of the following is not true about the social psychological approach? A. A person's need to make accurate judgments of the world often determines how he or she views a situation. B. You can only determine causality using the experimental method. C. To understand the power of social influence, you have to understand the nature of the objective situation, not how people perceive it. D. A person's need to maintain self-esteem often determines how he or she views a situation.

C. To understand the power of social influence, you have to understand the nature of the objective situation, not how people perceive it.

Ming is from China; Tyrone is from Canada. Both participate in an experiment in which they take a test, are given feedback and told that they did very well, and then are asked to make attributions for their performance. Based on cross-cultural research on the self-serving bias, you would expect that A. both Ming and Tyrone will say that they succeeded due to their high ability. B. Ming, but not Tyrone, will say that he succeeded due to his high ability. C. Tyrone, but not Ming, will say he succeeded due to his high ability. D. both Ming and Tyrone will say that they succeeded because the test was easy.

C. Tyrone, but not Ming, will say he succeeded due to his high ability.

Based on the study by Nisbett and his colleagues on statistical reasoning across four disciplines, which person would you count on to solve a statistical reasoning problem accurately? A. a physician B. a lawyer C. a psychologist D. a chemist

C. a psychologist

The tendency to think that other people are more susceptible to attributional biases in their thinking than we are is a(n) A. defensive attribution. B. availability heuristic. C. bias blind spot. D. schema.

C. bias blind spot.

In honor of Rosa Parks' contribution to the civil rights movement, the seat behind the driver on buses in major cities was reserved for one day, and people were asked not to sit there. Some people did actually sit in this seat. Making the assumption that these people were prejudiced or racist is an example of the A. use of heuristics. B. accuracy of internal attributions. C. fundamental attribution error. D. covariation effect.

C. fundamental attribution error.

Korsakov's syndrome is a neurological disorder that makes sufferers unable to form new memories. The world is disorienting and incoherent to patients with this disorder, so they often confabulate and invent fictions to make the world less scary and confusing. A social psychologist might say that these patients: A. have no schemas. B. are not concerned with reducing ambiguity. C. invent schemas where none exist. D. can only use controlled but not automatic processes.

C. invent schemas where none exist.

Research demonstrated that physical sensation can activate metaphors that influence our judgments about unrelated topics. If you wanted students to take your petition seriously, you should present the information about it: A. on a light clipboard. B. verbally rather than written. C. on a heavy tablet. D. on a piece of paper they can take with them.

C. on a heavy tablet.

Given the choice between distorting the world in order to enhance their self-esteem or viewing the world accurately, people often A. refuse to make a choice, behaving erratically. B. completely distort reality. C. put a slightly different spin on things in order to feel good about themselves. D. choose accuracy and thereby suffer from low self-esteem.

C. put a slightly different spin on things in order to feel good about themselves.

In order to examine the prevalence of drug use in several different generations of Americans, a researcher decides to collect the lyrics from the fifty most popular songs from each decade, starting in the 1940s and spanning to 2010, and then code those lyrics for how often drug-related themes were present. Which of the following methods is this researcher using? A: correlational B: observational C: archival D: cross-sectional

C: archival

Lee Ross describes the idea of "naïve realism" as the: A: scientific study of the way in which people's thoughts, feelings, and behaviors are influenced by the real or imagined presence of other people. B: way in which people perceive, comprehend, and interpret the social world. C: conviction that all of us have that we perceive things "as they really are." D: aspects of people's personalities that make them different from other people.

C: conviction that all of us have that we perceive things "as they really are."

Consider the following survey item: "If you found yourself on an airplane with engine problems, would you ________?" Most social psychologists would not include such an item on their survey because most respondents would A: never have experienced that situation, yielding too small a sample. B: be offended at such a personal question. C: find it difficult to accurately imagine what they would actually do. D: avoid flying in the future, instead opting for trains.

C: find it difficult to accurately imagine what they would actually do.

We can form impressions of people we interact with rather quickly and effortlessly. We do this by engaging in automatic analysis of our environments, based on A: biological signals we receive from other people. B: rewards given to us by our friends. C: our past experiences and knowledge of the world. D: information provided by our parents.

C: our past experiences and knowledge of the world.

After the mass suicides related to the cults at Jonestown, people tended to blame the victims and accuse them of being psychologically unstable or deranged. Social psychologists are more likely to explain these mass suicides as being due to: A: individual differences, such as antisocial personality. B: the imagined presence of an all-powerful deity. C: he social influence of cult leaders. D: mental illness in most of the cult members.

C: the social influence of cult leaders.

If you grew up in a household with a brother who was morbidly shy, you might have a different belief about a man you meet who avoids dating women than a person who grew up in a household with a brother who was gay. This difference in interpretation is due to differences in your and the other person's ___.

Chronic accessibility of gay vs. shy schemas.

Sheila fancies herself an outgoing extrovert. She smiles a lot and gestures dramatically as she talks. This example best illustrates the use of nonverbal behavior to ___.

Communicate personality traits

Research on people's use of the covariation model in making attributions suggests that people are less likely to use ________ information than Kelley's model predicts.

Consensus

Latané and Darley were interested in examining the reasons for the lack of bystander intervention in Kitty Genovese's murder. The ________ of what was happening caused people to perceive, comprehend, and interpret the situation differently; thereby, delaying helping.

Construal

Karen returns home from her first year of college, and is very proud of her first-year GPA. She earned a 3.0 ("B") average. She's unhappily surprised to discover that her parents are disappointed that she didn't perform better. This difference in interpretation illustrates the power of ___ in explaining social behavior.

Construals

A researcher is interested in the effects of self-esteem on people's choice of romantic partner. She asks her study participants to complete a measure of self-esteem and to bring in separate photos of themselves and their significant others. She then has these photos rated for attractiveness by a panel of independent judges, and relates the relative attractiveness of a person's significant other to the person's self-esteem. The study just described is a(n) ___.

Correlational study

Karen is so used to driving to her boyfriend's house that when she is taking her little brother to softball practice early one morning, before she is fully awake, she finds herself driving to her boyfriend's house instead. This is an example of a. judgmental heuristics. b. counterfactual thinking. c. controlled thinking. d. automatic thinking.

D

Which mental shortcut does the text suggest causes the Barnum effect? A) the judgmental heuristic B) the availability heuristic C) the base rate information D) the representativeness heuristic E) the cocktail party phenomenon

D) the representativeness heuristic

Melissa, a second-grade student, just finished going over her spelling list for the week. The list contained many adjectives that reflected positive emotions (e.g., happy, elated, delighted). As Melissa gazes out the window, she sees her sister Karin come running up to the house. If Melissa is like many of the participants in an experiment by Tory Higgins and his colleagues (1977), and if Melissa's construal is a function of priming, what is she most likely to think? A) "Mom told Karin not to run." B) "The neighbours must think Karin's crazy." C) "Why is Karin so scared?" D) "Karin must have something great to tell us." E) "I hope Karin is okay."

D) "Karin must have something great to tell us."

What technique will most efficiently determine the generalizability of social- psychological findings conducted in different situations and with different people? A) A series of replications. B) A conceptual review of the literature. C) A case study analysis. D) A meta-analysis. E) Multiple experiments that use random sampling.

D) A meta-analysis.

Which of the following is NOT an example of an operational definition? A) Defining "liking" as the number of times two people smile at each other. B) Defining "liking" as the number of times people get together in one week. C) Defining "aggression" as the number of times a child yells at a peer. D) Defining "love" as a unique and special feeling. E) Defining "aggression" as physically attacking another person.

D) Defining "love" as a unique and special feeling.

________ validity is the extent to which results of a study can be generalized to other situations or other people. A) Causal B) Statistical conclusion C) Internal D) External E) Sampling

D) External

In many respects, Allen Funt's old television show, Candid Camera, is similar to observational research in social psychology. In what crucial respect is Candid Camera most different from observational research? A) The people Funt filmed did not provide informed consent. B) The kind of situations that Funt filmed are not relevant to social psychologists. C) Funt intentionally manipulated the situations. D) Funt's observations were not conducted in a controlled, scientific manner. E) Funt did not use a random sample of people who confronted strange situations.

D) Funt's observations were not conducted in a controlled, scientific manner.

A major difference between a field experiment and a laboratory experiment is that A) Mundane realism is higher in a field experiment. B) The levels of the independent variable cannot be controlled by the experimenter. C) The results on the dependent variable are less predictable due to the variable behaviours of participants in a real-life setting. D) Participants are unaware of their involvement in a psychology experiment. E) Psychological realism is lower in a field experiment.

D) Participants are unaware of their involvement in a psychology experiment.

Recall that Norbert Schwarz and his colleagues (1991) asked some participants to think of 6 times they had behaved assertively and asked other participants to think of 12 times they had behaved assertively, and then asked all participants to rate how assertive they thought they really were. Participants who tried to generate 6 examples of assertive behaviours rated themselves as more assertive than did participants who tried to generate 12 examples. Why? A) Participants asked to think of 12 examples didn't have time to complete the experimental task. B) Participants who could generate 12 examples were really more assertive, or else they couldn't come up with the examples. C) Participants who were asked to generate 6 examples were more likely to use the representativeness heuristic. D) Participants asked to generate 6 examples could do so more easily than participants who were asked to generate 12. E) Participants asked to generate 12 examples could do so just as easily than participants who were asked to generate 6.

D) Participants asked to generate 6 examples could do so more easily than participants who were asked to generate 12.

________ allows researchers to rule out differences among participants as the cause of differences in the dependent variable. A) A factorial design B) Random sampling C) Measuring more than one dependent variable D) Random assignment to condition E) Matching

D) Random assignment to condition

A researcher is interested in the changing nature of sex roles in contemporary society. If she were to employ an archival analysis, what would she be most likely to do? A) Participate in the daily activities of a family in which the woman works and the man stays home. B) Observe both men and women in "non-traditional" occupations. C) Interview both male and female doctors to determine how they are treated by colleagues. D) Record how boys and girls are portrayed in children's books. E) Randomly assign people to "non-traditional" roles and record their behaviour.

D) Record how boys and girls are portrayed in children's books.

Which of the following statements about schemas is true? A) Schemas affect the information we notice, but only when we are aware of the schema. B) Schemas affect what we perceive while emotional processes are important for determining what we think about and remember. C) Schemas affect what we notice but not necessarily what we remember. D) Schemas affect the information we notice, think about, and remember. E) Schemas affect what we remember while intensity of stimuli determines what we notice.

D) Schemas affect the information we notice, think about, and remember.

Professionals like actors, writers, and filmmakers employ observational methods to learn about social situations. What makes their work different from the work of social psychologists? A) Social psychologists are more likely to be participant observers. B) These professionals seldom set out to answer a specific question. C) The situations or events that these professionals observe are not of interest to scientific social psychologists. D) Social psychologists tend to employ a pre-arranged set of criteria to guide their observations. E) These professionals are more interested in individual personality differences than a social psychologist would be.

D) Social psychologists tend to employ a pre-arranged set of criteria to guide their observations.

Why is it unwise to conclude that if two variables are correlated, one must have caused the other? A) Rarely are only two variables correlated. B) It is impossible to conclude that two variables are related unless one can measure them perfectly. C) Variables can never be measured with complete accuracy. D) Some unmeasured third variable might make them appear related when in fact they are not. E) Any single variable is bound to have multiple causes.

D) Some unmeasured third variable might make them appear related when in fact they are not.

Which of the following is a strong advantage of surveys over other research methods? A) Surveys can determine whether variables are directly or inversely related. B) Surveys are higher in mundane realism. C) Surveys help to establish the causal connections of phenomena evident in everyday life. D) Surveys allow researchers to determine the relationship between variables that are difficult to observe. E) Surveys are higher in external validity.

D) Surveys allow researchers to determine the relationship between variables that are difficult to observe.

What is the major advantage of processing information automatically? A) Automatic processing is more easily remembered than controlled processing. B) Automatic processing is more easily modified than controlled processing. C) It is easier to learn how to process information automatically. D) We can use our cognitive resources for other more important matters. E) Automatic processing yields more accurate judgments than does controlled processing.

D) We can use our cognitive resources for other more important matters.

Why do people often fall prey to the availability heuristic and draw incorrect inferences? A) What looks like the typical case may not be representative of the typical case. B) People are far better at making inferences about themselves than about others. C) People wrongly assume that correlation implies causation. D) What is easily brought to mind may not be typical of the big picture. E) People often ignore base rates.

D) What is easily brought to mind may not be typical of the big picture.

A survey would be a good investigative tool for a social psychologist to answer which of the following questions? A) How willing are people to help someone needing medical assistance in a shopping mall? B) Do people take longer to pull out of a parking spot if they notice another car is waiting? C) Do people under increased time pressure give less generous donations? D) What is the relationship between people's assessment of the risks of West Nile virus and their behaviour in preventing it? E) How willing are people to donate money to a well-known charity for disaster relief?

D) What is the relationship between people's assessment of the risks of West Nile virus and their behaviour in preventing it?

The experimental method always involves A) psychological realism. B) interrater reliability. C) mundane realism. D) a direct intervention on the part of the researcher. E) one experimental group and one control group.

D) a direct intervention on the part of the researcher.

The self-fulfilling prophecy occurs because we often A) work to revise our schemas when necessary. B) distort reality to support our schemas. C) abandon our schemas when we're under pressure. D) act based on our schemas. E) ignore base-rate information.

D) act based on our schemas.

After seeing the movie Fargo, Craig firmly believes that Minnesotans are the goofiest people in the United States. You believe that Minnesotans are no goofier than people any place else, and you would like to get Craig to change his judgment. To do this, you might A) tell him to stop being irrational. B) ask him to watch the film again. C) tell him that he needs a law school course. D) ask him to consider the opposite point of view. E) ask him to think more carefully about his judgment.

D) ask him to consider the opposite point of view.

Which of the following pairs of variables is most likely to be negatively correlated? A) intelligence; exam scores B) amount of practice; quality of performance C) education; income D) calories consumed; weight loss E) effort; success

D) calories consumed; weight loss

Recall that Robert Rosenthal and Lenore Jacobson (1968) informed grade school teachers that some of their students (called "bloomers") would show great academic improvement in the upcoming year. Also recall that students labeled "bloomers" actually showed greater increases on IQ tests, compared to students who were not labeled "bloomers." These differences occurred because teachers A) distracted "non-bloomers" by providing them too much personal attention. B) spent significantly more time interacting with "bloomers." C) decided to devote their attention to the students with the most promise. D) challenged "bloomers" more and gave them more and better feedback. E) ignored "non-bloomers."

D) challenged "bloomers" more and gave them more and better feedback.

When Frederic Bartlett (1932) interviewed both a Scotsman and a Bantu herdsman about the particulars of a cattle transaction, the Scot _______, whereas the Bantu herdsman _______. A) guessed; said that he couldn't remember B) remembered the faces of the traders; remembered the colour of the cattle C) erroneously recalled that sheep were traded; accurately recalled that oxen and cattle were traded D) consulted his notes; recalled details from memory E) guessed; remembered the colour of the cattle

D) consulted his notes; recalled details from memory

Holistic thinking is to ________ as analytic thinking is to ________. A) automatic; controlled B) priming; heuristics C) linear; circular D) context; object E) inefficient; efficient

D) context; object

Observational research is to describing a behaviour, as correlational research is to A) determining the causes of behaviour. B) manipulating a behaviour. C) explaining a behaviour. D) documenting relations between behaviours. E) understanding a behaviour.

D) documenting relations between behaviours.

Out of curiosity, you wonder whether some coworkers in your office are more likely than others to use profanity. During the day, each and every time a coworker curses, you write down his or her name, and the words he or she said. Your informal research is most like the ________ research conducted by social psychologists. A) archival B) survey C) correlational D) ethnographic E) experimental

D) ethnographic

A psychologist first conducted a study on helping behaviour with a sample of college undergraduates. This psychologist then replicated the study using the same procedure with a different participant sample. If similar results were obtained with both samples, this experimenter has demonstrated A) internal validity. B) population consistency. C) chronological consistency. D) external validity. E) a significant difference.

D) external validity.

If I tell you that someone I know is a man who has a beard, drinks wine, and likes to read poetry, and you guess that he is more likely to be a classics professor than a construction worker, you are A) paying too much attention to base rate information. B) being misled by the availability heuristic. C) probably correct. D) falling victim to the representativeness heuristic. E) probably not familiar with classics professors.

D) falling victim to the representativeness heuristic.

Consider the following survey item: "If you found yourself on an airplane with engine problems, would you...?" Most social psychologists would not include such an item on their survey because most respondents would A) be offended at such a personal question. B) avoid flying in the future, instead opting for trains. C) never have experienced that situation, yielding too small a sample. D) find it difficult to imagine what they would actually do. E) not understand the question.

D) find it difficult to imagine what they would actually do.

In a study by Hedden and Colleagues (2008) East Asian Americans showed greater cortical activity when instructed to ________, whereas European Americans showed greater cortical activity when instructed to ________. A) ignore objects; attend to objects B) think holistically; think analytically C) think automatically; think carefully D) ignore context; attend to context E) attend to context; ignore context

D) ignore context; attend to context

A researcher conducted a survey and found a negative correlation between education and the tendency to resort to violence during disputes. In other words, this researcher found that as education level ________, the tendency to use violence ________. A) increases; also increases B) decreases; also decreases C) increases; remains the same D) increases; decreases E) decreases; remains the same

D) increases; decreases

Which of the following is one of the ethical principles that psychologists must follow when using human participants? All participants must be A) contacted six months after the study to assure that no psychological harm resulted. B) compensated in some way for their participation in research. C) informed of the true nature of the study upon arriving at the laboratory. D) informed that they can withdraw from the study at any time. E) over the age of 18.

D) informed that they can withdraw from the study at any time.

There is invariably a trade-off between ________ and ________ in social psychology experiments. A) mundane realism; psychological realism B) validity; reliability C) independent variables; dependent variables D) internal validity; external validity E) psychological realism; reliability

D) internal validity; external validity

Professor Young is interested in the different ways that men and women communicate about their emotional experiences. Observational research may not be a good method to use because A) researchers' own emotions can colour their interpretations of conversations. B) it is difficult to achieve interjudge reliability when coding conversations. C) it is impossible to use random assignment. D) intimate communications about emotions are often conducted in private. E) people hesitate to talk about their emotions in front of researchers.

D) intimate communications about emotions are often conducted in private.

So much research has been dedicated to automatic thinking because A) it helps us make important decisions. B) it allows us to think deliberately about the social environment. C) automatic thinking takes over from controlled thinking when unusual events occur. D) it is pervasive and dominates much of people's mental life. E) it helps researchers understand counterfactual thinking.

D) it is pervasive and dominates much of people's mental life.

A researcher wants to examine the relation between viewing television violence and behaving aggressively. He has participants decide whether they would prefer to view a violent or a nonviolent film, and subsequently records the number of aggressive behaviours they show in a competitive game. The researcher cannot legitimately make a causal statement based on his findings, because the study A) lacks external validity. B) lacks psychological realism. C) is low on mundane realism. D) lacks random assignment. E) has no cover story.

D) lacks random assignment.

Recall that Bibb Latané and John Darley (1968) studied bystander intervention by having participants overhear a seizure while they were speaking on headphones with another discussant. Assuming that the participants believed that the seizure was real and felt anxious and confused about whether and how to help when they heard the seizure, Latané and Darley's (1968) experiment was A) high in mundane realism, but low in psychological realism. B) high in both mundane and psychological realism. C) high in construct validity, but low in external validity. D) low in mundane realism, but high in psychological realism. E) low in both psychological and mundane realism.

D) low in mundane realism, but high in psychological realism.

In what way is archival research most like the systematic observation of ongoing behaviours? Both methods A) are controversial methods of research in social psychology. B) look for relationships between variables. C) rely on the subjective judgments of trained observers. D) make use of specific, well-defined categories for coding. E) make use of random sampling techniques.

D) make use of specific, well-defined categories for coding.

Pepler and colleagues (2006) assessed the effects of anti-bullying programs in Toronto schools. They came to the conclusion that children who bullied ________ showed the ________ reduction in bullying behaviour as a result of these programs. A) least; most B) least; least C) most; most D) most; least E) moderately; least

D) most; least

Helen is examining the question of when people help. She is concerned about validity, so she conducts an experiment. On the streets of Toronto, she pretends to keel over from a heart attack and records how many people offer help. She performs this act both in the downtown area and in a small suburb. Given the design of her study, Helen was most concerned about the ________ of her research. A) internal validity B) replicability C) operational realism D) mundane realism E) psychological realism

D) mundane realism

Tony has been doing research on age and aggression. He has discovered that the older a person gets, the less likely he or she is to aggress against another person. What kind of relationship best describes Tony's findings? A) positive correlation. B) curvilinear correlation. C) zero correlation. D) negative correlation. E) random correlation.

D) negative correlation.

One hazard of using random assignment and controlling extraneous variables in an experiment is that A) debriefing becomes very cumbersome. B) it is difficult to obtain informed consent from the participants. C) participants often become angry at the deception involved. D) the experimental situation can become quite artificial. E) it is difficult to draw general conclusions.

D) the experimental situation can become quite artificial.

Before Jonas Salk discovered a vaccine to prevent polio, people noticed a correlation between outside temperature and the incidence of polio. Polio cases tended to occur more in the summer months than in the winter months, and the public assumed that high temperatures alone contributed to infection. As it turned out, there was a positive correlation between temperature and polio outbreaks because polio tended to be contracted in swimming pools and other places where children congregated. This illustrates which of the following? Correlations are A) indicative of a causal relation. B) variable. C) invalid. D) no guarantee of a causal relation. E) unreliable.

D) no guarantee of a causal relation.

Recall that Latané and Darley observed the number of participants in each experimental condition who left their cubicles to help the alleged victim of a seizure. The ________ was the independent variable in their experiment. A) number of participants who helped B) reaction of the participants C) personal problems discussed D) number of other discussants E) severity of the seizure

D) number of other discussants

A social psychologist employing the ________ method of research is most like a video camera. A) experimental B) participant observation C) archival analysis D) observational E) correlational

D) observational

The precise specification of how variables are measured or manipulated in a social psychological experiment is called A) ethnography. B) interjudge reliability. C) random assignment. D) operational definition. E) reliability.

D) operational definition.

In most cases, when we encounter a fact that is inconsistent with our schemas, we A) ponder the source of the inconsistency. B) abandon our schemas. C) examine the nature of the inconsistency. D) overlook the inconsistent fact. E) revise our schemas.

D) overlook the inconsistent fact.

Festinger, Riecken, & Schacter's (1956) study of a doomsday cult was conducted using A) archival analysis. B) quasi-experimental methods. C) systematic observation. D) participant observation. E) correlational analysis.

D) participant observation.

Michelle has worked on a psychiatric unit for three years. On the way home from work one day, she encounters a man who gestures wildly and talks to himself while he stands at the bus stop. Michelle thinks to herself, "That poor guy must be psychotic. He should be on medication." This example illustrates that ________ can increase the accessibility of traits. A) unusual situations B) random thoughts C) implicit personality theories D) past experience E) norms

D) past experience

Recall that Frederic Bartlett (1932) interviewed both a Scotsman and a Bantu herdsman about the particulars of a cattle transaction. The Bantu herdsman recited from memory many details, but the Scot had to consult his notes. If this difference were the result of the cultural influence on schemas, the Scot would be more likely to notice and remember the _______ than would the Bantu herdsman. A) songs sung at his daughter's wedding B) number on a basketball player's shirt C) day his father died D) plaid pattern on a man's shirt E) colour of the cattle

D) plaid pattern on a man's shirt

Linnea and Latrice have just left the movie theatre after watching Thelma and Louise, a film that chronicles the misadventures of two close friends, one a rape survivor and the other the wife of a piggish husband; they kill a man in self-defense and run from the law. At the end of the movie, Thelma and Louise chose to drive off a cliff rather than surrender to the men who have been pursuing them. As Linnea and Latrice walk through the lobby, every man they encounter looks loutish and threatening to them. This example illustrates the power of ________ in shaping our implicit personality theories. A) personal experience B) personality heuristics C) schematic norms D) priming E) cultural norms

D) priming

Professor Hui didn't bring enough course evaluation surveys to class, so he distributed the surveys he had to students at the front half of the lecture hall. By doing this, he gave up one of the biggest advantages of surveys: A) honest reports from respondents. B) random assignment to groups. C) the ability to collect large amounts of data. D) random selection to ensure generalizability. E) questions that are easily understood by respondents.

D) random selection to ensure generalizability.

Mundane realism is to ________ as psychological realism is to ________. A) operational definitions; construct validity B) internal validity; external validity C) real-life processes; real-life situations D) real-life situations; real-life processes E) external validity; internal validity

D) real-life situations; real-life processes

In which of the following disciplines are students most likely—by virtue of their experiences—to sigh, "Big deal. I could have predicted that"? A) particle physics B) organic chemistry C) marine biology D) social psychology E) theoretical mathematics

D) social psychology

According to research conducted by Richard Nisbett and his colleagues (1983, 1986, 1987), how might you improve your social reasoning? Take a(n) _______ course. A) history B) economics C) chemistry D) statistics E) biology

D) statistics

When Jana was chiding herself for losing her favorite necklace and reciting a list of all the things she could have done to prevent the loss, her grandmother said, "Now, Jana. Don't cry over spilled milk." In essence, Jana's grandmother was advising her to A) avoid the hindsight bias. B) use base rates rather than the representativeness heuristic. C) avoid the self-fulfilling prophecy. D) stop engaging in counterfactual thinking. E) adjust her judgment farther away from the anchor.

D) stop engaging in counterfactual thinking.

The first day of class Maria absolutely loved her new social psychology professor, but Daniel couldn't stand the professor. If their interpretations were influenced by schemas, in all likelihood, Maria had heard ________, whereas Daniel had heard ________. A) nothing about the professor; that the professor was horrible B) that the professor was great; nothing about the professor C) nothing about the professor; that the professor was supposed to be great D) that the professor was great; that the professor was horrible E) that the professor was an introvert; that the professor was an extravert

D) that the professor was great; that the professor was horrible

The research approach that provides information on the relationship between two variables is called A) unobtrusive observation. B) the experimental method. C) archival analysis. D) the correlational method. E) systematic observation.

D) the correlational method.

In the study by Kunda, Sinclair, and Griffin (1997), participants were told that a person was either a salesperson or an actor and very extroverted. Participants generated descriptions of the person that A) varied according to the occupation of the person. B) did not differ according to the occupation. C) varied depending on the attractiveness of the person. D) were better remembered when consistent with the occupation of the person. E) were similar to the participant's own personality.

D) were better remembered when consistent with the occupation of the person.

________ thinking is nonconscious, effortless, and unintentional, whereas ________ thinking is conscious, effortful, and intentional. A. Illogical; logical B. Controlled; automatic C. Biased; accurate D. Automatic; controlled

D. Automatic; controlled

In which of the following examples does the need for accurate information most conflict with the need for self-esteem? A. Isabel has to decide whether to pay attention to her coach's evaluation of her performance after they have won the semi-final. B. Troy has to decide whether to read his opening night reviews after the play's producer has promised that the play will have at least a six-week run. C. Joy has to decide whether to examine her job review file after receiving a promotion. D. Carlos has to decide whether to read the detailed red ink comments on the 'D' paper he just spent weeks writing.

D. Carlos has to decide whether to read the detailed red ink comments on the 'D' paper he just spent weeks writing.

Diana currently has a goal to lose fifteen pounds by the summer, and she notices that lately her friend, Sarah, hasn't been eating much. Based on your text's discussion of how current goals can affect accessibility, how is Diana most likely to interpret Sarah's behavior? A. Sarah must not like the food. B. Sarah is ill and has no appetite. C. Sarah has symptoms of depression. D. Sarah must be on a diet too.

D. Sarah must be on a diet too.

The ________ approach portrays humans as akin to Sherlock Holmes, who tries his best to accurately make sense of the facts. A. esteem enhancement B. self-esteem C. rational-economic D. social cognition

D. Social Cognition

What is the take-home message from the research regarding the effects of generating different numbers of examples of behavior? A. The number of examples does not affect how much the behavior seems to describe you. B. The fewer examples you have to think of, the less the behavior seems to describe you. C. The more examples you have to think of, the more the behavior seems to describe you. D. The more examples you have to think of, the less the behavior seems to describe you.

D. The more examples you have to think of, the less the behavior seems to describe you.

A group of pedestrians is standing on a corner, waiting for the light to change. A driver runs a red light and narrowly misses another car. According to work on the two-step process of making attributions, who is most likely to point out how quickly the light changed from amber to red when explaining the near accident? A. Dan, who is searching in his pocket for his house keys B. Cynthia, who is mentally rehearsing her shopping list C. Jeff, who is trying to think of what to cook for dinner D. Wesley, who is feeling alert and hasn't a concern in the world

D. Wesley, who is feeling alert and hasn't a concern in the world

Which of the following is an example of social influence? A. You didn't do well on the test because you stayed up all night cramming. B. When you get hungry, you have trouble concentrating. C. You almost fall asleep at the wheel, so you pull off the road to take a short nap. D. You feel guilty because you lied to your trusting professor about your assignment

D. You feel guilty because you lied to your trusting professor about your assignment

The text discusses three reasons why schemas can become accessible. Which of the following is not one of these reasons? A. related to a current goal B. temporary accessibility due to recent experiences C. chronic accessibility due to past experience D. base rate information about a person

D. base rate information about a person

When Marnie met Phillip, he was drunk and loud. Now, months later, she still doesn't like him very much, even though she recently found out the reason he was so drunk was because his cold medicine interacted with the alcohol. The lingering opinion of Phillip is a result of A. primacy effect. B. self-fulfilling prophecy. C. affect blending. D. belief perseverance.

D. belief perseverance.

When a researcher interviewed both a Scotsman and a Bantu herdsman about the particulars of a cattle transaction, the Scot ________, whereas the Bantu herdsman ________. A. guessed; said that he couldn't remember B. erroneously recalled that sheep were traded; accurately recalled that oxen and cattle were traded C. remembered the faces of the traders; remembered the color of the cattle D. consulted his notes; recalled details from memory

D. consulted his notes; recalled details from memory

On your birthday, you arrive home and are overjoyed to find a large bouquet of flowers from your best friend. Your eyes grow wide and a broad smile crosses your face. You laugh in delight. This example best illustrates the use of nonverbal behavior to A. convey attitudes. B. communicate one's personality traits. C. facilitate verbal communication. D. express emotions.

D. express emotions.

Research by Lee Ross suggests that even when people recognize that others perceive information differently,they persist in thinking that others are biased while they themselves are objective. This illustrates the idea of A. individual differences B. social psychology C. implicit values D. naïve realism

D. naive realism

The most important take-home message about controlled thinking is that it: A. replaces automatic thinking. B. increases the efficiency of thinking. C. precedes automatic thinking. D. provides a check on automatic thinking.

D. provides a check on automatic thinking.

Based on information from the authors of your text, what is one possible explanation as to why girls and boys perform about the same in math in elementary school, but a gender gap appears around puberty? A. peer pressure become more important to girls B. testosterone impairs cognition C. estrogen impairs cognition D. self-fulfilling prophecies seem to occur

D. self-fulfilling prophecies seem to occur

Which of the following is a likely criticism of the behaviorist approach? A: Scientists cannot deal with concepts like "thinking" or "feeling" in an objective way. B: Terms like "cognition" or "thinking" or "feeling" are too vague to be studied. C: Concepts like reinforcement and punishment are too vague. D: Behaviorist explanations are too simplistic to explain all human social behavior.

D: Behaviorist explanations are too simplistic to explain all human social behavior.

Social psychologists' emphasis on construals of social situations has its roots in ________ psychology. A: behavioral B: cognitive C: personality D: Gestalt

D: Gestalt

What is the take-home message from the research regarding the effects of generating different numbers of examples of behavior? A: The fewer examples you have to think of, the less the behavior describes you. B: The number of examples does not affect how much the behavior describes you. C: The more examples you have to think of, the more the behavior describes you. D: The more examples you have to think of, the less the behavior describes you.

D: The more examples you have to think of, the less the behavior describes you.

Dr. Yang wonders whether recent acts of airline terrorism have made the public more fearful of airline flight. She secures records of the amount of flight insurance that people have purchased, and compares the records of insurance purchases before the 9/11 terrorist attacks. Dr. Yang has employed a(n) ________ research method. A: participant observation B: systematic observation C: experimental D: archival

D: archival

New professors often have to concentrate very hard to give a lecture, and monitor themselves carefully to make sure they're not talking too fast and that they are using appropriate and clear examples. More seasoned professors do not have to concentrate as much and can talk clearly and engagingly, and keep an eye on students all at the same time. This example represents the transition from ________ thinking to ________ thinking as professors gain experience. A: random; systematic B: automatic; controlled C: unconscious; conscious D: controlled; automatic

D: controlled; automatic

A researcher hired by a department store wants to examine whether manipulating the room temperature to be hotter as opposed to cooler affects sales of air conditioners. The best research method for answering this question would be: A: archival analysis. B: observational. C: correlational. D: experimental

D: experimental

A researcher concludes that frustrating people by giving them a task that is impossible to complete causes them to behave more aggressively. Only ________ warrants this type of conclusion. A: observational research B: an archival analysis C: correlational research D: experimental research

D: experimental research

The hindsight bias occurs because after an event occurs, people: A: repress the memory of their previous predictions because they find them embarrassing. B: have to assert that they predicted it correctly as a way to enhance their self-esteem. C: imagine that the event turned out differently than it did, and this confuses them. D: explain why the event occurred to themselves, and explaining makes it seem inevitable.

D: explain why the event occurred to themselves, and explaining makes it seem inevitable.

A researcher conducted a survey and found a negative correlation between education and the tendency to resort to violence during disputes. In other words, this researcher found that as education level ________, the tendency to use violence ________. A: increases; also increases B: decreases; also decreases C: increases; remains the same D: increases; decreases

D: increases; decreases

Suppose a researcher wants to know whether frustration really does cause aggression. She collects a large number of experimental studies that involve both children and adults, and that are conducted both in the laboratory and in the "real world" in both the United States and other cultures. She then conducts a(n) ________ to determine whether there is enough consistency in findings across studies to determine the generalizability of the relationship between frustration and aggression. A: direct replication B: systematic replication C: archival study D: meta-analysis

D: meta-analysis

When you meet your new roommate for the first time, he is wearing glasses, listening to classical music, and reading a copy of the New Yorker Review of Books. You think, "Well, maybe rooming with an intellectual this year will be good for me." You instantly categorized your roommate as an intellectual on the basis of your A: memories. B: intuitions. C: heuristics. D: schemas.

D: schemas.

Jane analyzed the results of her study and found that the probability of getting such findings by chance was less than 1 in 100 (p < .01). Jane can say that her results are statistically A: generalizable. B: valid. C: realistic. D: significant

D: significant

Replications are designed to ensure that: A: the original study was high in psychological realism. B: the original study was internally valid. C: ethical procedures are always followed. D: the original results can be obtained in different populations and in different settings.

D: the original results can be obtained in different populations and in different settings.

Kurt Lewin once wrote, "There is nothing so practical as a good theory." By that, Lewin was referring to the idea that: A: basic researchers should not be afraid to conduct applied research aimed at solving social problems. B: applied researchers should have a higher status in the field of social psychology. C: the line between basic and applied research is clearer in social psychology than in other sciences. D: to solve social problems, psychologists must understand the processes that underlie them.

D: to solve social problems, psychologists must understand the processes that underlie them.

Christopher is considering breaking up with his high-school sweetheart. He spends many hours weighing the benefits and drawbacks, he asks his friends for advice, and he thinks about it further before coming to his decision. In this case, Christopher is using A: automatic thinking. B: self-fulfilling prophecies. C: the representativeness heuristic. D:controlled thinking.

D:controlled thinking.

Zuzanna had a hard time remembering the names of streets when she traveled because they were in a different language. In addition, the buildings did not look like what she was used to, so she got lost several times. These problems emphasize the role of schemas in ___.

Determining what is accessible.

If both Chinese and Americans are comparing two pieces of art, the Americans are most likely to notice ___.

Differences in foreground

Dr. Diehl and Dr. Jzreck both study aggression. However, Dr. Diehl studies the topic from the standpoint of society at large; Dr. Jrzeck studies it from the standpoint of the individual. Who is most likely to be the social psychologist?

Dr. Jzreck, because he studies individuals as his focus.

Which of the following is the best example of the observational method? A) Chris puts a glass to the wall so that he can hear his parents argue. B) Xena sends out a questionnaire to gain information on people's eating habits. C) Gary stops people on the street to ask them how they voted in the last elections. D) Twyla secretly videotapes guests at her sister's wedding. E) Elaine parks her car near a traffic light and records how many drivers run red lights.

E) Elaine parks her car near a traffic light and records how many drivers run red lights.

Tom and Julius are both in line for a movie, but it sells out before either of them can get tickets. Tom is the last person in line, but Julius is directly behind the person who got the last ticket. Based on the idea of counterfactual reasoning, why will Julius be more upset than Tom? A) Julius got there earlier than Tom did, so he felt he deserved to see the movie. B) Julius wanted to see the movie more than Tom did. C) Julius may have let someone stand in line in front of him. D) Julius won't be more upset than Tom. E) It was easier for Julius to imagine how he could have gotten a ticket.

E) It was easier for Julius to imagine how he could have gotten a ticket.

________ realism refers to the extent to which an experiment triggers the same psychological processes that often occur in everyday life. A) Process B) Situational C) Mundane D) Operational E) Psychological

E) Psychological

________ would increase the external validity of experiments, but social psychologists rarely do it. A) Using multiple independent variables B) Administering follow-up questionnaires C) The use of operational definitions D) Random assignment E) Random selection from the population

E) Random selection from the population

Alyssa and Sara both had their wallets stolen out of their shopping carts at the grocery store. Alyssa always keeps her wallet in her cart when she shops. Sara, however, usually keeps her wallet in a purse, which she holds close to her at all times. The day Sara had her wallet stolen she'd been in a hurry and had just grabbed her wallet and dumped it into the cart. Who would experience more counterfactual thoughts after the theft, Alyssa or Sara? A) Neither would experience counterfactual thinking. B) Alyssa. C) Both would experience about the same amount of counterfactual thinking. D) It is impossible to say. E) Sara.

E) Sara.

_______ are to structure as _______ are to process. A) Decision rules; schemas B) Decision rules; judgments C) Judgments; schemas D) Heuristics; schemas E) Schemas; heuristics

E) Schemas; heuristics

According to the authors, why are people often wrong in asserting that social psychology only reflects common sense? A) People's inferences about psychological phenomena are rarely based in fact. B) Common sense is never correct. C) People are largely ignorant about what exactly social psychologists study. D) Most grandmothers' advice is based on common experience, not scientific evidence. E) So-called common sense findings often make more sense in retrospect than in advance.

E) So-called common sense findings often make more sense in retrospect than in advance.

In describing the intricacies of cross-cultural research, the authors of your text assert that it's not as simple as translating one's materials into an appropriate language, traveling to another culture, and conducting the experiment there. Which of the following statements best captures the role of construals in making cross-cultural research very difficult to do? A) It is very difficult to translate some cover stories and dependent measures into some languages. B) There is variability among people, even within the same culture. C) People in different cultures differ on a number of personal characteristics. D) The experimenter may be culturally biased. E) The same experimental situation can have different meanings in different cultures.

E) The same experimental situation can have different meanings in different cultures.

Which of the following statements best captures the relation between automatic and controlled processing? A) We look before we leap. B) We jump from the frying pan into the fire. C) We judge a book by it's cover. D) We count our chickens before they hatch. E) We shoot first and ask questions later.

E) We shoot first and ask questions later.

According to the authors, why do people often rely on a variety of mental shortcuts? A) We're inherently flawed in our ability to reason. B) We're more interested in speed than in accuracy when we reason. C) We're motivated to enhance our self-esteem. D) We're not interested in much of the information we take in. E) We're confronted with an overwhelming amount of social information.

E) We're confronted with an overwhelming amount of social information.

Which statement below best captures the major ethical dilemma faced by experimental social psychologists? A) The good of the many often comes at the costs to the few. B) Some people do not enjoy participating in psychological research. C) Research results can be misused by those in power. D) It is necessary to lie to discover the truth. E) What is good science may not be ethical science.

E) What is good science may not be ethical science.

Carole has a subtle "Mona Lisa" smile that is much harder to detect than an obvious toothy grin. Assume that before you met Carole, a mutual friend described her as warm and friendly. The first time Carole smiled at you, you would probably conclude that she is A) bored talking to you. B) shy. C) a cool, aloof person. D) very hard to read. E) a friendly person.

E) a friendly person.

Some participants in a study by Tory Higgins and his colleagues (1977) memorized "positive" trait adjectives (e.g., adventurous, neat); others memorized "negative" adjectives (e.g., reckless, disrespectful). All participants then read an ambiguous description of a person named Donald and formed an impression of him. Some, but not all, negative adjectives led to negative interpretations of Donald's actions, and some, but not all, positive adjectives led to positive interpretations of those same actions. These results suggest that thoughts must be both _______ and _______ before they influence our impressions. A) vivid; applicable B) applicable; vivid C) widely shared; vivid D) accessible; widely shared E) accessible; applicable

E) accessible; applicable

Patricia and John have each independently recorded the number of times the words "right" and "responsibility" appeared in a civics textbook. They compared their counts, and found that of the thousands of references to rights and responsibilities, they only disagreed by two occurrences. This example illustrates high A) external validity. B) external reliability. C) internal reliability. D) internal validity. E) interjudge reliability.

E) interjudge reliability.

The greatest drawback to archival analysis is that A) archival data are very difficult to obtain. B) what gets recorded in the archives of a society changes over time. C) it is very difficult to train researchers to code archival data accurately. D) archival data are invariably biased by the researcher's preconceptions. E) archival data were not originally recorded to test hypotheses, so they may be incomplete.

E) archival data were not originally recorded to test hypotheses, so they may be incomplete.

What did Bibb Latané and John Darley (1968) do to ensure the internal validity of their laboratory experiment on the effects of the number of bystanders on participants' responses to an emergency? They A) randomly assigned participants to groups or conditions. B) made sure that the groups had the exact same number of men and women. C) made sure that the seizure overheard by participants was identical for everyone. D) queried participants about their knowledge about epilepsy before beginning the experiment. E) assigned participants at random to conditions that varied on one key variable.

E) assigned participants at random to conditions that varied on one key variable.

One of the most common errors in the social sciences is A) overgeneralizing research findings. B) selecting a biased sample from the population of interest. C) confusing dependent and independent variables. D) prematurely applying research findings to practical problems. E) assuming that correlation automatically implies causation.

E) assuming that correlation automatically implies causation.

Recall Dr. Robert Marion, who was the first physician to correctly diagnose a 9-year- old girl with a rare disease. He explains that "doctors are just like everyone else. We go to the movies, watch TV, read newspapers and novels. If we happen to see a patient who has symptoms of a rare disease that was featured on the previous night's 'Movie of the Week,' we're more likely to consider that condition when making a diagnosis." In essence, Dr. Marion is describing the _______ heuristic. A) representativeness B) primacy C) nonbiased D) counterfactual E) availability

E) availability

In a controlled experiment, Rosenthal and Jacobson (1968) informed grade school teachers that some of their students (called "bloomers") would show great academic improvement in the upcoming year. In reality, the "bloomers" were chosen randomly by the researchers, and were no smarter than any of the other students. Which of the following best describes the results of this study? At the end of the year, A) both bloomers and non-bloomers improved more on IQ tests. B) there were no IQ differences between bloomers and non-bloomers, but the bloomers were more interested in school. C) non-bloomers improved more on an IQ test than did bloomers. D) bloomers and non-bloomers performed equally well on an IQ test. E) bloomers improved more on an IQ test than did non-bloomers.

E) bloomers improved more on an IQ test than did non-bloomers.

You are trying to lose weight by reducing your consumption of desserts. To help you avoid the desserts that you love, you try not to think about them. If you are _______, you will actually be more likely to think about the desserts. A) unmotivated B) cognitively distracted C) not hungry D) extremely above your desired weight E) careless

E) careless

Internal validity is to ________ as external validity is to ________. A) generalizability; causality B) minimizing differences; generalizability C) generalizability; control D) causality; minimizing differences E) causality; generalizability

E) causality; generalizability

In distinguishing between availability and accessibility we can say that availability is related to _______ while accessibility is related to _______. A) stream of consciousness; making judgments quickly and efficiently B) making judgments quickly and efficiently; stream of consciousness C) resemblance to a typical case; making judgments quickly and efficiently D) stream of consciousness; ease of recall E) ease of recall; stream of consciousness

E) ease of recall; stream of consciousness

Why do the authors refer to the experimental method as the "crown jewel" of social- psychological research design? Experiments A) are the most efficient research method. B) require fewer participants than other methods. C) have internal validity. D) involve both dependent and independent variables. E) enable researchers to draw conclusions about cause-and-effect.

E) enable researchers to draw conclusions about cause-and-effect.

Recall that Harold Kelley (1950) told some college students that their guest instructor was a warm person, and told others that he was a cold person. Students who were told that the guest instructor was a warm person evaluated him more positively and participated more in class discussion than did students who were told that he was cold. These results support the idea that schemas are A) impossible to modify. B) generally accurate. C) different, depending on the information to which we are exposed. D) universal. E) especially influential when we encounter ambiguous information.

E) especially influential when we encounter ambiguous information.

No matter how artificial an experimental situation may at first appear, if participants think, feel, or react the way that people in a real-life situation would react, the experiment has A) low demand characteristics. B) high mundane realism. C) low mundane realism. D) little experimenter bias. E) high psychological realism.

E) high psychological realism.

Compared to those in Western cultures, those in East Asian cultures have a more ________ style of thinking. A) analytic B) automatic C) controlled D) schematic E) holistic

E) holistic

A researcher conducted a study and found a high positive correlation between age and income level. In other words, this researcher found that as age ________, income level tends to ________. A) increases; decrease B) increases; stay the same C) decreases; increase D) decreases; stay the same E) increases; increase

E) increases; increase

The main difference between the availability heuristic and accessibility is that availability refers to _______ while accessibility refers to _______. A) information on your mind at any given moment; adjusting an answer according to a starting point. B) the ease or difficulty of bringing a concept to mind; information on your mind at any given moment. C) adjusting an answer according to a starting point; information on your mind at any given moment. D) information your mind at any moment; calculation according to how similar it is to a typical case. E) information on your mind at any moment; the ease or difficulty of bringing a concept to mind.

E) information on your mind at any moment; the ease or difficulty of bringing a concept to mind.

Recall that Latané and Darley (1968) arranged for participants to overhear a seizure during a discussion of personal problems adjusting to college life. Their study was an experiment because A) the taped seizure was actually fake and was identical for everyone. B) the results were analyzed statistically. C) all participants were provided the same opportunity to help. D) the size of the discussion groups was systematically varied by the experimenters. E) participants were randomly assigned to systematically different discussion groups.

E) participants were randomly assigned to systematically different discussion groups.

People are most likely to use judgmental heuristics when A) they have all of the relevant facts. B) they confront a simple "yes-no" decision. C) they are motivated to seek out all relevant facts. D) a lot rides on making the correct decision. E) pre-existing schemas are inappropriate.

E) pre-existing schemas are inappropriate.

When recent experiences are ________, personality traits become ________, increasing the likelihood that those traits will be used to interpret social stimuli. A) ignored; less accessible B) ignored; more accessible C) automatic; less accessible D) primed; less accessible E) primed; more accessible

E) primed; more accessible

An important function of controlled thinking is to A) increase the efficiency of processing. B) replace automatic processing. C) speed automatic processing. D) precede automatic processing. E) provide a check on automatic processing.

E) provide a check on automatic processing.

Participants in a classic experiment conducted by Stanley Milgram (1963) found themselves in the uncomfortable position of being asked to administer increasingly strong shocks to a learner every time he made a mistake. Videotapes of the participants showed them laughing hysterically, chain-smoking, sweating, and fidgeting nervously. Based on these videotapes, it is safe to assume that Milgram's experiment was high on A) external validity. B) internal validity. C) experimental control. D) mundane realism. E) psychological realism.

E) psychological realism.

A social psychologist questions whether or not the participants' willingness to help a seizure victim in Latané and Darley's (1968) experiment was influenced by the fact that it was an apparent medical emergency. This researcher could best investigate this issue by A) surveying people as to how they respond to different types of intervention situations. B) surveying people as to how they usually respond to medical emergencies. C) replicating the study using different types of medical emergencies. D) reanalyzing the data from the original study. E) replicating the study using different types of emergencies.

E) replicating the study using different types of emergencies.

Assume that Latané and Darley want to demonstrate the external validity of their 1968 experiment on bystander intervention. To do this, they conduct a study in which participants solve crossword puzzles alone, or in large or small groups. While the participants are working on the puzzles, they overhear a woman fall from a ladder and cry out, "Oh, my leg! I can't move it!" The researchers then observe and record the number of participants who attempt to help. The study described here represents a(n) ________ of the original "seizure" study. A) needless duplication B) encore C) psychological copy D) internal validity check E) replication

E) replication

"If it walks like a duck, and quacks like a duck, it must be a duck..." This expression best captures the essence of the A) judgmental heuristic. B) correspondence heuristic. C) fundamental attribution error. D) availability heuristic. E) representativeness heuristic.

E) representativeness heuristic.

Researchers at the University of Manitoba assessed attitudes toward deception research in 1970 and again in 1990 (Sharpe, Adair & Roese, 1992). The results show that A) participants who were deceived were less likely to agree that deception research was necessary. B) although students did not react negatively to the deception experiments they did show a greater distrust of psychologists. C) attitudes toward deception research were less favourable in the more recent (1990) survey. D) attitudes toward deception research were more favourable in the more recent (1990) survey. E) respondents did not feel negatively about their experiences in either time period.

E) respondents did not feel negatively about their experiences in either time period.

One of the authors of your text supervised an honours student from China who attended a curling championship in Winnipeg. The student was very amused to see people paying so much money to watch "a couple of guys mopping the floor." This story suggests that A) many schemas are common to different cultures. B) some cultures do not use schematic processing. C) different cultural schemas influence what people remember. D) the use of schemas is a human universal, no matter what the culture. E) schema are affected by one's native culture.

E) schema are affected by one's native culture.

Before the experiment begins, the researcher is ethically obligated to A) get permission from the Canadian Psychological Association. B) debrief all participants. C) randomly assign subjects to conditions. D) provide a cover story. E) secure informed consent.

E) secure informed consent.

From an ethical perspective, what two goals often collide in social psychological research? On the one hand, social psychologists ________; on the other hand, they ________. A) provide cover stories to disguise their true purposes; debrief participants at the close of the study B) use a limited number of participants in their studies; want to generalize to all people C) are concerned about their participants; want to find interesting results D) want to satisfy their intellectual curiosity; want to respond to real-life problems E) seek both mundane and psychological realism; don't want to cause participants unnecessary discomfort or harm

E) seek both mundane and psychological realism; don't want to cause participants unnecessary discomfort or harm

If Professor Klein is using EEGs and fMRIs to study social behaviour, she is most likely to be involved in A) clinical psychology. B) computer science research. C) research on internet social networking. D) cognitive neuroscience. E) social neuroscience.

E) social neuroscience.

Although there are only a few runners on your college track team, you see a woman on campus who has the physique of a runner. You decide that she's a member of the track team. Your conclusion is based on A) the availability heuristic. B) the base rate fallacy. C) base rate information. D) biased sampling. E) the representativeness heuristic.

E) the representativeness heuristic.

Jason assumes that redheads are hot-tempered. When asked whether Linda, a redhead, has a quick temper, Jason replies, "My confident prediction is that she is hot- tempered." Jason has just applied _______ to _______. A) the availability heuristic; the representativeness heuristic B) his schema for redheads; the representativeness heuristic C) the vividness effect; his schema for redheads D) the availability heuristic; his schemas for redheads E) the representativeness heuristic; his schema for redheads

E) the representativeness heuristic; his schema for redheads

According to research by Dijksterhuis (2004), participants who had to choose among four different apartments in a short period of time made the best choice when A) the researcher asked the participant to choose immediately the apartment that they thought was the best. B) the researcher asked the participant to think carefully about the apartments for three minutes and then make a choice. C) the researcher asked participants to go with their "gut feeling". D) the researcher asked participants to reach consensus. E) the researcher gave the participants a distracting task for three minutes and then asked them to make a decision.

E) the researcher gave the participants a distracting task for three minutes and then asked them to make a decision.

A major problem with schemas is that A) they take a long time to learn. B) they are more often wrong than right. C) we rely on them too much. D) they allow us to "fill in the gaps" and interpret ambiguous information. E) they operate automatically, making it difficult for us to know when we're using them.

E) they operate automatically, making it difficult for us to know when we're using them.

In their experiment, why did Bibb Latané and John Darley (1968) expose all participants to the same audiotaped seizure? They wanted A) the dependent variable to be the same for all participants. B) the independent variable—the overheard seizure—to be different for different participants. C) to be able to assign participants at random. D) the independent variable to be the same for all participants. E) to control extraneous variables, such as the quality of the fake seizure.

E) to control extraneous variables, such as the quality of the fake seizure.

Assume that the winning lottery ticket was purchased at the convenience store just around the corner from your house. According to research on counterfactual thinking, you would probably feel worse than if the winning ticket had been purchased in another city, because A) it would be easy to imagine all the wonderful things you could do with the prize. B) you would assume that the local winner is similar to you. C) you could easily remember what it was like to struggle to make every penny count. D) you would be reminded of your loss whenever you see the store. E) you could think back on a dozen opportunities that you had to buy the winning ticket, but didn't.

E) you could think back on a dozen opportunities that you had to buy the winning ticket, but didn't.

Which of the following is the best example of the observational method?

Elaine parks her car near a traffic light and records how many drivers run red lights.

A cult in the 1950s believed that the world was coming to an end very soon. Researchers Leon Festinger and his colleagues studied this cult by joining the group and pretending to share these beliefs. What research method were they using?

Ethnography

"Evolutionary psychology is silly!" exclaims Julie. "If we are so adapted to the environment, why haven't we evolved away from racism?" A reasonable response to this, based on the explanation of evolutionary psychology presented by the authors of your text, is that ___.

Evolution occurs very slowly, and thus genetic adaptations have not "caught up."

Justin isn't sure if he wants to date Mary, with whom he shares many similarities, or Emma, who is very different from him. His friend says, "Opposites attract," and advises him to date Emma. But his brother says, "Birds of a feather flock together," and suggests that he pursue Mary. This best exemplifies that ___.

Folk wisdom is often full of contradictions

Jake had a hypothesis about the outcome of the Ross and colleagues (2004) study about the "Wall Street Game" and the "Community Game." Jake hypothesized that the players would respond based on their personalities, not just the name of the game they played. His hypothesis is most likely based on which tendency?

Fundamental attribution error

Marta left a party feeling very upset. Rather than trying to recall each conversation she had during the party, Marta tried to explain her feelings by reflecting on the party as a whole. The process Marta used resembles the approach used by ________ psychologists.

Gestalt

The whole is different from the sum of its parts. This statement reflects a tenet of________ psychology.

Gestalt

In their approach to understanding social behavior, social psychologists are most similar to

Gestalt psychologists.

In commenting on the development of social psychology in the U.S., Dorwin Cartwright (1979) wrote, "If I were required to name the one person who has had the greatest impact on the field, it would have to be Adolf Hitler." By this he meant that

Gestalt psychology came to the U.S. when Gestalt psychologists fled the Nazi regime.

The shortcuts that people use to make judgments and decisions quickly and efficiently are called...

Heuristics

When distinctiveness is ___, consensus is ___, and consistency is high, people are likely to make an external attribution.

High; high

Research has demonstrated that metaphors about the body and social judgments influence how we think and the decisions we make. If that is true, the next time you are feeling lonely and as if the world is a cold, heartless place, what should you do and why?

Hold a warm beverage in your hands because warmth is associated with friendliness

The authors of your text discuss three snares in peoples' ability to reason accurately. All of the following are snares in reasoning except

People are seldom motivated to be accurate.

According to the authors of your text, why do people sometimes construe information to feel good about themselves?

People have a strong need to maintain their self-esteem.

How would a personality psychologist most likely explain the mass suicide in Jonestown?

People who have traits of being unstable are more likely to join cults.

The Kitty Genovese murder inspired research on bystander apathy. This example illustrates the usefulness of relying on ___ in formulating research hypotheses.

Personal observations of everyday life

Dr. Levowitz researches the question, "Are some people more aggressive than others?" This type of question is most closely in the realm of ___.

Personality psychology

Darwin believed that emotional expressions began as ________ that came to have evolutionary value because they ________.

Physiological reactions; convey emotional states to other members of the species

You've rented the movie Sophie's Choice. There is a scene in which Sophie, a recent immigrant from Poland, is both surprised and amused that Americans have so many words for the concept "fast." She reports that in Polish, there is only one word. You've just read Chapter 3 (Social Cognition), and a friend asks you why the cultures differ in that regard. What would you say?

Poles don't have well-developed schemas for the concept "fast."

One day, Professor Burns came into class and put his briefcase onto the desk in the front of the room. The desk promptly fell apart and the briefcase tumbled to the floor. Because of perceptual salience, what will students be most likely to think?

Professor Burn ruined that desk.

The study by Rosenthal and Jacobson on self-fulfilling prophecies concluded that the teachers treated the "bloomers" in class differently than the other children. If you behaved similarly with a child or younger sibling, how might you behave?

Providing more constructive feedback.

The authors of your text present a study by Masuda and his colleagues (2008) that examined cultural differences in social perception. They compared how participants perceived a central character when the surrounding characters in a picture either matched or mismatched the central character's facial expression. When the central and peripheral characters' expressions do not match, Japanese participants were most likely to ___.

Rate the central character more similarly to the peripheral characters.

Professor Rothman is interested in tracking changes in racial stereotypes in the U.S. If he decides to conduct an archival analysis, he should ___.

Record how minorities are portrayed in cartoons in the New Yorker magazine from 1940-2010.

Recall that Ross and Samuels (1993) randomly assigned participants previously identified as either competitive or cooperative to one of two games: the "Wall Street Game" or the "Community Game." Also recall that fully twice as many players in the "Wall Street Game" behaved competitively compared to people who played the "Community Game." What do these findings suggest?

Seemingly minor aspects of a social situation can override personality differences.

Several studies conducted by Nisbett and his colleagues involved showing participants two similar pictures and asking them to find the differences between them. Nisbett and colleagues found that East Asian participants were more likely to focus on details such as ___.

Shape of the control tower.

Nora has been blind since birth, and has never seen a smile, frown, or furrowed brow. At age three, how capable would she be of expressing emotions using the facial expressions of sighted people?

She would be able to express all six basic emotions just as well as sighted people.

Remy has recently learned about Kelley's covariation model. He uses it when his sister does something that isn't typical for her so the consistency of that behavior is low. What kind of attribution does he make for her behavior?

Situational attribution

In general, when people first encounter a novel situation, they tend to ___.

Size up the situation quickly and effortlessly and, for the most part, correctly.

_______ refers to the way people select, interpret, remember, and use social information to make judgments and decisions about themselves and others.

Social Cognition

Aya is eight months old, and her mother pretends her baby food is a train in order to convince her to eat it. Aya's mother is using a rather creative form of ___.

Social Influence

Which of the following is true about social psychologists' interest in social problems?

Social problems have been a concern since the beginning of social psychology.

Professor Hume has spent the last ten years studying the effects of people's levels of self-esteem on their tendency to discriminate against others. She is most likely to be

a social psychologist.

According to the perspective presented in Chapter 3 (Social Cognition), a person who grew up in a home with an alcoholic parent who sees a man acting in a somewhat strange manner may be ________ likely to interpret this behavior as due to alcohol because of ________. a. more; chronic accessibility of an alcoholic schema b. more; ego-defensive biases c. less; chronic accessibility of an alcoholic schema d. less; ego-defensive biases

a.

Although there are only a few runners on your college track team, you see a woman on campus who has the physique of a runner. You decide that she's a member of the track team. Your conclusion is based on a. the representativeness heuristic. b. the availability heuristic. c. base rate information. d. biased sampling.

a.

Assume that you want to avoid the self-fulfilling prophecy the next time you meet someone who fits a schema that you hold. According to work on the On Automatic Pilot: Low-Effort Thinking described in the text, what should you do? You should make sure that a. you're not preoccupied with other things. b. you let the other person direct the conversation. c. your schema has worked in the past. d. you seek out evidence that contradicts your schema.

a.

At the grocery store, mothers often keep children busy by letting them "drive" the cart. Little do the children realize that the steering wheel they are playing with does not control anything. This example demonstrates our sometimes misplaced sense of control over a. free will and our own behavior. b. self-esteem and our own feelings. c. schemas and their contents. d. heuristics and their processes.

a.

Blair's co-worker, Susan, asks her, "Am I a good employee?" Blair hadn't really thought about it before, but using the availability heuristic, which of the following thought processes is she most likely to have? a. Blair recalls the last few times she's seen Susan do a good job, and tells her yes. b. Blair makes a list of every behavior Susan has done in the organization before answering. c. Blair uses the oldest information she can recall about Susan's performance. d. Blair makes a snap judgment based on what she thinks Susan wants to hear.

a.

Ethan and Heather see a man stumbling around as he walks down the street. Ethan, who belongs to Alcoholics Anonymous, thinks the man is drunk, but Heather, who just watched a TV special on Michael J. Fox, thinks the man has Parkinson's disease. These differing interpretations of the same behavior seem to be caused by Ethan and Heather's differences in a. accessibility. b. current goals. c. stereotypes. d. self-fulfilling prophecies.

a.

Eva runs an unscrupulous organization that attempts to scam money from people while posing as a charity. Based on the research by Shariff and Norenzayan (2007), the best strategy for doing this involves a. mentioning fairness several times in the brochures b. putting happy pictures on the brochures c. telling potential donors they are "stars" d. mentioning potential donors' names several times

a.

If both Chinese and Americans are comparing two pieces of art, the Americans are most likely to notice a. differences in the foreground. b. the color schemes. c. differences in how objects are oriented. d. differences in brushstrokes.

a.

Isaac is on an interview for a job for which he is well qualified. However, he is worried that some of the people may have negative schemas about his ethnicity and that they may shape his behavior on the interview with self-fulfilling prophecies. Of the people he meets with (listed below), who is the least likely to use self-fulfilling prophecies? a. Tyler, who is paying close attention to Isaac, and wants to make an accurate judgment of him b. Lisa, who wants to make an accurate judgment, but can't stop thinking about her report due next week c. Jorge, who is paying close attention to the interview, but isn't very motivated to be accurate because he has no say in the hiring decision d. Jasmine, who is preoccupied with a toothache, but has the final say in whether Isaac gets the job

a.

The study by Rosenthal and Jacobson on self-fulfilling prophecies concluded that the teachers treated the "bloomers" in class differently than the other children. If you behaved similarly with a child or younger sibling, how might you behave? a. Providing more constructive feedback. b. Giving extra time to do their homework. c. Giving less time to respond to questions. d. Scolding the child when they get an answer wrong.

a.

What is the major advantage of processing information automatically? a. We can use our cognitive resources for other, more important matters. b. Automatic processing yields more accurate judgments than does controlled processing. c. It is easier to learn how to process information automatically. d. Automatic processing is more easily modified than controlled processing.

a.

When Robin meets her friend's new roommate, she sees that she has easels and paints strewn about, and has decorated her side of the room with lots of hand-made ceramic pots and paintings. At first, Robin is most likely a. to use her "artistic" person schema and assume that her friend's roommate is free-thinking and creative. b. not to make any assumptions about her friend's roommate's preferences and tastes. c. to use her "artistic" person schema and focus on her friend's roommate's deviations from the schema, such as the fact that she is majoring in business. d. to change her "artistic" person schema when she discovers that her friend's roommate is a business major.

a.

When asked a question during Trivial Pursuit, Sylvia is certain that her answer is correct, even though her teammates disagree. Sylvia cannot be persuaded by her teammates to reconsider how accurate her response is. She is demonstrating the a. overconfidence barrier. b. priming effect. c. availability heuristic. d. monitoring process.

a.

When you meet your new roommate for the first time, he is wearing glasses, listening to classical music, and reading a copy of the New Yorker Review of Books. You think, "Well, maybe rooming with an intellectual this year will be good for me." You instantly categorized your roommate as an intellectual on the basis of your a. schemas. b. memories. c. intuitions. d. heuristics.

a.

You wrongly assume that because of Linda's short hair, unshaved legs, and liberal views she is a political activist. Your (incorrect) assumption was based on the a. representativeness heuristic. b. availability heuristic. c. anchoring and adjustment heuristic. d. self-fulfilling prophecy.

a.

Your roommate questions how charitable you really are. She says to you, "Give me just one example of the last time you gave money to a homeless person." Odds are that her challenge will convince you that you really are a generous person. Based on research by Schwarz and his colleagues (1991), why is that? a. It's easy to bring to mind one example that supports your self-schema. b. Your roommate's challenge makes you feel defensive and threatens your self-esteem. c. Homeless people are representative of people who need our help. d. It's easier to think of "actuals" than counterfactuals.

a.

Shanika is an executive, and asked her assistant repeatedly to make some copies for her. The assistant repeatedly failed to successfully complete the assignment. If Shanika is thinking like a social psychologist about this situation, what is she most likely to think about her assistant?

a. "My assistant is incompetent and I should fire him immediately." b. "All of my subordinates are incompetent." c. "I am the only intelligent person in this office." *d. "Perhaps my assistant is under stress from something else."

Which of the following is an example of a direct persuasion attempt? a. A bully threatens Billy and steals his lunch money. b. Ramona works hard in school to make her mother proud. c. Marianne thinks of her ex-boyfriend and becomes sad. d. Jason moves from New York to Atlanta and picks up a Southern accent.

a. A bully threatens Billy and steals his lunch money.

Which of the following is true? a. All human beings have the same cognitive "tools" that they can use. b. When people move from one culture to another they generally do not learn to think like people in the new culture. c. East Asians tend to think more holistically and Westerners tend to think more analytically because of genetic differences between East Asians and Westerners. d. American college students were more likely to notice changes in the background of a picture whereas Japanese college students were more likely to notice changes in the main objects in the foreground of the picture.

a. All human beings have the same cognitive "tools" that they can use.

Which of the following is the best summary of research on automatic thinking? a. Automatic thinking is vital to human survival, but it is not perfect and can produce mistaken judgments that have important consequences. b. Automatic thinking is amazingly accurate and rarely produces errors of any consequence. c. Automatic thinking is a problem because it usually produces mistaken judgments. d. Automatic thinking works best when it occurs consciously.

a. Automatic thinking is vital to human survival, but it is not perfect and can produce mistaken judgments that have important consequences.

Which of the following is a likely criticism of the behaviorist approach?

a. Concepts like reinforcement and punishment are too vague. *b. Behaviorist explanations are too simplistic to explain all human social behavior. c. Terms like "cognition" or "thinking" or "feeling" are too vague to be studied. d. Scientists cannot deal with concepts like "thinking" or "feeling" in an objective way.

Wan is a Korean student and Jim is an American student. Both take part in an experiment in which they listen to another student read aloud her essay, knowing that this other student's essay was on an assigned position. Then they are asked to indicate what this other student really believes. Based on the results of cross-cultural research on the fundamental attribution error, which of the following is true? a. It is possible for both Wan and Jim to make the fundamental attribution error. b. It is possible for Wan, but not Jim, to make the fundamental attribution error. c. It is possible for Jim, but not Wan, to make the fundamental attribution error. d. Neither Jim nor Wan will ever make the fundamental attribution error.

a. It is possible for both Wan and Jim to make the fundamental attribution error.

Recall that Roesch and Amirkham (1997) studied attributions for success and failure in athletes. Based on their findings, which of the following athletes would be most likely to make an external attribution for losing? a. John, a tennis player new to the professional circuit b. Joe, a ten-year veteran of the San Francisco 49ers football team c. Karim, a rookie (i.e., new player) on an NFL basketball team d. Wayne, a hockey player whose skills have deteriorated due to injuries

a. John, a tennis player new to the professional circuit.

According to the authors of your text, when faced with a puzzling social question, it may be tempting to ask people why they behaved as they did. Why is this not always the best way to understand social behavior?

a. People almost always lie when they are interviewed. b. People would feel defensive, even when asked benign questions. *c. People would not necessarily know why they behaved as they did. d. People would simply answer randomly

Research on controlled thinking and free will shows that: a. There is a disconnect between our conscious sense of how much we are causing our actions and how much we are really causing our actions. b. It doesn't really matter whether or not people believe that they have free will. c. Some primates have just as much free will as human beings. d. People definitely do not have free will.

a. There is a disconnect between our conscious sense of how much we are causing our actions and how much we are really causing our actions.

Ross and colleagues (2004) randomly assigned participants previously identified as either competitive or cooperative to one of two games: the "Wall Street Game" or the "Community Game." They found that two-thirds of the players in the "Wall Street Game" behaved competitively compared to one-third of people who played the "Community Game" who behaved competitively. What do these findings suggest?

a. True personality differences do not exist. b. It is not important to study individual differences in personality. *c. Seemingly minor aspects of a social situation can override personality differences. d. All the competitive people ended up playing the "Wall Street Game."

Isaac is on an interview for a job for which he is well qualified. However, he is worried that some of the people may have negative schemas about his ethnicity and that they may shape his behavior on the interview with self-fulfilling prophecies. Of the people he meets with (listed below), who is the least likely to use self-fulfilling prophecies? a. Tyler, who is paying close attention to Isaac, and wants to make an accurate judgment of him b. Lisa, who wants to make an accurate judgment, but can't stop thinking about her report due next week c. Jorge, who is paying close attention to the interview, but isn't very motivated to be accurate because he has no say in the hiring decision d. Jasmine, who is preoccupied with a toothache, but has the final say in whether Isaac gets the job

a. Tyler, who is paying close attention to Isaac, and wants to make an accurate judgment of him

Both social psychologists and sociologists are interested in aggression. Compared to sociologists, which of the following questions is a social psychologist most likely to ask?

a. What is the effect of handgun laws on homicide rates in different states? *b. When does anger lead to aggression? c. Are homicide rates higher among members of the lower socioeconomic class? d. Do prisons deter homicide?

Based on the idea of "telling more than you can know" from your text, which of the following survey questions would be least likely to yield accurate responses? a. Would you ever consider physician-assisted suicide? b. Have you ever been the victim of a violent crime? c. How many different magazines do you subscribe to? d. In your opinion, who performed better in the last presidential debate?

a. Would you ever consider physician-assisted suicide?

The "father" of modern experimental social psychology, Kurt Lewin, pointed out the importance of understanding how people ________ their social environments.

a. control *b. construe c. choose d. structure

Schemas are most like which of the following? a. lenses through which we look at and learn about the world b. tires on which we move about in the world c. food which helps us stay energized to explore the world d. pillows which help us recover from our time in the world

a. lenses through which we look at and learn about the world

Ross and colleagues (2004) found that people playing the "Wall Street Game" were ________ competitive than people playing the "Community Game," ________ of individual differences in competitiveness and cooperativeness.

a. more; because *b. more; regardless c. less; because d. less; regardless

The word "construal" refers to

a. objective reality. b. information provided by other people. c. mental energy. *d. personal interpretations.

Sheila shows up for a blind date with her hair disheveled and her clothes a mess. Her date, Jamal, thinks, "She must be a total slob!" Jamal's thought about Sheila is an example of

a. the hindsight bias. *b. the fundamental attribution error. c. a self-fulfilling prophecy. d. gender differences in perception

Social and personality psychologists share which common goal?

a. understanding individual differences b. understanding how the presence of others influences people c. understanding people who are mentally ill *d. understanding causes of human behavior

imagine that you have just moved into a house with four other people. One of your housemates, Tony, doesn't seem to like you all that much. If you were to make use of the Ben Franklin effect, what can you do to increase Tony's liking for you?

ask Tony if he could give you a ride to the grocery store

After seeing the movie Fargo, Craig firmly believes that Minnesotans and North Dakotans are the goofiest people in the United States. You believe that Minnesotans and North Dakotans are no goofier than people any place else, and you would like to get Craig to change his judgment. To do this, you might

ask him to consider the opposite point of view

Alice goes to see a rerun of Jaws (a movie about a vicious shark) a few days before she takes off to Florida on spring break. On break, she finds that she is now too nervous to spend much time actually in the water, since she is convinced that anything she sees in the water is a shark. Her heightened sensitivity is due to a. self-fulfilling prophecy. b. priming. c. anchoring and adjustment. d. representativeness.

b.

Allison is a very sweet, intelligent, amicable person. However, Joe hears from her ex-boyfriend that she is hard to get along with and very snobby. According to research on the self-fulfilling prophecy, Joe acts in a(n) a. neutral way toward Allison, even though he has heard bad things about her. b. unfriendly way toward Allison. This in turn causes Allison to act unfriendly toward Joe, and Joe to believe that her exboyfriend was right about her. c. unfriendly way toward Allison, yet he finds her to be very amicable and decides that her ex-boyfriend must not know her very well. d. friendly way toward Allison even though he has heard bad things about her, and finds her to be much friendlier than her ex-boyfriend said she was.

b.

Carol has a subtle "Mona Lisa" smile that is much harder to detect than an obvious toothy grin. Assume that before you met Carol, a mutual friend described her as warm and friendly. The first time Carol smiled at you, if you were to rely on your schemas, you would probably conclude that she is a. bored talking to you. b. a friendly person. c. a cool, aloof person. d. very hard to read.

b.

Fidan wrecked his dad's car. When his dad got home from work, Fidan met him at the door with a cold beer and his favorite magazine. The smell from his dad's favorite meal was wafting through the house, and his dad's favorite CD was playing. Fidan is trying to use ________ to get his father to think of him as a good, responsible son before he tells him about the car. a. representativeness heuristic b. priming c. controlled thinking d. counterfactual thinking

b.

Ms. Honeywell is an elementary school teacher who maintains that in her thirty-five years of experience, boys just tend to do better at math than girls. Recently, she has read a number of studies that show that girls and boys actually have about the same 22 Copyright © 2013, 2010, 2007 Pearson Education, Inc. All rights reserved. level of math ability. Based on information about how teachers create and sustain self-fulfilling prophecies, what is Ms. Honeywell most likely to think about the study results? a. "Well, I guess everyone is wrong once in a while—maybe boys and girls are the same." b. "I don't buy it—three times as many boys as girls in my classes have excelled at math." c. "Come to think of it, I think girls may actually be better than boys." d. "Who cares, girls are better than boys at reading anyway—the differences even out."

b.

Recall the words of Dr. Robert Marion, who was the first physician to correctly diagnose a nine-year-old girl with a rare disease: "Doctors are just like everyone else. We go to the movies, watch TV, read newspapers and novels. If we happen to see a patient who has symptoms of a rare disease that was featured on the previous night's 'Movie of the Week,' we're more likely to consider that condition when making a diagnosis." In essence, Dr. Marion is describing the ________ heuristic. a. representativeness b. availability c. primacy d. counterfactual

b.

Research demonstrated that physical sensation can activate metaphors that influence our judgments about unrelated topics. If you wanted students to take your petition seriously, you should present the information about it a. on a piece of paper they can take with them. b. on a heavy tablet. c. on a light clipboard. d. verbally rather than written.

b.

Research has demonstrated that metaphors about the body and social judgments influence how we think and the decisions we make. If that is true, the next time you are feeling lonely and as if the world is a cold, heartless place, what should you do and why? a. clean your room because it is associated with "washing away sins" b. hold a warm beverage in your hands because warmth is associated with friendliness c. carry something heavy because that is associated with "carrying the weight of the world" d. take a nap because "life is but a dream"

b.

Tiffany has a hard time trusting other people her age because she believes they are irresponsible. Accordingly, when she makes dinner plans with one friend, she also makes backup plans with someone else, and she goes to one or the other. Her friends soon in turn begin to "blow off" their arrangements since it is not clear whether or not Tiffany will show up. Tiffany has created a. a confirmation bias. b. a self-fulfilling prophecy. c. counterfactual thinking. d. controlled unbelieving.

b.

When Ashley first starts typing, she uses the "hunt and peck" method and finds typing very effortful. Now, after taking a typing class, she feels like her papers practically type themselves once she figures out what she wants to say. This change in her typing reflects a shift from a. automatic to controlled thinking. b. controlled to automatic thinking. c. emotional to cognitive thinking. d. cognitive to emotional thinking.

b.

When Matilda met Joe, she instantly categorized him as a nerd because of his appearance and intense interest in technology and mathematics. Matilda's categorization is an example of her use of a. scripts. b. schemas. c. controlled processes. d. base rate information.

b.

When asked to guess whether Mark is from Montana or California, you guess California because more people live in California. You have used ________ in making your decision. a. the representativeness heuristic b. base rate information c. the availability heuristic d. counterfactual thinking

b.

Which of the following is not a typical example of the kind of knowledge that is contained in a schema? Your knowledge a. of the typical character traits and behaviors of lawyers. b. of the first date when you spilled red punch all over your partner. c. that fairy tales begin, "Once upon a time...." d. that it is common to leave a 15-percent tip when dining at an eat-in restaurant.

b.

Which of the following is not an example of automatic thinking? a. forming a first impression of someone within seconds of meeting him or her b. conducting a search on the Internet c. having a sense of whether an object is near or far from us d. driving down the highway while also carrying on a conversation

b.

You stayed up all night cramming for this examination and didn't do as well as you had hoped. "If only I had started studying sooner and gotten a good night's rest, I'd have done much better," you think to yourself. You have just engaged in a. self-justification. b. counterfactual thinking. c. wish fulfillment. d. unrealistic fantasy.

b.

Which of the following is a likely criticism of the behaviorist approach? a. Concepts like reinforcement and punishment are too vague. b. Behaviorist explanations are too simplistic to explain all human social behavior c. Terms like "cognition" or "thinking" or "feeling" are too vague to be studied. d. Scientists cannot deal with concepts like "thinking" or "feeling" in an objective way.

b. Behaviorist explanations are too simplistic to explain all human social behavior

Based on information from the authors of your text, who would be more likely to express a feeling of shame publicly? a. Ken, from an individualist culture b. Tran, a man from a collectivist culture c. Lisa, from an individualist culture d. Nanami, a woman who grew up in both individualist and collectivist cultures

b. Tran, a man from a collectivist culture

Based on everything you've read in this chapter, what is the best conclusion about social cognition? a. People would be better off if we could turn off automatic thinking and rely solely on controlled thinking. b. Whereas people are very sophisticated social thinkers who have amazing cognitive abilities, there is also plenty of room for improvement. c. Social cognition is pretty much the same throughout the world in all cultures that have been studied. d. One purpose of controlled thinking is to set goals for ourselves; that cannot be done with automatic thinking.

b. Whereas people are very sophisticated social thinkers who have amazing cognitive abilities, there is also plenty of room for improvement.

James has a goal of being able to run a marathon by next year. When he sees a woman dart out of a store and start running he immediately thinks, "Oh, she must be a runner too!" rather than, "She's a shoplifter trying to get away." Which of the following is the best explanation for why one schema was accessible rather than another? a. chronic accessibility due to past experience b. a current goal c. base-rate information d. applicability of priming

b. a current goal

Which of the following is not an example of automatic thinking? a. forming a first impression of someone within seconds of meeting him or her b. conducting a search on the Internet c. having a sense of whether an object is near or far from us d. driving down the highway while also carrying on a conversation

b. conducting a search on the Internet

Which of the following is used in experiments, but not in the correlational method? a. measurement of a variable such as how aggressive or how helpful people are b. systematically manipulating the situation c. representative sampling of people from a population d. surveys

b. systematically manipulating the situation

Lucy is finishing a long jog in scorching heat. She promised to visit her friends Jessie and Ellen today. Even though she likes Ellen more, she decides to stop by Jessie's house at the end of her run because Jessie keeps her house cool, and always has cold drinks in her fridge. What motivated Lucy to decide to visit Jessie?

biological drive

Brigit is trying to not think about the fight she had with a roommate because she needs to pay attention during her psychology class that starts in 20 minutes. According to research in the text, what advice would you give her?

both a and b

In which of the following examples does the need for accurate information most conflict with the need for self-esteem? a.) Joy has to decide whether to examine her job review file after receiving a promotion. b.)Troy has to decide whether to read his opening night reviews after the play's producer has promised that the play will have at least a six-week run. c.) Carlos has to decide whether to read the detailed red ink comments on the 'D' paper he just spent weeks writing. d.)Isabel has to decide whether to pay attention to her coach's evaluation of her performance after they have won the semi-final.

c

Which of the following is not true about the social psychological approach? a.)You can only determine causality using the experimental method. b.)A person's need to make accurate judgments of the world often determines how he or she views a situation. c.) To understand the power of social influence, you have to understand the nature of the objective situation, not how people perceive it. d.)A person's need to maintain self-esteem often determines how he or she views a situation.

c

Based on the study by Shariff and Norenzayan (2007) presented in your text, if you pass by several churches, then a block or so later a homeless person asks you for change, how are you likely to behave? a. You would probably take the person to the churches you passed to get help. b. You would be very unlikely to give the person money. c. You would be more likely to give the person money. d. Passing by churches would not prime you to give more money, but passing by banks might.

c.

Christopher is considering breaking up with his high-school sweetheart. He spends many hours weighing the benefits and drawbacks, he asks his friends for advice, and he thinks about it further before coming to his decision. In this case, Christopher is using a. automatic thinking. b. the representativeness heuristic. c. controlled thinking. d. self-fulfilling prophecies.

c.

Consider the results from a study by Miyamoto and colleagues in which participants viewed either photos of U.S. city scenes or Japanese city scenes, then were asked to detect differences between two similar pictures. Based on their findings, if an American was touring in Tokyo, Japan, which of the following aspects of a picture would she be most likely to pick up on? a. foreground b. objects c. background d. contrasting colors

c.

Diana currently has a goal to lose fifteen pounds by the summer, and she notices that lately her friend, Sarah, hasn't been eating much. Based on your text's discussion of how current goals can affect accessibility, how is Diana most likely to interpret Sarah's behavior? a. Sarah is ill and has no appetite. b. Sarah has symptoms of depression. c. Sarah must be on a diet too. d. Sarah must not like the food.

c.

If you grew up in a household with a brother who was morbidly shy, you might have a different belief about a man you meet who avoids dating women than a person who grew up in a household with a brother who was gay. This difference in interpretation is due to differences in your and the other person's a. schemas for gay men and shy men. b. prejudices against gays. c. chronic accessibility of gay vs. shy schemas. d. temporary priming of gay vs. shy schemas.

c.

Isabella goes to a fortune teller who tells her, "You are the type of person who is very outgoing and sociable, yet at times you enjoy being alone." Isabella thinks this captures her personality amazingly well, because she finds many instances in her memories of such behaviors. Isabella has just fallen prey to the a. base rate heuristic. b. priming effect. c. Barnum effect. d. self-fulfilling prophecy.

c.

New professors often have to concentrate very hard to give a lecture, and monitor themselves carefully to make sure they're not talking too fast and that they are using appropriate and clear examples. More seasoned professors do not have to concentrate as much, and can talk clearly and engagingly, and keep an eye on students all at the same time. This example represents the transition from ________ thinking to ________ thinking as professors gain experience. a. automatic; controlled b. unconscious; conscious c. controlled; automatic d. random; systematic

c.

The first day of class, Maria absolutely loved her new social psychology professor, but Daniel couldn't stand her. If their interpretations were influenced by schemas, in all likelihood, Maria had heard ________, whereas Daniel had heard ________. a. that the professor was great; nothing about the professor b. nothing about the professor; that the professor was horrible c. that the professor was great; that the professor was horrible d. that the professor was an introvert; that the professor was an extravert

c.

Tom and Julius are both in line for a movie, but it sells out before either of them can get tickets. Tom is the last person in line, but Julius is directly behind the person who got the last ticket. Based on the idea of counterfactual reasoning, why will Julius be more upset than Tom? a. Julius wanted to see the movie more than Tom did. b. Julius got there earlier than Tom did, so he felt he deserved to see the movie. c. It was easier for Julius to imagine how he could have gotten a ticket. d. Julius won't be more upset than Tom.

c.

When Jana was chiding herself for losing her favorite necklace and reciting a list of all the things she could have done to prevent the loss, her grandmother said, "Now, Jana. Don't cry over spilled milk." In essence, Jana's grandmother was advising her to a. use base rates rather than the representativeness heuristic. b. avoid the self-fulfilling prophecy. c. stop engaging in counterfactual thinking. d. adjust her judgment farther away from the anchor.

c.

Which of the following decisions would you be least likely to make via automatic thinking? A decision about what a. brand of shampoo to buy. b. to wear to class today. c. major to pursue. d. to order for dinner.

c.

Which of the following provides the best example of the self-fulfilling prophecy? a. A lawyer asks you to describe the last fight you were in and the jury now believes you have a pronounced history of aggressive behavior, even though that was your only fight. b. A person tries to bring a shy person out of her shell by acting in a very friendly manner. c. Due to a rumor of a stock market crash, Wall Street has a significant drop when too many people take their money out of the market. d. You remember that your waitress friend prefers beer to wine, but mistakenly recall that your librarian friend prefers wine to beer.

c.

Winston is looking for a job, and he notices a man in the coffee shop wearing a tie and white shirt, working on his computer, while also flipping through the ads in the paper. Based on your text's discussion of how current goals can affect accessibility, how is Winston most likely to interpret the man's behavior? a. The man must be waiting on a client. b. The man must be depressed. c. The man must be looking for a job too. d. The man must be angry and tired.

c.

You are listening to a radio broadcast which describes a person who donated a kidney to a complete stranger in need of a transplant, when the phone rings and the fireman's association asks you to make a donation. The idea of priming suggests that you will a. be less likely to make a donation because your self-esteem has been threatened by realizing that you will never be as saintly as the person in the story. b. be more likely to make a donation because you feel in a good mood after hearing the heartwarming story. c. be more likely to make a donation because the schema of being charitable has been made more accessible. d. not be affected in your likelihood of making a donation by the radio broadcast.

c.

You have just arrived for your first year of college, but your roommate, Leah, is nowhere to be found. Your next-door neighbor, Tim, tells you that he's met Leah, and that she's really shy and introverted. If you wanted to counteract the self-fulfilling prophecy, what might you do when you meet Leah? a. Ask her if she likes to spend a lot of time alone. b. Ask her what books she's read lately. c. Try to get her to talk about fun activities she did in high school. d. Be quiet and shy yourself, so as not to intimidate her.

c.

You were supposed to make an important presentation at work, but you overslept, the bus was late, and you missed the meeting. If you are able to generate many ways in which to "undo" this terrible outcome, you will experience ________ emotional reaction. a. no b. a mild c. a strong d. an unclear

c.

You've rented the movie Sophie's Choice. There is a scene in which Sophie, a recent immigrant from Poland, is both surprised and amused that Americans have so many words for the concept "fast." She reports that in Polish, there is only one word. You've just read Chapter 3 (Social Cognition), and a friend asks you why the cultures differ in that regard. What would you say? a. English has borrowed more words from foreign languages than has Polish. b. Polish has different roots than does English. c. Poles don't have well-developed schemas for the concept "fast." d. Americans are generally better educated than Poles.

c.

Your roommate is interested in dating Chris, whom you know from a history class. Your roommate asks if you think Chris is generally a considerate person. You remember how Chris lent you notes when you missed class last week, and tell your roommate you think Chris is very considerate. In making your judgment, you have relied upon the ________ heuristic. a. representativeness b. base rate c. availability d. counterfactual thinking

c.

Zuzanna had a hard time remembering the names of streets when she traveled because they were in a different language. In addition, the buildings did not look like what she was used to, so she got lost several times. These problems emphasize the role of schemas in a. influencing what we notice and remember b. influencing how we feel about people c. determining what is accessible d. determining how much cognitive activity we need to use

c.

The government is putting together an ad campaign to encourage citizens to wash their hands more often during flu season. Based on your knowledge about fundamental motives and the use of denial, which slogan would work the best? a. "Wash your hands, or you WILL get sick." b. "People who do not wash their hands often are twenty times more likely to contract a serious communicable illness." c. "Washing your hands only takes a minute, and can keep you well." d. "I should have washed my hands" shown with a picture of a person in a hospital bed

c. "Washing your hands only takes a minute, and can keep you well."

Which of the following is true about the use of schemas? a. Schemas are an example of controlled thinking. b. When people have an incorrect schema, rarely do they act in a way to make it come true. c. Although schemas can lead to errors, they are a very useful way of organizing information about the world and filling in gaps in our knowledge. d. The schema we use is influenced only by what information is chronically accessible and not by our goals or by what has been primed recently.

c. Although schemas can lead to errors, they are a very useful way of organizing information about the world and filling in gaps in our knowledge.

Which of the following provides the best example of the self-fulfilling prophecy? a. A lawyer asks you to describe the last fight you were in and the jury now believes you have a pronounced history of aggressive behavior, even though that was your only fight. b. A person tries to bring a shy person out of her shell by acting in a very friendly manner. c. Due to a rumor of a stock market crash, Wall Street has a significant drop when too many people take their money out of the market. d. You remember that your waitress friend prefers beer to wine, but mistakenly recall that your librarian friend prefers wine to beer.

c. Due to a rumor of a stock market crash, Wall Street has a significant drop when too many people take their money out of the market.

In trying to make sense of the mass suicide in Jonestown, a Gestaltist would probably ___. a. Examine the external rewards and punishments used by Jim Jones. b. Consult a social psychologist. c. Ponder the subjective meaning of the act to Jim Jones's followers. d. Ask about the traumatic events in the lives of Jones's followers.

c. Ponder the subjective meaning of the act to Jim Jones's followers.

Rosenberg and his colleagues (1992) conducted a study that found that women who relied on the diaphragm or contraceptive sponge had fewer STDs than women who used condoms. The media jumped to the conclusion that condom use contributes to STDs. Of the limitations of this study, which one should have prevented the media from drawing a causal conclusion? a. Women who use public clinics are not representative of women in the U.S. b. The researchers did not study women who were on the pill. c. The choice to use condoms may have stemmed from STD contraction rather than the reverse. d. No such relationship between birth control device and STDs exists for men.

c. The choice to use condoms may have stemmed from STD contraction rather than the reverse.

George thinks of himself as an honest person until his brother reminds him that he's been known to keep extra change given to him by a cashier and to stock his home office for a sideline business with supplies taken from his job. George is now probably feeling a sense of discomfort known as

cognitive dissonance.

When Prya and Mohammed returned home after a night on the town, they found their baby crying because her diaper needed to be changed. The babysitter was sitting in front of the television, ignoring the baby's crying. Mohammed and Prya decided that the babysitter was callous and uncaring, and they swore that they would never hire him again. Mohammed and Prya's interpretation of the babysitter's actions was based on

common sense.

Correlational studies are used to....

compare two variables (1 vs. 2) but there is no definite answer for 'why'

When confronted with two contradictory beliefs about social behavior, such as "absence makes the heart grow fonder" and "out of sight, out of mind," a social psychologist is likely to

conduct an experiment to determine the conditions under which each belief holds true.

A personality psychologist would probably explain the suicides at Jonestown by focusing on the

conformist personality of the cult members.

research on people's use of the covariation model in making attributions suggests that people are less likely to use _________ information than Kelley's model predicts

consensus

people are cognitive misers.....

conserve cognitive resources as much as possible stingy w/sharing resources and ideas with others lazy people want to conserve mental resources

Social psychologists use the term ________ to describe the way in which people perceive, comprehend, and interpret their social worlds.

construal

Given the choice between distorting the world in order to enhance their self-esteem or viewing the world accurately, people often a.)refuse to make a choice, behaving erratically. b.) choose accuracy and thereby suffer from low self-esteem. c.)completely distort reality. d.) put a slightly different spin on things in order to feel good about themselves.

d

Jeremy is in love with Carol and views her temper as an endearing example of her "feistiness." Hercoworkers, however, interpret Carol's temper as rude and insensitive. The difference between Jeremy and thecoworkers illustrates the power of love to influence our a.) behaviors b.)influence attempts c.)relationships d.) construals

d

Which of the following people have fallen prey to the fundamental attribution error? a.)Guillermo, who explains his girlfriend's tears by saying, "She didn't get enough sleep last night." b.)Cindy, who explains her poor exam performance by pointing out how hard the questions were. c.) Dien, who points to an erratic driver and says, "Look at that! The roads are slick tonight." d.)Tim, who points to a person who fell down and says, "What a clumsy oaf!"

d

A teenager wants to borrow the family car to go to an out-of-town concert. Her parents resist the idea. Based on the results of the research study on the effects of generating different numbers of examples of behavior described in your text, the teenager in question is most likely to view her parents as stubborn and unreasonable when they say to her, a. "Give us five reasons why you should be allowed to borrow the car." b. "Tell us again what you would do if the car broke down." c. "How many other friends are going with you?" d. "Give us one reason why we shouldn't worry ourselves to death."

d.

According to the authors of your text, people raised in which of the following countries would be most likely to have a holistic thinking style? a. United States b. Italy c. Great Britain d. China

d.

Based on the study by Nisbett and his colleagues on statistical reasoning across four disciplines, which person would you count on to solve a statistical reasoning problem accurately? a. a physician b. a chemist c. a lawyer d. a psychologist

d.

Consider information from the authors of your text about the availability heuristic. If you wanted to improve customer satisfaction survey ratings for your company, which of the following would you ask customers in terms of their suggestions for improvement? a. Don't ask them for any criticisms or suggestions for improvement. b. Ask them to list their biggest criticism/suggestion. c. Ask them to list as many as they can. d. Ask them to list ten criticisms/suggestions.

d.

Iain is from Australia, where people drive on the left-hand side of the road. When he moved to the United States, his accident rate was higher than any of his friends'. Why might this be? a. They don't have drivers' education courses in Australia. b. Iain had no schema for driving behaviors. c. There is no speed limit in Australia. d. Driving schemas differ between the United States and Australia.

d.

Warren believes that Tom is an outgoing, gregarious person. "Whom did you hang out with this weekend?" Warren asks Tom. "Tell me about all of the fun things that you have planned for the summer," Warren continues. Although Tom is usually rather quiet and reserved, he responds to Warren in an outgoing, friendly manner. This is an example of a. the perseverance effect. b. the primacy effect. c. reconstructive memory. d. a self-fulfilling prophecy.

d.

When most Americans walk into a fast food restaurant chain, they know they should walk up to the counter to order, pay, and pick up their food, rather than sit down and wait to be served. This knowledge of how to act in such a situation is called a a. controlled thinking. b. stereotype. c. counterfactual thought. d. schema.

d.

Which of the following decisions would you be most likely to make via automatic thinking? A decision about a. whether to pursue treatment of cancer using radiation or chemotherapy. b. which graduate program to attend. c. which apartment to rent. d. which person to talk to at a cocktail party.

d.

You are taking a university course with a combined graduate and undergraduate enrollment. You know that there are many more undergraduate than graduate students enrolled in this course. There is a woman who looks to be about thirty sitting next to you. If you use only the base rate to guess her student status, you would guess that she is a(n) ________ student because ________. a. graduate; she's older b. undergraduate; you have a bias to expect undergraduates in your courses c. graduate; she fits your schema for graduate students d. undergraduate; there are more of them enrolled in the class

d.

Which of the following article titles most reflects basic as opposed to applied research? a. "Predictors of Tips Earned by Male and Female Servers in Restaurants" b. "The Effects of Instructions to Disregard Statements on Jury Decisions" c. "The Illusion of Control and Disaster Preparedness" d. "The Effect of Emotion Regulation on Memory for Personal Life Events"

d. "The Effect of Emotion Regulation on Memory for Personal Life Events"

Tiffany has a hard time trusting her friends because she believes they are irresponsible. Accordingly, when she makes dinner plans with one friend, she also makes backup plans with someone else, and she goes to one or the other. Her friends soon in turn begin to "blow off" their arrangements with Tiffany, because they are never sure whether she will show up. Tiffany thinks to herself, "See, I was right, my friends are irresponsible." Which of the following best explains why Tiffany made this conclusion? a. Accurate social perception due to controlled processes b. A self-fulfilling prophecy c. Holistic thinking d. Accurate social perception due to automatic processes

d. Accurate social perception due to automatic processes

Based on research by Becker and his colleagues (2007) about decoding emotions presented by the authors of your text, which person's expression would be the easiest to decode? a. Ann's expression of anger b. Blake's expression of happiness c. Robert's expression of disgust d. Amanda's expression of happiness

d. Amanda's expression of happiness

Consider information from the authors of your text about the availability heuristic. If you wanted to improve customer satisfaction survey ratings for your company, which of the following would you ask customers in terms of their suggestions for improvement? a. Don't ask them for any criticisms or suggestions for improvement. b. Ask them to list their biggest criticism/suggestion. c. Ask them to list as many as they can. d. Ask them to list ten criticisms/suggestions.

d. Ask them to list ten criticisms/suggestions.

Suppose you wanted your friend Stephan to feel like a more assertive person. According to research on ______, you should ask him to think of _____ times in the past when he acted in an unassertive manner. a. Representativeness heuristic; 12 b. Availability heuristic; 3 c. Representativeness heuristic; 3 d. Availability heuristic; 12

d. Availability heuristic; 12

Jennifer and Nate are walking along the street when they see a man walk out of a convenience store clutching a bag. The owner of the store runs out and shouts for the man to stop and come back. Jennifer immediately assumes that there has been a robbery, whereas Nate immediately assumes that the man forgot to get his change and that the store owner wants to give it to him. What is the best explanation for why Jennifer and Nate interpreted this event differently? a. Jennifer and Nate were engaged in controlled thinking that resulted in different assumptions about what was going on. b. Jennifer and Nate have different personalities. c. Jennifer and Nate fell prey to the self-fulfilling prophecy. d. Different schemas were accessible in Jennifer and Nate's minds, perhaps because they had different recent experiences that primed different schemas.

d. Different schemas were accessible in Jennifer and Nate's minds, perhaps because they had different recent experiences that primed different schemas.

A recent study found that the more time fathers spend with their children, the less likely they are to abuse their children. Which of the following is the most valid interpretation of this finding? a. Fathers who spend a lot of time with their children are more likely to develop loving relationships, reducing the likelihood that they will abuse their children. b. Many fathers who abuse their children feel guilty and thus avoid spending time with their children. c. There is some third variable, such as having an anti-social personality, that increases the likelihood that fathers will abuse their children and that they will spend little time with them. d. Either A, B, or C could be true, and one can't tell which, based on the results of the study.

d. Either A, B, or C could be true, and one can't tell which, based on the results of the study.

Which of the following situations is at odds with the self-esteem approach to understanding human behavior? a. At first, Jim can't stand his social psychology course, but he works hard all semester to get a good grade. At the end of the semester, when he evaluates the course, he gives it high marks. b. Susan drives 200 miles to buy a dress that most people would call incredibly ugly. She loves it and can't wait to wear it to the next party. c. Nguyen studied for years to pursue a career for which she was poorly suited. She works sixty hours a week in the sleet and hot sun. Still, she says that she is thrilled with her job. d. Jorge absolutely hates the camp he is attending. As part of the activities, he had to wear funny hats, and sing silly songs.

d. Jorge absolutely hates the camp he is attending. As part of the activities, he had to wear funny hats, and sing silly songs.

Which of the following is not a likely consequence of the human tendency to prefer self-justifying information to accurate information? a. People maintain their level of self-esteem. b. People become less likely to learn from their experiences. c. People may make faulty decisions about future behavior. d. People gain feedback that makes for better decisions.

d. People gain feedback that makes for better decisions.

Suppose you are trying to raise money for your favorite charity and you set up a table in the lobby of a campus building. Which of the following is likely to increase the likelihood that passersby will donate money? a. Give them a very light clipboard with information about your charity. b. Ask people to hold a cold bottle of water while they listen to what you have to say. c. Show them pictures of Japanese cities so that they think holistically. d. Spray some citrus-scented cleaning solution on the table.

d. Spray some citrus-scented cleaning solution on the table.

Which of the following is not a major drawback of observational methods? a. They cannot be used to study rarely occurring behaviors very well. b. They do not allow scientists to predict behavior. c. Archival research is at the mercy of the quality of the original reports. d. These methods cannot accurately describe behavior.

d. These methods cannot accurately describe behavior.

Which of the following questions is most likely to be asked by a social psychologist? a. Are some kinds of people more susceptible to recruitment into cults? b. Do some types of people make better leaders than others? c. Are some characteristics genetically determined? d. What situations cause people to behave rudely?

d. What situations cause people to behave rudely?

Which of the following is most likely to be studied by a social psychologist? a. differences in nonverbal behavior between members of different cultures b. the effects of social class on religious beliefs c. the effects of occupational segregation on income d. sex differences in self-concepts

d. sex differences in self-concepts

If a researcher were to use deception as part of her experimental procedure, when would she explain the purpose of this deception to her participants?

during the debriefing

In which ways is counterfactual thinking a controlled process? It is

effortful and conscious.

Julia eats granola bars and avoids chocolate bars, even though the granola bars contain 50 percent more fat and 15 percent more sodium than the chocolate bars. According to a social cognition approach, why might that be? Julia

failed to seek out all the relevant facts about the nutritional value of those snacks.

T/F if 10 people are all telling you the same thing you are more likely to conform publicly to their opinion than if just 5 people are telling you the same thing

false

T/F people in a sad mood are less likely to help others than are people in a neutral mood

false

T/F physically attractive people are usually seen as less intelligent than physically unattractive people

false

T/F promising and delivering rewards to people for doing an enjoyable activity will, in the long run, make hem enjoy the activity less

false

T/F the greater the cohesiveness or solidarity of a group, the better its decisions will be

false

Which of the following is false with regard to cross-cultural research in social psychology? Social psychologists

find cross-cultural studies of social behavior valuable because these allow the use of research methods deemed unethical in the United States.

According to Carol Dweck in her TED talk where she summarized her research on school children, receiving a grade of "not yet" when one is not meeting the standard requirements of a course, is an example of

growth mindset

An environment that promotes getting along well with others, being connected to one another, and seeing the larger context of social situations is most likely to promote _______ thinking. Correct! holistic

holistic

Nao was raised in Japan, and is walking through Times Square in New York City. Based on results from Nisbett's studies on cultural differences in thinking styles, which of the following would Nao be most likely to notice relative to people raised in the U.S.?

how the buildings are arranged relative to each other

When a person agrees to participate in an experiment in full awareness of the nature of the experiment, which has been provided prior to their participation, the participant has provided

informed consent.

theory

integrated principles that explain and predict observed events

Tina is very happy in her marriage. She is going to make ________ attributions about the positive behaviors of her spouse.

internal

By controlling all extraneous variables and by randomly assigning people to different experimental conditions, an experimenter can achieve high

internal validity

Shannon conducts an experiment to test her hypothesis that longer arguments cause couples to become angrier at each other. She randomly assigns couples to argue for either ten minutes or twenty minutes. After her assistant Ken runs the study, Ken tells Shannon that he allowed the couples who were really fighting hard to keep arguing longer than Shannon had originally stated. Shannon cringes, for she knows that the ________ of his study is very low.

internal validity

According to the Khan Academy presentation on Attribution Error and Culture, individualist cultures tend to attribute success to __________ factors, whereas collectivist cultures attribute ________ to external factors.

internal; success

Assume that you were a participant in the experiment conducted by Festinger and Carlsmith (1959) in which participants were paid either a large or small sum of money to tell an innocent stranger that the boring, tedious task you had just completed was really enjoyable and very interesting. Further assume that you were paid a large amount of money to tell the stranger that lie. In this situation, you would be most likely to

maintain your original assessment of the task as dull and boring.

When we commit the fundamental attribution error, we ________ the power of________.

overestimate; personal influence

In an effort to convince more women to get mammograms, public service ads downplay fear of breast cancer and discomfort of this cancer detection procedure. Instead such ads point to the benefits of early detection and consequent cure rate. According to the authors of your text, an ad would be designed this way because

overly frightened women might engage in denial about the odds that they would contract breast cancer.

Recall that Higgins, Rholes, and Jones (1977) had participants memorize a number of words before reading an ambiguous passage about a character named Donald. The results of this study showed that

people who memorized any positive words about Donald, whether or not they applied to the passage, viewed him more positively.

In the required video about over justification, it was stated that _________ rewards would help people to want to do the task again __________________.

performance; and do it better the next time.

The Kitty Genovese murder inspired research on bystander apathy. This example illustrates the usefulness of relying on ________ in formulating research hypotheses.

personal observations of everyday life

Sometimes when we encounter behavior that is unpleasant or unexpected, we assume that something about the person-and not the situation-caused the behavior. In this sense, lay people are most like

personality psychologists.

The most important take-home message about controlled thinking is that it

provides a check on automatic thinking.

Given the choice between distorting the world in order to enhance their self-esteem or viewing the world accurately, people often

put a slightly different spin on things in order to feel good about themselves.

The only way to be certain that the results of a survey represent the behavior of a particular population is to ensure that the respondents are ________ that population.

randomly selected from

Jason cannot stop thinking about the fact that because he turned in his term paper late, he missed getting an "A" in the class. He repetitively focuses on this negative part of the class; in other words, his counterfactual thinking has turned into

rumination.

When you meet your new roommate for the first time, he is wearing glasses, listening to classical music, and reading a copy of the New Yorker Review of Books. You think, "Well, maybe rooming with an intellectual this year will be good for me." You instantly categorized your roommate as an intellectual on the basis of your

schemas.

People often construe the world as they do in order to maintain a favorable image of themselves. This assumption underlies the ________ approach.

self-esteem

According to the authors of your text, people's evaluations of how good, competent, and decent they are is the definition of

self-esteem.

Recall that Rosenthal and Jacobson (1968) told teachers that some of their students were "bloomers." Consequently, those students actually performed better than students who were not labeled as bloomers. This finding suggests that

teachers' expectations were powerful in influencing the bloomers' behaviors.

hypothesis

testable predictions that allow theories to be tested

Nanami and April were playing in the den when April's mother entered the room and scolded them for making a mess. Nanami decided then and there that April's mother was a grouch. Nanami's inference is an example of

the fundamental attribution error.

Sheila shows up for a blind date with her hair disheveled and her clothes a mess. Her date, Jamal, thinks, "She must be a total slob!" Jamal's thought about Sheila is an example of

the fundamental attribution error.

A study was done in which people named as most helpful and least helpful by their friends were asked to make a contribution to a food drive. These students were also either given a reminder phone call the night before the food drive or not. As a social psychologist, you would predict that

the people given the reminder phone call would donate to the food drive, while those who were not

According to the definition of social psychology presented in your text, social psychology is the study of how ________ affect the thoughts, feelings, and behaviors of humans.

the presence of real or imagined others

Hindsight bias

the tendency to exaggerate one's ability to have foreseen an outcome after learning about it

Which of the following is the best definition of the fundamental attribution error?

the tendency to underestimate the power of social influence

which of the following questions is a behaviorist most likely to ask?

what are the external rewards in this situation

Case Study is only used to describe....

what happened

Dan Gilbert in his TED talk, " The Surprising Science of Happiness," found that synthetic happiness is

what we make when we do not get what we want.

dependent variable

what's being measured ie aggression

Your friend Jamie shows you the gift she bought for her mother's birthday. It's an atrociously ugly fake marble statue of an angel, with the saccharine words "My Mother's an Angel" sloppily lettered on the bottom. Jamie asks you what you think, and because her feelings are easily hurt, to spare her, you say, "It's wonderful! Maybe I'll get one for my mom!" In this case, you ________ experience dissonance because there is sufficient ________ justification for your action.

will not; external


Ensembles d'études connexes

Physical Activity & Health Final

View Set

Streetcar reading questions (midterm)

View Set

North Carolina Laws and Rules Insurance

View Set

Molgen practice questions - Exam 2

View Set

Ch 3 lesson 1 The Protestant Reformation

View Set

Honors Chem Unit 5: Periodic Table

View Set

Foundations of Education Parkay Test 6-8

View Set

Finance 302 Test 2 Questions & Answers

View Set

Ch 4/Week 9 Intro to Decision Structures

View Set

2020 IAAP CAP Exam Domain 1 - Organizational Communication (PO1)

View Set